Крок 2 - Медицина 2019 (буклет)

1 / 200
15-річна дівчина доставлена до лікаря зі скаргами на біль у суглобах з обмеженням рухів, ранкову скутість. Зі слів пацієнтки все розпочалося з болю та припухання обох колінних суглобів. Протягом 2-х років подібні симптоми поступово розвивались у гомілкових, ліктьових, п'ястково-фалангових та проксимальних міжфалангових суглобах. При обстеженні лімфаденопатія, гепатоспленомегалія та висип відсутні. При лабораторному дослідженні ШЗЕ - 42 мм/год, антинуклеарні антитіла (ANA) не виявлено, ревматоїдний фактор не виявлено. Який діагноз є найбільш імовірним у цієї пацієнтки? A 15-year-old girl was taken to the doctor with complaints of pain in the joints with restriction of movement, morning stiffness. According to the patient, it all started with pain and swelling of both knee joints. Over the course of 2 years, similar symptoms gradually developed in the tibia, elbow, metacarpal-phalangeal, and proximal interphalangeal joints. Lymphadenopathy, hepatosplenomegaly, and rash were absent during the examination. In the laboratory examination, the ESR was 42 mm/h, and no antinuclear antibodies (ANA) were detected , no rheumatoid factor was detected. What is the most likely diagnosis for this patient?

Системний червоний вовчак Systemic lupus erythematosus

Реактивний артрит Reactive arthritis

Анкілозивний спондилоартрит Ankylosing spondylitis

Ювенільний ідіопатичний артрит Juvenile idiopathic arthritis

Гостра ревматична лихоманка Acute rheumatic fever

2 / 200
Хворий 58-ми років звернувся до лікаря зі скаргою на невелике безболісне випинання в пахвинній ділянці протягом останнього місяця. Випинання не збільшується, з'являється при стоянні та зникає в положенні лежачи на спині. Утрудненого сечовипускання та закрепів немає. Пальпація черевної стінки безболісна. Ректальне дослідження патологічних змін не виявило. Що із перерахованого є найбільш доцільним наступним кроком лікаря у даній ситуації? A 58-year-old patient came to the doctor with a complaint of a small painless bulge in the inguinal area during the last month. The bulge does not increase, appears when standing and disappears in position lying on the back. Difficulty urinating and no constipation. Palpation of the abdominal wall is painless. Rectal examination did not reveal pathological changes. Which of the following is the most appropriate next step for the doctor in this situation?

Провести відкриту пластику із застосуванням сітки негайно Make an open plastic using a grid immediately

Направити на КТ-дослідження черевної порожнини та малого тазу Send for CT scan of abdomen and pelvis

Проінформувати пацієнта про ознаки защемлення і відпустити додому Inform the patient about signs of entrapment and let him go home

Провести тонкоголкову пункційну біопсію Perform fine-needle puncture biopsy

Накласти бандаж Apply bandage

3 / 200
На 8-му добу життя у новонародженого хлопчика з масою тіла 3500 г підвищується температура тіла та з'являється висип*. Напередодні дитина була дуже дратівлива та мала субфебрильну температуру, що поступово підвищувалася. Мати повідомила, що до зазначеного епізоду дитина була здорова та ніяких ліків не приймала. Температура тіла - 38,9°С, артеріальний тиск - 90/50 мм рт.ст., пульс - 160/хв., частота дихання - 17/хв., SpOo - 98% при кімнатному повітрі. При огляді виявлено висип у вигляді пухирів, що легко лопаються. Висип займає 60% поверхні тіла, навкруги рота, але не виявлено на слизових оболонках. Позитивний симптом Нікольського. Який діагноз є найбільш імовірним? On the 8th day of life, a newborn boy with a body weight of 3500 g has an increased body temperature and a rash appears*. The day before, the child was very irritable and had a low-grade fever , which gradually increased. The mother reported that the child was healthy before the specified episode and did not take any medications. Body temperature - 38.9°C, blood pressure - 90/50 mm Hg, pulse - 160/min, frequency breathing - 17/min., SpOo - 98% in room air. Examination reveals a rash in the form of blisters that burst easily. The rash covers 60% of the body surface, around the mouth, but is not found on the mucous membranes. Positive Nikolsky sign. What is the diagnosis is the most likely?

Кропив'янка Hives

Вроджена вітряна віспа Congenital chicken pox

Токсичний епідермальний некроліз Toxic epidermal necrolysis

Стафілококовий синдром 'обпеченої шкіри' (SSSS, синдром Ріттера) Scalded skin staphylococcal syndrome (SSSS, Ritter's syndrome)

Синдром Стівенса-Джонсона Stevens-Johnson syndrome

4 / 200
54-річний чоловік звернувся до лікаря зі скаргами на гострий біль за грудниною та задишку. В положенні лежачи біль посилюється. В анамнезі гіперхолестеринемія протягом 4-х років (лікує симвастатином). Задишку пов'язує із нещодавнім захворюванням на грип. При фізикальному обстеженні пульс - 92/хв., частота дихання - 16/хв.. артеріальний тиск - 140/80 мм рт.ст. Під час огляду помітно розширені шийні вени. Аускультативних змін в легенях та серці не виявлено. На ЕКГ у всіх відведеннях елевація сегменту ST. На рентгенограмі збільшення тіні серця. Який діагноз є найбільш імовірним? A 54-year-old man went to the doctor with complaints of sharp pain behind the sternum and shortness of breath. The pain intensifies when lying down. He has a history of hypercholesterolemia for 4 years (treats (simvastatin). Shortness of breath is associated with a recent flu illness. During physical examination, the pulse is 92/min., the respiratory rate is 16/min., blood pressure is 140/80 mm Hg. During the examination, the neck veins are noticeably dilated. Auscultatory changes in the lungs and heart were not detected. ST segment elevation in all leads on the ECG. An increase in the shadow of the heart on the X-ray. What is the most likely diagnosis?

Гострий інфаркт міокарда Acute myocardial infarction

Міокардит Myocarditis

Тромбоемболія легеневої артерії (ТЕЛА) Thromboembolism of the pulmonary artery (PEA)

Перикардит Pericarditis

5 / 200
Жінка 36-ти років звернулася до лікаря зі скаргами на часте сечовипускання, біль у лівому боці, нудоту та підвищену температуру. При фізикальному обстеженні температура тіла - 39,4°С, артеріальний тиск - 120/80 мм рт.ст., пульс - 89/хв., частота дихання - 18/хв., SрО2 - 98% при кімнатному повітрі. При пальпації болючість у лівому костовертебральному куті та лівому боці. Який наступний крок лікаря у веденні пацієнтки буде найбільш доречним? A 36-year-old woman consulted a doctor with complaints of frequent urination, pain in the left side, nausea, and high temperature. During a physical examination, the body temperature was 39.4 °C, blood pressure - 120/80 mm Hg, pulse - 89/min, respiratory rate - 18/min, SрО2 - 98% in room air. On palpation, tenderness in the left costovertebral corner and left side. What will the next step of the doctor in the management of the patient be the most appropriate?

Розпочати внутрішньовенне введення цефтриаксону Start intravenous ceftriaxone

Призначити триметоприм-сульфаметоксазол на 10 днів Prescribe trimethoprim-sulfamethoxazole for 10 days

Здати загальний аналіз сечі та виконати бактеріологічне дослідження сечі Pass a general urinalysis and perform a bacteriological examination of urine

Рекомендувати пройти тест на вагітність Recommend to take a pregnancy test

Призначити нітрофурантоїн перорально на 10 днів Prescribe nitrofurantoin orally for 10 days

6 / 200
Чоловік 27-ми років звернувся до лікаря зі скаргами на біль та набряк у II та III пальцях лівої стопи, почервоніння та гнійні виділення з очей, різь та біль при виділенні сечі. Зі слів пацієнта має декількох статевих партнерів, зрідка користується презервативами. При фізикальному обстеженні лікарем виявлено двосторонній кон'юнктивіт та дактиліт пальців стопи. Обстеження інших органів і систем патології не виявило. У загальному аналізі крові ШЗЕ - 40 мм/год., біохімічний аналіз без відхилень. На рентгенограмах пальців стопи збільшення об'єму м'яких тканин, звуження суглобової щілини. Який діагноз є найбільш імовірним? A 27-year-old man consulted a doctor with complaints of pain and swelling in the II and III toes of the left foot, redness and purulent discharge from the eyes, tearing and pain in urinary excretion. According to the patient, he has several sexual partners, occasionally uses condoms. During the physical examination, the doctor revealed bilateral conjunctivitis and dactylitis of the toes. The examination of other organs and systems did not reveal pathology. In the general blood test, ESR - 40 mm/h. biochemical analysis without abnormalities. X-rays of the toes show an increase in the volume of soft tissues, narrowing of the joint space. What diagnosis is most likely?

Реактивний артрит Reactive arthritis

Сифіліс Syphilis

Синдром Шегрена Sjogren's syndrome

Подагра Gout

Ревматоїдний артрит Rheumatoid arthritis

7 / 200
Жінка 32-х років звернулася до лікаря жіночої консультації зі скаргами на хронічний тазовий біль, який підсилюється під час менструацій, диспареунію, кровомазання напередодні та після менструації. Остання менструація 3 тижні потому. При огляді у дзеркалах: на шийці матки 2 кісти діаметром 3 і 5 мм синьо-багрового кольору, з яких виділяється темно-коричнева рідина. При бімануальному дослідженні: тіло матки шароподібної форми, збільшене до 6 тижнів вагітності, болісне при пальпації. Придатки з обох боків без особливостей. Лікаря проінформовано, що у найближчий час народження дитини не планується. Яка тактика лікування цієї пацієнтки є найбільш доречною? A 32-year-old woman turned to a gynecologist with complaints of chronic pelvic pain that worsens during menstruation, dyspareunia, bleeding before and after menstruation. Last menstruation 3 weeks later. On examination in mirrors: on the cervix 2 cysts with a diameter of 3 and 5 mm of blue-purple color, from which a dark brown liquid is secreted. On bimanual examination: the body of the uterus is spherical in shape, enlarged to 6 weeks of pregnancy, painful on palpation . Appendages on both sides without any features. The doctor was informed that the birth of a child is not planned in the near future. What treatment tactics for this patient is the most appropriate?

Хірургічне втручання Surgery

Призначення андрогенів Androgens prescription

Призначення антагоністів гонадотропін-рилізинг гормону Prescription of gonadotropin-releasing hormone antagonists

Призначення комбінованих оральних контрацептивів Prescription of combined oral contraceptives

Контрольована гіперстимуляція яєчників Controlled ovarian hyperstimulation

8 / 200
Жінка 55-ти років скаржиться на пронос, лущення і пігментацію відкритих ділянок тіла (шиї, рук та стоп), дратівливість і занепокоєння. З анамнезу відомо, що основним продуктом харчування для неї є кукурудза. Салатні овочі і бобові жінка вживає рідко, м'ясо і рибу в їжу не вживає зовсім. Про яке захворювання йдеться? A 55-year-old woman complains of diarrhea, peeling and pigmentation of open areas of the body (neck, hands and feet), irritability and anxiety. From the anamnesis, it is known that the main the food product for her is corn. The woman rarely eats salad vegetables and legumes, she does not eat meat and fish at all. What disease is it about?

Псоріаз Psoriasis

Цинга Scurvy

Бері-бері Beri-beri

Пелагра Pellagra

Дерматит Свіфта Swift's dermatitis

9 / 200
Чоловік 50-ти років звернувся до лікаря зі скаргами на біль в стегні після падіння на сходах. Піднятись та ходити не може. Права нога коротша за ліву на 5 см. ступня ротована назовні. Пульс на правій тильній артерії стопи задовільний. Стегно збільшене в об'ємі, деформоване, в середній третині болюча пальпація. Виявляється патологічна рухомість, крепітація. Який діагноз є найбільш імовірним? A 50-year-old man went to the doctor complaining of hip pain after falling down the stairs. He can't get up and walk. The right leg is 5 cm shorter than the left . the foot is rotated outward. The pulse on the right posterior artery of the foot is satisfactory. The thigh is enlarged, deformed, painful palpation in the middle third. Pathological mobility, crepitus is detected. What diagnosis is most likely?

Забій сідничного нерва Contusion of sciatic nerve

Вивих правого стегна Dislocation of right hip

Хибний суглоб False joint

Перелом діафіза стегнової кістки Fracture of the diaphysis of the femur

Закритий перелом кісток миски Closed pelvic fracture

10 / 200
У чоловіка 20-ти років невдовзі після екстракції зуба розпочалася сильна кровотеча. З анамнезу відомо, що пацієнт хворіє на гемофілію А. При лабораторному дослідженні у загальному аналізі крові: еритроцити - 3,2•1012/л, Hb- 98г/л, кольоровий показник - 0.92, лейкоцити - 7,4 • 10[9]/л, тромбоцити - 240 • 109/л, швидкість осідання еритроцитів - 11 мм/год. Час кровотечі за Дюке - 3 хв., час зсідання крові за Лі-Уайтом - 20 хв. Яка фармакотерапія є найбільш ефективною для цього пацієнта? A 20-year-old man started bleeding heavily shortly after tooth extraction. From the anamnesis, it is known that the patient suffers from hemophilia A. During the laboratory examination in the general blood test: erythrocytes - 3.2•1012/l, Hb - 98g/l, color index - 0.92, leukocytes - 7.4 • 10[9]/l, platelets - 240 • 109/l, erythrocyte sedimentation rate - 11 mm/h . Bleeding time according to Duke - 3 minutes, blood clotting time according to Lee-White - 20 minutes. Which pharmacotherapy is the most effective for this patient?

Рекомбінантний XI фактор Recombinant XI factor

Рекомбінантний X фактор О Recombinant X Factor O

Тромбоцитарна маса Platelet mass

Рекомбінантний VIII фактор Recombinant factor VIII

Еритроцитарна маса Erythrocyte mass

11 / 200
Мати привела до лікаря хлопчика 7-ми років на плановий медичний огляд. Вона скаржиться, що дитина дуже багато часу проводить за ґаджетами та дуже мало грає з однолітками на свіжому повітрі. Фізичний розвиток відповідає віку дитини. Згідно з карткою профілактичних щеплень до 6-ти років включно отримав усі щеплення, передбачені Національним календарем. При фізикальному обстеженні температура тіла *–37,1°С, пульс - 88/хв., артеріальний тиск - 110/80 мм рт.ст., частота дихання - 16/хв. Які подальші дії по веденню хлопчика будуть найбільш доречними? The mother brought a 7-year-old boy to the doctor for a routine medical examination. She complains that the child spends a lot of time on gadgets and plays very little with his peers outdoors air. Physical development corresponds to the child's age. According to the preventive vaccination card, up to and including the age of 6, he received all the vaccinations provided for by the National Calendar. During the physical examination, the body temperature *–37.1°С, pulse - 88/min., blood pressure - 110/80 mm Hg, respiratory rate - 16/min. What further actions for the boy's management would be most appropriate?

Призначити оральну поліомієлітну вакцину (ОПВ) Prescribe oral polio vaccine (OPV)

Провести пробу Манту Perform Mantoux test

Провести бесіду щодо важливості активного способу життя Have a conversation about the importance of an active lifestyle

Призначити вакцину БЦЖ Prescribe BCG vaccine

Призначити вакцину КПК Prescribe PDA vaccine

12 / 200
Чоловік 27-ми років, спортсмен, під час тренування раптово втратив свідомість. Пульс і артеріальний тиск не визначаються. Дихання відсутнє. Тони серця не вислуховуються. Зіниці широкі. На ЕКГ: поліморфна шлуночкова тахікардія. Розпочато компресію грудної клітки. Який наступний крок буде найбільш доречним? A 27-year-old man, an athlete, suddenly lost consciousness during training. Pulse and blood pressure are not detected. Breathing is absent. Heart sounds are not heard. Pupils are wide. On the ECG: polymorphic ventricular tachycardia. Chest compressions are started. What would be the most appropriate next step?

Вагусні проби Vagus samples

Внутрішньовенне введення лідокаїну Intravenous administration of lidocaine

Електрична дефібриляція Electrical defibrillation

Внутрішньовенне введення дигоксину Intravenous digoxin

Черезстравохідна кардіостимуляція Transesophageal cardiac stimulation

13 / 200
У хірургічний кабінет звернувся хлопець 17-ти років, зі скаргами на наявність рубаної рани правої стопи. Із анамнезу відомо, що травму отримав сокирою під час роботи по господарству. Усі щеплення отримував відповідно до віку за Національним календарем профілактичних щеплень. Об'єктивно: температура тіла - 36.8°С, пульс - 68/хв., артеріальний тиск - 120/75 мм рт.ст. Який наступний крок лікаря буде найбільш доречними? A 17-year-old boy came to the surgery with complaints of a cut wound on his right foot. From the anamnesis, it is known that he was injured with an ax while working on the farm. I received all vaccinations according to age according to the National calendar of preventive vaccinations. Objectively: body temperature - 36.8°C, pulse - 68/min., blood pressure - 120/75 mm Hg. What next step of the doctor would be most appropriate?

Ввести правцевий анатоксин з дифтерійним анатоксином Enter tetanus toxoid with diphtheria toxoid

Ввести правцевий анатоксин Enter tetanus toxoid

Ввести протиправцеву сироватку Enter tetanus serum

Ввести правцевий анатоксин та протиправцевий імуноглобулін Inject tetanus toxoid and anti-tetanus immunoglobulin

Динамічне спостереження Dynamic monitoring

14 / 200
Хвора 63-х років оперована з приводу багатовузлового еутиреоїдного зобу великих розмірів. З технічними труднощами вимушено виконана субтотальна резекція обох часток щитоподібної залози. На 4-й день після операції з'явилися судоми м'язів обличчя і верхніх кінцівок, біль в животі. Позитивні симптоми Хвостека і Труссо. Чим найімовірніше зумовлений такий стан хворої? A 63-year-old patient was operated on for a large multinodular euthyroid goiter. With technical difficulties, a subtotal resection of both lobes of the thyroid gland was forced to be performed. On the 4th day after the operation muscle spasms of the face and upper limbs appeared, pain in the abdomen. Positive symptoms of Khvostek and Trousseau. What is the most likely cause of this condition of the patient?

Післяопераційний гіпотиреоз Postoperative hypothyroidism

Недостатність паращитоподібних залоз Parathyroid insufficiency

Трахеомаляція Tracheomalacia

Пошкодження зворотного нерву Injury of the recurrent nerve

Тиреотоксичний криз Thyrotoxic crisis

15 / 200
До ортопеда звернулись батьки 3-місячної дитини зі скаргами на асиметрію шкірних складок правого стегна. Під час огляду виявлено: різна кількість складок шкіри на правому та лівому стегні, їх асиметрія, обмеження розведення правої нижньої кінцівки зігнутої під кутом 90° в кульшовому та колінному суглобах. Який діагноз є найбільш імовірним? The parents of a 3-month-old child turned to the orthopedist with complaints about the asymmetry of the skin folds of the right thigh. During the examination, it was found: a different number of skin folds on the right and left thigh, their asymmetry, restriction of extension of the right lower extremity bent at an angle of 90° in the hip and knee joints. What diagnosis is most likely?

Ушкодження кульшового суглоба під час пологів Injury of the hip joint during childbirth

Вроджений вивих стегна Congenital hip dislocation

Контрактура кульшового суглоба Hip joint contracture

Дисплазія кульшового суглоба Hip dysplasia

16 / 200
Хворий 60-ти років скаржиться на інтенсивні болі стискаючого характеру, локалізовані за грудниною з іррадіацією у ліву лопатку та нижню щелепу. Біль тривалий, не знімається нітрогліцерином. Об'єктивно: холодний піт. Хворий збуджений. Аускультативно:тахікардія, приглушеність тонів. Артеріальний тиск - 100/70 мм рт.ст. ЕКГ- комплекси QS та підйом S-T вище ізолінії в I, aVL, V1, V2,VЗ, V4. Який діагноз є найбільш імовірним? A 60-year-old patient complains of intense squeezing pains localized behind the sternum with radiation to the left scapula and lower jaw. The pain is long-lasting and is not relieved by nitroglycerin. Ob' Objectively: cold sweat. The patient is excited. Auscultatively: tachycardia, muffled tones. Blood pressure - 100/70 mm Hg. ECG - QS complexes and ST elevation above the isoline in I, aVL, V1, V2, VЗ, V4. What is the diagnosis is the most likely?

Тромбоемболія легеневої артерії Thromboembolism of the pulmonary artery

Лівосторонній плеврит Left-sided pleurisy

Інфаркт міокарда Myocardial infarction

Стенокардія Принцметала Prinzmetal Angina

Міжреберна невралгія Intercostal neuralgia

17 / 200
У пацієнта 64-х років протягом останніх 2 місяців з'явилося відчуття утрудненого ковтання твердої їжі з затримкою її посередині стравоходу. Втрата ваги - 4 кг. Батько хворого помер від раку шлунка. Запідозрено рак стравоходу. За допомогою якого дослідження найбільш імовірно можна верифікувати цю патологію? A 64-year-old patient had a feeling of difficulty swallowing solid food with its retention in the middle of the esophagus during the last 2 months. Weight loss - 4 kg. The patient's father died from stomach cancer. Esophageal cancer is suspected. Which test is most likely to verify this pathology?

ФЕГДС з біопсією FEGDS with biopsy

Дихальний уреазний тест Breathing urease test

Внутрішньостравохідна рН-метрія Intraesophageal pH-metry

Рентгенконграстне дослідження стравоходу і шлунка X-ray contrast examination of esophagus and stomach

Комп'ютерна томограма грудної клітки Computer tomography of the chest

18 / 200
Хворий 22-х років звернувся до лікаря зі скаргами на появу білих плям на тлі засмаглої шкіри. На шкірі тулуба множинні гіпопігментовані плями, при пошкрябуванні спостерігається мукоподібне лущення. Проба Бальцера позитивна. Яке захворювання можна припустити? A 22-year-old patient turned to the doctor with complaints about the appearance of white spots on the background of tanned skin. There are multiple hypopigmented spots on the skin of the body, and when scraped, a muco-like peeling is observed. Sample Balzera is positive. What disease can be assumed?

Рожевий лишай Pink lichen

Вітиліго Vitiligo

Висівкоподібний лишай Lichen bran

Лейкодерма Leukoderma

Склеродермія Scleroderma

19 / 200
Хворий 43-х років страждає на виразкову хворобу 12-палої кишки протягом 8-ми років. Під час роботи на садовій дільниці відчув різкий біль у епігастральній дільниці, слабкість, запаморочення. Об'єктивно: шкіра бліда, пульс - 102/хв., слабкий, артеріальний тиск - 100/60 мм рт.ст., живіт не приймає участі у акті дихання, напружений, болісний у епігастрії, печінкова тупість відсутня. Яке захворювання є найбільш імовірним? A 43-year-old patient has been suffering from duodenal ulcer disease for 8 years. While working in the garden area, he felt sharp pain in the epigastric area, weakness , dizziness. Objectively: the skin is pale, the pulse is 102/min., weak, blood pressure is 100/60 mm Hg, the stomach does not take part in the act of breathing, tense, painful in the epigastrium, there is no liver dullness. the disease is most likely?

Кишкова непрохідність Intestinal obstruction

Загострення панкреатиту Exacerbation of pancreatitis

Шлункова кровотеча Gastric bleeding

Перфорація виразки Ulcer perforation

Інфаркт міокарда Myocardial infarction

20 / 200
У заводській їдальні виникло харчове отруєння, клініка якого вказувала на стафілококову етіологію. Захворіло 15 чоловік. Які матеріали від хворого необхідно надіслати на дослідження у лабораторію аби підтвердити харчове отруєння? Food poisoning occurred in the factory canteen, the clinic of which indicated a staphylococcal etiology. 15 people fell ill. What materials from the patient should be sent to the laboratory for research to confirm food poisoning?

Кров (клінічний аналіз) Blood (clinical analysis)

Сечу Urine

Слину Saliva

Кров на гемокультури Blood for blood culture

Блювотні маси хворого Vomiting masses of the patient

21 / 200
Жінка 22-х років звернулася до лікаря з метою планування вагітності. В анамнезі цукровий діабет протягом двох років. Діабетичних ускладнень при опитуванні та огляді не виявлено. Глікемія натще в межах 6,0-7,0 ммоль/л. Який аналіз серед перерахованих слід призначити, щоб адекватно оцінити контроль глікемії? A 22-year-old woman consulted a doctor in order to plan a pregnancy. She has a history of diabetes for two years. Diabetic complications were not detected during the survey and examination. Fasting blood glucose in between 6.0-7.0 mmol/l. Which analysis among the listed should be prescribed to adequately assess glycemic control?

Рівень глікозильованого (глікованого) гемоглобіну (НbАІс) The level of glycosylated (glycated) hemoglobin (HbAIs)

Глюкозуричний профіль Glucosuric profile

Глікемічний профіль Glycemic Profile

Рівень С-пеитиду C-peitide level

Постпрандіальна глікемія Postprandial glycemia

22 / 200
Проведено дослідження з метою визначення зв'язку між споживанням мікронутрієнтів та захворюванням на катаракту серед людей похилого віку. Участь у дослідженні взяли 2900 людей у віці від 49 до 97 років. Кожний учасник заповнював опитувальник частоти споживання певних харчових продуктів та проходив офтальмоскопію з фотографуванням кришталика. У результаті дослідження, було виявлено, що достатня кількість у раціоні вітаміну А, рибофлавіну. ніацину та тіаміну попереджують виникнення ядерної катаракти. Який з перерахованих варіантів найбільш точно описує вид проведеного клінічного дослідження? A study was conducted to determine the relationship between micronutrient intake and cataracts in the elderly. The study involved 2,900 people aged 49 to 97 Each participant filled out a questionnaire about the frequency of consumption of certain food products and underwent an ophthalmoscopy with lens photography. As a result of the study, it was found that sufficient amounts of vitamin A, riboflavin, niacin and thiamine in the diet prevent the occurrence of nuclear cataracts. Which of the listed options most accurately describes the type conducted clinical research?

Рандомізоване клінічне дослідження Randomized clinical trial

Опис серії клінічних випадків Description of a series of clinical cases

Дослідження 'випадок-контроль' Case-control study

Крос-секціональне поперечне дослідження Cross-sectional cross-sectional study

Проспективне когортне дослідження Prospective cohort study

23 / 200
Спортсмен 26-ти років впав з упором на правий плечовий суглоб. Зазначає різкий біль, обмеження пасивних рухів, активні рухи в плечовому суглобі неможливі. Відведену і зігнуту у плечовому суглобі травмовану руку підтримує здоровою. Об'єктивно: в ділянці дельтоподібного м'яза западання, під шкірою різко випинається акроміальний відросток лопатки, голівка плечової кістки пальпується під дзьобоподібним відростком. Який найбільш імовірний клінічний діагноз? A 26-year-old athlete fell with an emphasis on the right shoulder joint. He notes sharp pain, limitation of passive movements, active movements in the shoulder joint are impossible. Abducted and bent shoulder the joint keeps the injured hand healthy. Objectively: in the area of the deltoid muscle, the acromial process of the scapula protrudes sharply under the skin, the head of the humerus can be palpated under the beak-like process. What is the most likely clinical diagnosis?

Перелом голівки плечової кістки Fracture of humeral head

Вивих плеча Shoulder dislocation

Відрив великого пагорбка плечової кістки Detachment of the humeral humerus

Перелом хірургічної шийки плеча Fracture of the surgical neck of the shoulder

Перелом анатомічної шийки плеча Fracture of the anatomical neck of the humerus

24 / 200
Чоловік 64-х років знаходиться на амбулаторному лікуванні з приводу ішемічної хвороби серця, дифузного кардіосклерозу, постійної форми фібриляції передсердь, серцевої недостатності III функціонального класу. Фармакотерапія складається з кордарону 2 рази на день, торасеміду через день, триметазидину 2 рази на день. Сімейний лікар з метою профілактики тромбоутворення рекомендував хворому прийом варфарину (3 мг на день). Який метод контролю ефективності та безпеки антикоагулянтної терапії в даному випадку буде найбільш доречним? A 64-year-old man is undergoing outpatient treatment for ischemic heart disease, diffuse cardiosclerosis, permanent form of atrial fibrillation, heart failure of the III functional class. Pharmacotherapy consists of cordarone 2 times a day, torasemide every other day, trimetazidine 2 times a day. In order to prevent thrombosis, the family doctor recommended the patient to take warfarin (3 mg per day). What method of monitoring the effectiveness and safety of anticoagulant therapy would be most appropriate in this case?

Час згортання крові за Лі-Уайтом Lee-White blood clotting time

Кількість тромбоцитів Number of platelets

Рівень фібриногену Fibrinogen level

Швидкість зсідання еритроцитів Red blood cell sedimentation rate

Міжнародне нормалізоване співвідношення International Normalized Ratio

25 / 200
Дитині 8 років. Скарги на температуру тіла 39,3°С, головний біль, блювання. 1-й день захворювання. Шкіра чиста. Зів гіперемований. Язик обкладений. Кон'юнктивіт. Склерит. Слабко виражена ригідність потиличних м'язів. Позитивний симптом Керніга справа. Ліквор: цитоз - 340/мкл, лімфоцити - 87%, нейтрофіли - 13%, білок - 140 мг/л. Яким буде найімовірніший діагноз? The child is 8 years old. Complaints about the body temperature of 39.3°C, headache, vomiting. 1st day of the disease. The skin is clean. The throat is hyperemic. The tongue is coated . Conjunctivitis. Scleritis. Mild rigidity of the occipital muscles. Positive Kernig's sign on the right. CSF: cytosis - 340/μL, lymphocytes - 87%, neutrophils - 13%, protein - 140 mg/L. What would be the most likely diagnosis?

Пухлина мозку Brain tumor

Арахноїдит Arachnoidite

Гнійний менінгіт Suppurative meningitis

Нейротоксикоз Neurotoxicosis

Серозний менінгіт Serous meningitis

26 / 200
Чоловік 64-х років надійшов у відділення невідкладної допомоги зі скаргами на набряки нижніх кінцівок, попереку та передньої черевної стінки. В анамнезі хронічний бронхіт з бронхоектазами. При фізикальному обстеженні температура тіла - 37,2°С, артеріальний тиск - 110/75 мм рт.ст., пульс - 82/хв., частота дихання - 19/хв. При лабораторному дослідженні у загальному аналізі сечі (ЗАС):питома вага - 1025, білок - 9,9 г/л, лейкоцити - 2-3 у полі зору, еритроцити - 1-2 у полі зору, циліндри - немає. Добова протеїнурія - 11,4 г/добу. У біохімічному аналізі крові загальний білок - 52 г/л, альбуміни - 30 г/л, холестерин - 9,6 ммоль/л. Який тип ураження нирок у цього хворого є найбільш імовірним? A 64-year-old man was admitted to the emergency department with complaints of swelling of the lower extremities, lower back, and anterior abdominal wall. He has a history of chronic bronchitis with bronchiectasis. During physical examination body temperature - 37.2°C, blood pressure - 110/75 mm Hg, pulse - 82/min, respiratory rate - 19/min. During laboratory research in the general analysis of urine (Urinalysis): specific gravity - 1025 , protein - 9.9 g/l, leukocytes - 2-3 in the field of vision, erythrocytes - 1-2 in the field of vision, cylinders - none. Daily proteinuria - 11.4 g/day. In the biochemical blood analysis, total protein - 52 g/l, albumin - 30 g/l, cholesterol - 9.6 mmol/l. What type of kidney damage is the most likely in this patient?

Пієлонефрит, сечовий синдром Pyelonephritis, urinary syndrome

Тубуло-інтсрстиціальний нефрит, нефротичний синдром Tubulo-interstitial nephritis, nephrotic syndrome

Гломерулонефрит, нефритичний синдром Glomerulonephritis, nephritic syndrome

АА амілоїдоз, нефротичний синдром AA amyloidosis, nephrotic syndrome

Діабетична нефропатія, нефритичний синдром Diabetic nephropathy, nephritic syndrome

27 / 200
1-місячна дитина доставлена матір'ю до відділення невідкладної допомоги зі скаргами на блювання з домішками жовчі після годування та випорожнення із домішками крові. Мати народила дитину у 30 років на 32 тижні гестації. При народженні стан дитини на 5-й хвилині 6 балів за шкалою Апгар, маса тіла - 1300 г. При фізикальному дослідженні дитина млява, значне здуття живота. Який наступний крок лікаря буде найбільш доречним? A 1-month-old child was brought to the emergency department by his mother complaining of bilious vomiting after feeding and bloody stools. The mother gave birth to a child at the age of 30 at the 32nd week of gestation. At birth, the child's condition at the 5th minute was 6 points on the Apgar scale, body weight - 1300 g. During the physical examination, the child is lethargic, with significant abdominal distension. What next step of the doctor would be most appropriate?

Направити на езофагогастроскопію Send to esophagogastroscopy

Направити до дитячого хірурга Refer to pediatric surgeon

Призначити емпіричну антибіотикотерапію Prescribe empiric antibiotic therapy

Рекомендувати перейти на безлактозне штучне вигодовування Recommend switching to lactose-free artificial feeding

Направити на рентгенографію черевної порожнини Send for abdominal X-ray

28 / 200
Хлопчик 12-ти років звернувся до хірурга зі скаргою на нестерпний біль кінчика II пальця правої кисті. Лікарем з'ясовано, що 4 дні тому він вколов це місце дротом. Протягом останніх трьох днів з'явився різкий пульсуючий біль, набряк тканин та гіперемія дистальної фаланги пальця. Об'єктивно: температура тіла – 38,1°С. Який з перерахованих діагнозів є найбільш імовірним? A 12-year-old boy turned to the surgeon complaining of excruciating pain in the tip of the II finger of the right hand. The doctor found out that 4 days ago he pricked this place with a wire . During the last three days, sharp throbbing pain, tissue swelling and hyperemia of the distal phalanx of the finger appeared. Objectively: body temperature - 38.1°C. Which of the listed diagnoses is the most likely?

Целюліт Cellulite

- -

Панарицій Panaricius

Еризипелоїд Erysipeloid

Пароніхія Paronychia

29 / 200
Чоловіку 42-х років, з діагнозом спастичний парапарез призначено міорелаксант тизанідин за схемою поступового збільшення дози. Хворий порушив рекомендацію і вжив відразу добову дозу, після чого відчув загальну слабість, запаморочення, зниження сили в ногах та на короткий час втратив свідомість. При фізикальному обстеженні артеріальний тиск -75/55 мм рт.ст., пульс - 69/хв., температура тіла - 36,6°С, загальна м'язова гіпотонія. Який невідкладний стан найбільш імовірно виник у хворого? A 42-year-old man with a diagnosis of spastic paraparesis was prescribed the muscle relaxant tizanidine according to the scheme of gradually increasing the dose. The patient violated the recommendation and immediately took the daily dose, after which he felt general weakness , dizziness, decreased strength in the legs and lost consciousness for a short time. During the physical examination, blood pressure - 75/55 mm Hg, pulse - 69/min, body temperature - 36.6°С, general muscular hypotonia What emergency condition most likely occurred in the patient?

Симпато-адреналовий криз Sympatho-adrenal crisis

Панічна атака Panic attack

Транзиторна ішемічна атака Transient ischemic attack

Синкопальний стан Syncopal state

Ваго-інсулярний криз Vago-insular crisis

30 / 200
У регулювальника дорожнього руху з'явились скарги на головний біль, запаморочення. порушення сну, зниження пам'яті та уваги, задишку, біль у ділянці серця. У крові під час лабораторного обстеження виявлено підвищений вміст карбоксигемоглобіну. Який чинник навколишнього середовища найімовірніше міг спричинити такі зміни в організмі? The traffic controller complained of headache, dizziness, sleep disturbance, decreased memory and attention, shortness of breath, pain in the heart area. In the blood an increased content of carboxyhemoglobin was detected during a laboratory examination. What environmental factor could most likely cause such changes in the body?

Діоксид азоту Nitrogen dioxide

3,4-безпірен 3,4-bezpyrene

Оксид азоту Nitrogen oxide

Оксид вуглецю Carbon monoxide

Діоксид вуглецю Carbon dioxide

31 / 200
Хлопчик 5-ти років прийшов з матір'ю на прийом до лікаря зі скаргами на припухлість обличчя, біль у вухах та підвищену температуру протягом останніх 5 днів. Через релігійні переконання матері вакцинація дитини неповна. Під час фізикального обстеження виявлено двосторонній, болючий набряк у ділянці жувального м'яза та гарячку 39,3°С. Що з перерахованого буде найбільш доцільно призначити цьому пацієнту? A 5-year-old boy came with his mother to a doctor's appointment with complaints of facial swelling, ear pain, and high temperature for the past 5 days. Due to religious the mother's belief that the child's vaccination is incomplete. During the physical examination, bilateral, painful swelling in the area of the masticatory muscle and a fever of 39.3°C were revealed. Which of the following would be most appropriate to prescribe for this patient?

Цефтриаксон Ceftriaxone

Парацетамол Paracetamol

Імуноглобулін людини нормальний Human immunoglobulin is normal

Ацикловір Acyclovir

Пеніцилін Penicillin

32 / 200
Дівчинка 6-ти років надійшла зі скаргами на підвищення температури тіла до 37,2°С, часті та болісні сечовипускання, які з'явилися після переохолодження. У сечі: сеча каламутна, питома вага - 1012, білок - 0,033%о, мікроскопія: лейкоцити - 40-45 в п/з, еритроцити - 8-9 в п/з (свіжі), епітелій плаский: 5-8 в п/з, слиз. Який етіологічний фактор у даному випадку найбільш імовірний? A 6-year-old girl came in with complaints of an increase in body temperature to 37.2°C, frequent and painful urination, which appeared after hypothermia. In the urine : cloudy urine, specific gravity - 1012, protein - 0.033%, microscopy: leukocytes - 40-45 in p/z, erythrocytes - 8-9 in p/z (fresh), squamous epithelium: 5-8 in p/z , mucus. What is the most probable etiological factor in this case?

Candida albicans Candida albicans

Proteus mirabilis Proteus mirabilis

Klebsiella pneumoniae Klebsiella pneumoniae

Staphylococcus aureus Staphylococcus aureus

Escherichia coli Escherichia coli

33 / 200
Вагітній 20-ти років 2 дні тому в терміні 18 тижнів вагітності виконано кримінальний аборт на дому. Об'єктивно: температура тіла - 39.4°С,лихоманка, загальний стан тяжкий, пульс - 120/хв., артеріальний тиск - 80/60 мм рт.ст. Дно матки на 4 см вище лона, матка болюча. Сечі за останні 6 годин - 100 мл. Який наступний крок є найбільш доречним? A 20-year-old pregnant woman underwent a criminal abortion at home 2 days ago at 18 weeks of pregnancy. Objectively: body temperature - 39.4°C, fever, general the condition is serious, pulse - 120/min, blood pressure - 80/60 mm Hg. The bottom of the uterus is 4 cm above the womb, the uterus is painful. Urine in the last 6 hours - 100 ml. What is the most appropriate next step?

Антибіотики широкого спектру дії внутрішньовенно, інфузійна терапія, дилатація та кюретаж матки Broad-spectrum antibiotics intravenously, infusion therapy, uterine dilation and curettage

Антибіотики широкого спектру дії внутрішньовенно. гістеректомія з придатками Broad-spectrum antibiotics intravenously. hysterectomy with appendages

Бактеріологічний посів крові, інфузійна терапія, гістеректомія з придатками Bacterial culture of blood, infusion therapy, hysterectomy with appendages

Антибіотики широкого спектру дії внутрішньовенно, дилатація і кюретаж матки Broad-spectrum antibiotics intravenously, dilatation and curettage of the uterus

Бактеріологічний посів крові, форсований діурез, гістеректомія з придатками Bacterial culture of blood, forced diuresis, hysterectomy with appendages

34 / 200
В організм людини з атмосферного повітря надходять декілька хімічних речовин. Як називається тип комбінованої дії, при якому сумісний ефект менший від суми ефектів кожної з речовин, що входить у комбінацію, при їх ізольованій дії на організм? Several chemical substances enter the human body from atmospheric air. What is the type of combined action called, in which the combined effect is less than the sum of the effects of each of the substances included in the combination , with their isolated action on the body?

Потенціювання Potentiation

Комплексна дія Complex Action

Антагонізм Antagonism

Ізольована дія Isolated Action

Поєднана дія Combined Action

35 / 200
Жінка 29-ти років звернулася до лікаря зі скаргами на остуду, біль у горлі, утруднене ковтання та підвищення температури до 38°С. При фізикальному обстеженні лікарем на обох мигдаликах виявлено нальоти у вигляді сірих щільних плівок, збільшення і болючість підщелепних лімфатичних вузлів. Використання якого діагностичного методу буде найбільш доречним наступним кроком? A 29-year-old woman consulted a doctor with complaints of a cold, sore throat, difficulty swallowing, and a temperature rise to 38°C. During a physical examination by a doctor on both plaques in the form of gray dense films, enlargement and tenderness of the submandibular lymph nodes were found in the tonsils. Using which diagnostic method would be the most appropriate next step?

Експрес-тест для виявлення антигенів стрептококу групи А Express test for detection of group A streptococcus antigens

Рентгенографія м'яких тканин шиї X-ray of soft tissues of the neck

Пункція підщелепних лімфовузлів Puncture of submandibular lymph nodes

Бактеріологічне дослідження мазку із порожнини носа та горла Bacterial examination of a swab from the nose and throat cavity

Аналіз на гетерофільні антитіла Heterophilic antibody analysis

36 / 200
Дівчина 22-х років прийшла у жовтні до свого сімейного лікаря на профілактичний огляд. Самопочуття задовільне, скарг не має. Протягом 3-х років хворіє на добре контрольований цукровий діабет І типу. Її 70-річна бабуся хворіє на рак грудей. Останній ПАП-тест було зроблено рік тому, відхилень не зафіксовано. Її зріст - 172 см, вага - 61 кг, ІМТ - 20 кг/м2. Показник глісованого гемоглобіну A1сскладає 6,5%, останній показник глюкометра -6,54 ммоль/л. Яка рекомендація лікаря цій пацієнтці буде найбільш доречною? A 22-year-old girl came to her family doctor for a preventive check-up in October. She is feeling well, has no complaints. She has been suffering from well-controlled diabetes for 3 years type 1 diabetes. Her 70-year-old grandmother has breast cancer. The last Pap test was done a year ago, no abnormalities were recorded. Her height is 172 cm, weight is 61 kg, BMI is 20 kg/m2. The indicator of glycated hemoglobin A1 is 6 .5%, the last reading of the glucometer is -6.54 mmol/l. Which doctor's recommendation would be the most appropriate for this patient?

Пройти мамографію Get a mammogram

Вакцинуватися від грипу Get vaccinated against the flu

Пройти тест на вірус папіломи людини (ВПЛ-тест) Take a human papillomavirus test (HPV test)

Модифікація дієти з метою зниження ваги Diet modification for weight loss

Легкі аеробні вправи щодня Light aerobic exercise every day

37 / 200
Хворий 52-х років звернувся до лікаря зі скаргами на задишку, кашель із мокротинням з домішками крові, підвищену пітливість вночі. В сімейному анамнезі хворих на рак легень немає. За даними картки профілактичних щеплень вакцину БЦЖ отримав вчасно. При фізикальному обстеженні температура - 37,2°С, артеріальний тиск -110/70 мм рт.ст.. пульс - 100/хв., частота дихання - 20/хв.. SpO2- 94% при кімнатному повітрі. При аускультації грудної клітки справа дихальні шуми не прослуховуються. На рентгенограмі ОГК правосторонній гідроторакс. Який наступний крок лікаря буде найбільш доречним? A 52-year-old patient consulted a doctor with complaints of shortness of breath, cough with blood-tinged sputum, increased sweating at night. There is no family history of lung cancer patients. According to the preventive vaccination card, he received the BCG vaccine on time. During the physical examination, the temperature was 37.2°C, the blood pressure was 110/70 mmHg, the pulse was 100/min., the respiratory rate was 20/min.. SpO2- 94% in room air. When auscultating the chest on the right side, no breath sounds are heard. Right-sided hydrothorax on the X-ray of the OGK. What would be the most appropriate next step for the doctor?

Бронхоскопія Bronchoscopy

Тест вивільнення гамма-інтерферону Gamma interferon release test

Проба Манту Mantoux Test

Діагностичний торакоцентез Diagnostic thoracentesis

УЗД легень Lung ultrasound

38 / 200
Жінка 42-х років прийшла до лікаря на профілактичний прийом. Скарг не має. Загальний стан задовільний. Зріст 162 см, маса тіла 87 кг, ІМТ = 33 кг/м2. Артеріальний тиск справа - 140/90 мм рт.ст., зліва - 145/85 мм рт.ст., пульс - 72/хв. Аускультативно тони серця приглушені, шуми не прослуховуються. В легенях дихання везикулярне. Нижній край печінки виступає на 1,5-2 см з-під нижнього краю. Набряків не виявлено. Яку оцінку конституціональній будові тіла цієї пацієнтки має надати лікар (за рекомендаціями ВООЗ)? A 42-year-old woman came to the doctor for a preventive appointment. She has no complaints. The general condition is satisfactory. Height 162 cm, body weight 87 kg, BMI = 33 kg /m2. Arterial pressure on the right - 140/90 mm Hg, on the left - 145/85 mm Hg, pulse - 72/min. On auscultation, heart sounds are muffled, no murmurs can be heard. Breathing is vesicular in the lungs. The lower edge of the liver protrudes 1.5-2 cm from the lower edge. No edema was detected. What assessment of the constitutional structure of this patient's body should the doctor give (according to WHO recommendations)?

Надлишкова вага Overweight

Ожиріння II класу Obesity class II

Ожиріння І класу Obesity of class I

Ожиріння III класу Obesity class III

Нормальна вага Normal weight

39 / 200
Дільничному лікарю доручили підготувати план проведення комплексу лікувальнопрофілактичних заходів серед населення на підпорядкованій території. Які заходи щодо вторинної профілактики захворювань він повинен внести у цей план? The district doctor was instructed to prepare a plan for carrying out a complex of medical and preventive measures among the population in the subordinate territory. What measures for the secondary prevention of diseases should he include in this plan?

Попередження виникнення захворювань Disease prevention

Усунення чинників виникнення хвороб Elimination of factors causing diseases

Попередження ускладнення хвороб Prevention of complications of diseases

Покращання умов життя населення Improving living conditions of the population

Проведення реабілітаційних заходів Carrying out rehabilitation measures

40 / 200
Жінка 52-х років прийшла до свого лікаря на щорічний профілактичний огляд. Вона ніколи не госпіталізувалася до лікарні та виконує легкі аеробні вправи чотири рази на тиждень. У її батька було захворювання серцево-судинної системи. Її зріст - 172 см, вага - 77 кг, ІМТ- 26 кг/м2. При фізикальному обстеженні температура - 36,8°С, пульс - 81/хв., частота дихання - 16/хв., артеріальний тиск сидячи - 160/100 мм рт.ст. на обох руках. Офтальмоскопія очного дна виявила звуження судин сітківки. Лабораторні показники без відхилень. Що з перерахованого буде найбільш доречним призначенням з метою зниження ризику смерті цієї пацієнтки від серцево-судинних захворювань? A 52-year-old woman comes to her doctor for an annual check-up. She has never been hospitalized and does light aerobic exercise four times a week. Her father had a disease of the cardiovascular system. Her height - 172 cm, weight - 77 kg, BMI - 26 kg/m2. During physical examination, temperature - 36.8°C, pulse - 81/min., respiratory rate - 16/min. , sitting blood pressure of 160/100 mmHg on both arms.Fundus ophthalmoscopy revealed narrowing of the retinal vessels.Laboratory findings were normal.Which of the following would be the most appropriate appointment to reduce this patient's risk of death from cardiovascular disease?

Аспірин Aspirin

Еналанрил Enalanril

Варфарин Warfarin

Дипіридамол Dipyridamole

Нітрогліцерин Nitroglycerin

41 / 200
Хворий скаржиться на почервоніння, печіння, відчуття стороннього тіла у правому оці. Захворів раптово. Під час огляду: гострота зору правого та лівого ока - 1,0. У правому оці - гіперемія та набряк кон'юнктиви, поверхнева ін'єкція. У кон'юнктивальному мішку гнійне виділення. Рогівка прозора.Колір та малюнок райдужки не змінені, зіниця рухлива. Який найбільш імовірний діагноз? The patient complains of redness, burning, feeling of a foreign body in the right eye. He fell ill suddenly. During the examination: the visual acuity of the right and left eye is 1.0. in the right eye - hyperemia and swelling of the conjunctiva, superficial injection. Purulent discharge in the conjunctival sac. The cornea is transparent. The color and pattern of the iris have not changed, the pupil is mobile. What is the most likely diagnosis?

Гострий напад глаукоми Acute attack of glaucoma

Гострий іридоцикліт Acute iridocyclitis

Гострий дакріоцистит Acute dacryocystitis

Стороннє тіло рогівки Corneal foreign body

Гострий кон'юнктивіт Acute conjunctivitis

42 / 200
45-річний чоловік звернувся до лікаря зі скаргами на біль у горлі та лихоманку протягом 10-ти днів. В анамнезі: хворів на виразковий коліт та хронічний біль у спині. Викурює 1 пачку сигарет на день протягом 10-ти років. Батько пацієнта помер у 50 років від раку товстої кишки. При фізикальному обстеженні температура тіла -38,6°С, артеріальний тиск - 130/80 мм рт.ст.. пульс - 72/хв., частота дихання - 18/хв., при огляді ротової порожнини глотка еритематозна, у криптах мигдаликів ексудат. Антигенний тест на СГА (стрептокок групи А) позитивний. Яка рекомендація лікаря зі вторинної профілактики онкологічних захворювань найбільш імовірно буде доречною цьому пацієнту? A 45-year-old man went to the doctor complaining of a sore throat and fever for 10 days. He had a history of ulcerative colitis and chronic back pain . Smokes 1 pack of cigarettes a day for 10 years. The patient's father died at the age of 50 from colon cancer. During physical examination, the body temperature is -38.6°C, blood pressure - 130/80 mm Hg. pulse - 72/min., respiratory rate - 18/min., when examining the oral cavity, the pharynx is erythematous, there is exudate in the crypts of the tonsils. The antigen test for SGA (group A streptococcus) is positive. What recommendation of a doctor for the secondary prevention of oncological diseases is most likely to be appropriate for this patient ?

Провести колоноскопію Perform colonoscopy

Провести гнучку сигмоскопію Perform flexible sigmoscopy

Зробити ПСА-тест та пройти пальцеве ректальне дослідження Do a PSA test and undergo a digital rectal examination

Провести низькодозову КТ легень Perform a low-dose CT scan of the lungs

Зробити аналіз калу на приховану кров Do stool analysis for occult blood

43 / 200
Хлопчикові 14 років. Протягом 3-х років спостерігається підвищення артеріального тиску до 170/100 мм рт.ст. Лікування анти-гіпертензивними препаратами неефективне. ХолтерАТ: стабільна артеріальна гіпертензія. Доплер судин нирок: часткова візуалізація ниркових артерій зліва. Яким буде попередній діагноз? The boy is 14 years old. For 3 years, an increase in blood pressure has been observed up to 170/100 mm Hg. Treatment with anti-hypertensive drugs is ineffective. Holter BP: stable arterial hypertension. Doppler of renal vessels: partial visualization of the renal arteries on the left. What will be the preliminary diagnosis?

Реноваскулярна гіпертензія Renovascular hypertension

Феохромоцитома Pheochromocytoma

Гіпоталамічний синдром Hypothalamic syndrome

Первинна артеріальна гіпертензія Primary arterial hypertension

Адреногенітальний синдром Adrenogenital syndrome

44 / 200
Серед пацієнтів загальної лікарської практики за останні роки відбувається неухильне збільшення кількості осіб похилого та старечого віку. Переважання якого типу патології необхідно чекати в структурі захворюваності населення, яке обслуговується? Among the patients of general medical practice, there has been a steady increase in the number of elderly and senile people in recent years. What type of pathology should be expected to predominate in the morbidity structure of the population being served?

Хронічної патології Chronic pathology

Виробничої патології Production pathology

Гострої патології Acute pathologies

Неепідемічної патології Non-epidemic pathology

Інфекційної патології Infectious pathology

45 / 200
27-річний хлопець вперше за останні декілька років звернувся до свого лікаря занепокоєний результатами ліпідограми. Він повідомив, що не хоче захворіти на атеросклероз та почав робити такий аналіз щорічно з 22-х років. Цього року у нього вперше зафіксовано показник (5,0 ммоль/л), що є вищим у порівнянні з попередніми чотирма роками (не більше 4,1 ммоль/л). Артеріальної гіпертензії, або ранньої ішемічної хвороби серця у сімейному анамнезі немає. Його бабуся хворіє на цукровий діабет II типу з 58-ми років. Його 64-річний дідусь переніс радикальну простатектомію з приводу раку простати. Окрім інформування щодо основних факторів ризику церебро-васкулярної патології та рекомендацій по збалансованому харчуванню, який наступний крок у веденні пацієнта буде найбільш доречним? A 27-year-old guy went to his doctor for the first time in the last few years, worried about the results of a lipidogram. He said that he did not want to get atherosclerosis and began to do such an analysis annually from 22 years old. This year, for the first time, he recorded a reading (5.0 mmol/l), which is higher compared to the previous four years (no more than 4.1 mmol/l). Arterial hypertension, or early coronary heart disease in no family history.His grandmother has type 2 diabetes since age 58.His 64-year-old grandfather underwent a radical prostatectomy for prostate cancer.In addition to being informed about the main risk factors for cerebrovascular disease and recommending a balanced diet, what is the next step will be the most appropriate in the management of the patient?

Повторити визначення рівня загального холестерину через 5 років Repeat determination of total cholesterol level after 5 years

Визначити рівень простатспецифічного антигену (ПСА-тест) Determine the level of prostate-specific antigen (PSA test)

Пройти глюкозотолерантний тест Pass glucose tolerance test

Розпочати терапію статинами Start statin therapy

46 / 200
Чоловік 58-ми років, скаржиться на постійний біль в епігастрії, що іррадіює в спину і посилюється після прийому жирної їжі. Пацієнт відмічає слабкість, за останні 2 місяці втратив у вазі до 9 кг. Місяць тому з'явилась жовтяниця, що поступово зростає. Живіт м'який, печінка не збільшена. При лабораторному дослідженні крові гемоглобін - 68 г/л, лейкоцити - 9,0 • 109/л, швидкість зсідання еритроцитів - 36 мм/год. На УЗД: жовчний міхур великих розмірів, конкрементів не містить, холедох розширений, в головці підшлункової залози - округле утворення до 4 см в діаметрі з нечіткими контурами. Який діагноз є найбільш імовірним? A 58-year-old man complains of constant pain in the epigastrium, which radiates to the back and worsens after eating fatty food. The patient notes weakness, over the past 2 months he has lost weighing up to 9 kg. Jaundice appeared a month ago, which is gradually increasing. The abdomen is soft, the liver is not enlarged. During a laboratory blood test, hemoglobin is 68 g/l, leukocytes are 9.0 • 109/l, the erythrocyte sedimentation rate - 36 mm/h. On ultrasound: the gallbladder is large, does not contain calculi, the choledochus is dilated, in the head of the pancreas - a rounded formation up to 4 cm in diameter with unclear contours. What diagnosis is most likely?

Рак шлунка з метастазами в печінку Gastric cancer with liver metastases

Рак головки підшлункової залози Cancer of the head of the pancreas

Хронічний холецистопанкреатит Chronic cholecystopancreatitis

Виразкова хвороба дванадцятипалої кишки з пенетрацією в головку підшлункової залози Duodenal ulcer with penetration into the head of the pancreas

Хронічний індуративний панкреатит Chronic indurative pancreatitis

47 / 200
Хвора 55-ти років скаржиться на випорожнення 3-4 рази протягом 1-2 годин переважно вранці, після сніданку. Посилення діареї пов'язує з підвищеними вимогами з боку оточення, хвилюванням, очікуванням небезпеки. Об'єктивно: метушлива, багатомовна, збільшеної ваги. Шкіра без змін, тургор нормальний, живіт м'який безболісний. В аналізі крові змін не має; аналіз калу: несформований, незначна кількість крохмальних зерен та м'язових волокон, лейкоцити - 3-4 в п/з; колоноскопія без патології. Призначення якого препарату буде найбільш доречним? A 55-year-old patient complains of defecation 3-4 times during 1-2 hours, mainly in the morning, after breakfast. The increase in diarrhea is associated with increased demands from surroundings, excitement, anticipation of danger. Objectively: fussy, talkative, increased weight. Skin unchanged, turgor normal, abdomen soft and painless. Blood test shows no changes; stool analysis: unformed, insignificant amount of starch grains and m ulcer fibers, leukocytes - 3-4 in p/z; colonoscopy without pathology. Which drug would be most appropriate to prescribe?

Ампіцилін Ampicillin

Лоперамід Loperamide

Анаприлін Anaprilin

Дротаверин Drotaverin

Бісакодил Bisacodyl

48 / 200
Чоловік 25-ти років, звернувся до лікаря зі скаргами на появу висипань в міжпальцевих проміжках, в ділянці променевозап'ястних суглобів, паховій ділянці та живота. які супроводжуються сильним свербежем, особливо в нічний час. Пацієнт зазначає, що після вечірнього душу покращення не наступає. Також йому здається, що як тільки висип проходить в одному місці, він одразу з'являється в іншому. При фізикальному обстеженні на фоні неураженої шкіри множинні еритематозні папули та екскоріації. Який діагноз є найбільш імовірним? A 25-year-old man turned to the doctor with complaints of rashes in the interdigital spaces, in the area of the radiocarpal joints, inguinal area, and abdomen, which are accompanied by strong itching, especially at night. The patient notes that there is no improvement after an evening shower. It also seems to him that as soon as the rash goes away in one place, it immediately appears in another. On physical examination, on the background of intact skin, multiple erythematous papules and excoriations. What is the most likely diagnosis?

Короста Scabies

Екзема Eczema

Епідермофітія Epidermophyta

Обсесивно-компульсивний розлад Obsessive-compulsive disorder

49 / 200
Жінка 53-х років, звернулася до лікаря зі скаргами на розпираючий біль в м'язах лівої гомілки та набряк, що з'явилися тиждень тому. Раніше подібного стану не виникало. Температура тіла - 37,6°С.При огляді гомілка в середній третині збільшена в об'ємі на 5 см. Шкіра гомілки та ступні синюшна, блискуча, відмічається місцева гіпертермія. При пальпації гомілки - різка болючість. Позитивні симптоми Хоманса та Мозеса. Пульс на артеріях стоп задовільний. Який діагноз є найбільш імовірним? A 53-year-old woman turned to the doctor with complaints of excruciating pain in the muscles of the left leg and swelling that appeared a week ago. Previously, a similar condition did not occur. Body temperature - 37.6°C. During examination, the lower leg in the middle third is enlarged in volume by 5 cm. The skin of the lower leg and feet is bluish, shiny, local hyperthermia is noted. When palpating the lower leg, there is sharp pain. Positive symptoms of Homans and Moses. The pulse on the arteries of the feet is satisfactory. What is the most likely diagnosis?

Післятромбофлебітичний синдром Postthrombophlebitic syndrome

Гострий тромбоз підколінної вени Acute thrombosis of popliteal vein

Хвороба Рейно Raynaud's disease

Гострий тромбоз підколінної артерії Acute thrombosis of popliteal artery

50 / 200
Через 1,5 години після народження на 32 тижні у дитини спостерігаються роздування крил носа, хрюкаючий видих, тахіпное та втяжіння міжреберних проміжків. Забруднення амніотичної рідини меконієм виявлено не було. Температура новонародженого –37,4°С,пульс - 180/хв., частота дихання - 80/хв. Шкіра ціанотична. Аускультативно в легенях послаблене дихання з обох боків. РаО - 32 мм рт.ст., РаСОа - 48 мм рт.ст. На рентгенограмі органів грудної клітки дифузний сітчасто-зернистий рисунок, повітряна 'бронхограма'. Який діагноз є найбільш імовірним? In 1.5 hours after birth at 32 weeks, the baby has flaring of the wings of the nose, grunting exhalation, tachypnea, and retractions of the intercostal spaces. Meconium contamination of the amniotic fluid was not detected The newborn's temperature is 37.4°С, pulse - 180/min, respiratory rate - 80/min. The skin is cyanotic. Auscultation in the lungs has weakened breathing on both sides. PaO - 32 mm Hg, PaSOa - 48 mm Hg .art. On the X-ray of the chest organs, there is a diffuse mesh-granular pattern, air 'bronchogram'. What is the most likely diagnosis?

Ідіопатичний легеневий фіброз Idiopathic pulmonary fibrosis

Транзиторне тахіпное новонароджених Transient tachypnea of newborns

Аспіраційна пневмонія Aspiration pneumonia

Респіраторний дистрес-синдром Respiratory distress syndrome

Крововилив у легені Hemorrhage in the lungs

51 / 200
Чоловік 46-ти років звернувся до лікаря зі скаргами на появу висипань в ділянці кистей рук, що супроводжуються сильним свербежем і турбують протягом тижня. У анамнезі бронхіальна астма, сезонні алергії. Пацієнт відзначає, що має дуже суху шкіру. При фізикальному обстеженні на тильній стороні правої та лівої кисті виявлено множинні еритематозні папули та везикули з серозним вмістом, ділянки ліхеніфікації та екскоріації. Який діагноз є найбільш імовірним? A 46-year-old man went to the doctor with complaints about the appearance of rashes in the area of the hands, accompanied by severe itching and bothering him for a week. He has a history of bronchial asthma, seasonal allergies. The patient notes that he has very dry skin. Physical examination revealed multiple erythematous papules and vesicles with serous contents, areas of lichenification and excoriation on the back of the right and left hand. What diagnosis is most likely?

Псоріаз Psoriasis

Екзема Eczema

Короста Scabies

Себорейний дерматит Seborrheic dermatitis

Контактний дерматит Contact dermatitis

52 / 200
Чоловіка 42-х років, шахтаря, через 12 годин звільнено з-під завалу. Об'єктивно: обидві гомілки і ступні блідого кольору. Пульсації периферійних судин немає. Чутливість та пасивні рухи в суглобах відсутні. Доставлений із джгутами на обох кінцівках. Які невідкладні заходи першої лікарської допомоги необхідно провести для запобігання міоглобінурії та гострої ниркової недостатності? A 42-year-old man, a miner, was freed from the rubble after 12 hours. Objectively: both lower legs and feet are pale in color. There is no pulsation of peripheral vessels. Tenderness and passive motion in the joints are absent. Delivered with tourniquets on both limbs. What emergency first aid measures should be taken to prevent myoglobinuria and acute renal failure?

Джгут не знімати, гіпербарична оксигенація Do not remove tourniquet, hyperbaric oxygenation

Джгут не знімати, дезінтоксикаційна терапія Do not remove tourniquet, detoxification therapy

Зняти джгут, дезінтоксикаційна терапія Remove tourniquet, detoxification therapy

Джгут не знімати, ампутація кінцівок вище джгута Do not remove the tourniquet, amputation of limbs above the tourniquet

Зняти джгут, гіпербарична оксигснація Remove tourniquet, hyperbaric oxygenation

53 / 200
Під час медичного огляду робітників металургійного заводу після вимірювання індексу маси тіла (ІМТ) у 20% осіб було виявлено надлишкову вагу. Вміст яких продуктів необхідно зменшити в раціоні в першу чергу для нормалізації маси тіла у цієї групи осіб? During the medical examination of workers at a metallurgical plant, after measuring the body mass index (BMI), 20% of them were found to be overweight. Which foods should be reduced in the diet first queue for the normalization of body weight in this group of people?

Фруктів Fruits

М'ясних та рибних продуктів Meat and fish products

Молока та молочних продуктів Milk and milk products

Овочів Vegetables

Хлібобулочних виробів Bakery

54 / 200
При розслідуванні випадку харчового отруєння встановлено діагноз харчової токсикоінфекції, що спричинена парагемолітичним вібріоном. Який із перелічених продуктів і страв може бути найімовірнішою причиною даного отруєння? When investigating a case of food poisoning, a diagnosis of food poisoning caused by parahemolytic vibrio was established. Which of the listed products and dishes could be the most likely cause of this poisoning?

Ковбаса сирокопчена Raw-smoked sausage

Виноград Grapes

Устриці Oysters

Молоко пастеризоване Milk pasteurized

Гриби консервовані Canned mushrooms

55 / 200
Чоловік 58-ми років звернувся до лікаря зі скаргами на значне підсилення загальної слабкості за останні кілька тижнів, ціаноз, болі у животі, нудоту, періодичне блювання. запах ацетону з роту, гіперпігментацію шкірних складок. З анамнезу відомо, що пацієнт хворіє на туберкульоз. При фізикальному обстеженні артеріальний тиск - 80/40 мм рт.ст., пульс - 124/хв. При лабораторному дослідженні крові: натрій - 125 ммоль/л, хлориди - 74 ммоль/л, калій - 5,7 ммоль/л. глюкоза - 3,5 ммоль/л, сечовина - 14 ммоль/л. У загальному аналізі сечі: білок - 0,66 г/л, лейкоцити - 10-12 в п/з, еритроцити - 5-6 в п/з. циліндри - поодинокі. Який наступний крок у веденні пацієнта буде найбільш доречним? A 58-year-old man consulted a doctor with complaints of a significant increase in general weakness over the past few weeks, cyanosis, abdominal pain, nausea, periodic vomiting. the smell of acetone from the mouth, hyperpigmentation of skin folds. From the anamnesis, it is known that the patient has tuberculosis. During the physical examination, blood pressure - 80/40 mm Hg, pulse - 124/min. During the laboratory blood test: sodium - 125 mmol/l, chlorides - 74 mmol/l, potassium - 5.7 mmol/l, glucose - 3.5 mmol/l, urea - 14 mmol/l. In the general analysis of urine: protein - 0.66 g/l, leukocytes - 10 12 in p/z, erythrocytes - 5-6 in p/z. cylinders - single. What next step in the management of the patient would be the most appropriate?

Визначення концентрації кетонових тіл в сечі Determining the concentration of ketone bodies in urine

Визначення концентрації діастази в сечі Determination of diastase concentration in urine

Визначення концентрації кортизолу в крові Determination of cortisol concentration in blood

Визначення концентрації лужної фосфатази в крові Determining the concentration of alkaline phosphatase in the blood

Визначення концентрації креатиніну в крові Determination of creatinine concentration in blood

56 / 200
Жінка 60-ти років з нормальною масою тіла отримує з добовим раціоном 50 г білка, 70 г жиру, 300 г вуглеводів. У раціоні переважають зернобобові, достатньо овочів, але обмежена кількість молока і молочних продуктів. Весною щоденно вживає щавель і ревінь. Ризиком якого захворювання для жінки у першу чергу може бути такий раціон? A 60-year-old woman with a normal body weight receives 50 g of protein, 70 g of fat, 300 g of carbohydrates in her daily diet. The diet is dominated by legumes, there are enough vegetables, but a limited amount of milk and dairy products. In the spring, she eats sorrel and rhubarb every day. What risk of disease for a woman can such a diet pose in the first place?

Остеопорозу Osteoporosis

Атеросклерозу Atherosclerosis

Ожиріння Obesity

Сечокам'яної хвороби Urolithiasis

Гіпертонічної хвороби Hypertensive disease

57 / 200
Хворого після перенесеного Q-інфаркту міокарда турбує задишка при незначних навантаженнях, нічні напади сухого кашлю, відчуття хрипів у грудній клітці і ортопное. В ході огляду:акроціаноз, частота серцевих скорочень - 96/хв., тахікардія, ослаблення I-го тону над верхівкою, вислуховується ІІІ-й тон. В легенях в базальних відділах незвучні хрипи. Під час ехокардіоскопії дилатація лівого шлуночка, витончення і дискінезія міжшлуночкової перегородки, фракція викиду - 39%. Який варіант дисфункції міокарда є найбільш імовірним? After a Q-myocardial infarction, the patient is troubled by shortness of breath with minor exertion, night attacks of dry cough, a feeling of wheezing in the chest and orthopnea. During the examination: acrocyanosis, frequency heart rate - 96/min., tachycardia, weakening of the 1st sound above the apex, the 3rd sound is heard. In the lungs in the basal parts, there are silent rales. During echocardioscopy, dilatation of the left ventricle, thinning and dyskinesia of the interventricular septum, ejection fraction - 39% . What variant of myocardial dysfunction is the most probable?

Змішана дисфункція обох шлуночків Mixed dysfunction of both ventricles

Діастолічна дисфункція правого шлуночка Diastolic dysfunction of the right ventricle

Систолічна дисфункція правого шлуночка Right ventricular systolic dysfunction

Систолічна дисфункція лівого шлуночка Left ventricular systolic dysfunction

Діастолічна дисфункція лівого шлуночка Diastolic dysfunction of the left ventricle

58 / 200
Вагітна в терміні 34 тижні доставлена в пологовий будинок в зв'язку з кровотечею зі статевих шляхів, що з'явилася під час сну. Біль не турбує. Кровотеча близько 300,0 мл. Який діагноз є найбільш імовірним? A 34-week pregnant woman was taken to the maternity ward due to bleeding from the genital tract that appeared during sleep. The pain does not bother her. The bleeding is close 300.0 ml. Which diagnosis is the most probable?

Розрив судин пуповини Rupture of umbilical cord vessels

Передчасне відшарування нормально розташованої плаценти Premature detachment of a normally located placenta

Кровоточива ерозія Bleeding erosion

Передлежання плаценти Placenta previa

Рак шийки матки Cervical cancer

59 / 200
Хвора 60-ти років скаржиться на біль в міжфалангових суглобах кистей, який посилюється при роботі. Об'єктивно: дистальні та проксимальні суглоби II-IV пальців деформовані, з вузлами Гебердена, Бушара, болючі, з обмеженою рухомістю. Рентгенограма суглобів: суглобові щілини звужені, краєві остеофіти. субхондральний склероз. Який діагноз найбільш ймовірний? A 60-year-old patient complains of pain in the interphalangeal joints of the hands, which intensifies during work. Objectively: the distal and proximal joints of II-IV fingers are deformed, with Heberden's, Bouchard's nodes, painful, with limited mobility. X-ray of the joints: joint spaces are narrowed, marginal osteophytes. subchondral sclerosis. What is the most likely diagnosis?

Хвороба Рейтера Reiter's disease

Ревматичний артрит Rheumatic arthritis

Хвороба Бехтерєва Bekhterev's disease

Деформуючий остеоартроз Deforming osteoarthritis

Псоріатичний артрит Psoriatic arthritis

60 / 200
Хворий 30-ти років звернувся до сімейного лікаря через 2 місяці після операції з приводу відкритого перелому плечової кістки. При огляді стан хворого задовільний, в ділянці операційної рани є нориця з незначним гнійним виділенням, почервоніння, флюктуація. На рентгенограмі: деструкція плечової кістки з секвестрами. Яке ускладнення виникло у хворого в післяопераційному періоді? A 30-year-old patient consulted a family doctor 2 months after surgery for an open fracture of the humerus. On examination, the patient's condition is satisfactory, there is a fistula in the area of the surgical wound with slight purulent discharge, redness, fluctuation. X-ray: destruction of the humerus with sequestrations. What complications did the patient experience in the postoperative period?

Нагноєння рани Wound suppuration

Післятравматична флегмона Post-traumatic phlegmon

Гематогенний остеомієліт Hematogenous osteomyelitis

Лігатурна нориця Ligatary fistula

Посттравматичний остеомієліт Post-traumatic osteomyelitis

61 / 200
В ході обстеження дитини 5-ти років зі скаргами на постійний кашель з виділенням гнійного мокротиння та вологі хрипи справа у задньонижніх відділах виявлено: гнійний ендобронхіт II-III ступеня, переважно справа, хлориди поту - 36 ммоль/л (норма - 0-35 ммоль/л). на KT - циліндричні бронхоектази в S9 та S10. Запропонуйте оптимальний метод лікування даної дитини: During the examination of a 5-year-old child with complaints of a constant cough with purulent sputum discharge and wet wheezing on the right in the lower back, it was found: purulent endobronchitis II-III degree, mainly on the right, sweat chlorides - 36 mmol/l (normal - 0-35 mmol/l). on KT - cylindrical bronchiectasis in S9 and S10. Suggest the optimal method of treatment for this child:

Генно-інженерна терапія Genetic engineering therapy

Лаваж бронхіального дерева Bronchial tree lavage

Видалення уражених сегментів Delete affected segments

Кінезотерапія Kinesiotherapy

Тривала антибіотикотерапія Long-term antibiotic therapy

62 / 200
Дівчинка 13-ти років скаржиться на головний біль, запаморочення, відчуття пульсації в голові, шум у вухах, носові кровотечі, серцебиття, біль в ділянці серця, задишку. При огляді блідість шкірних покривів. При пальпації виявлено ослаблення пульсу на стегнових артеріях. Артеріальний тиск на верхніх кінцівках - 125/60 мм рт.ст., на нижніх - 110/60 мм рт.ст. Аускультативно гучний систолічний шум, що іррадіює у сонні артерії. Який з перерахованих діагнозів є найбільш імовірним? A 13-year-old girl complains of headache, dizziness, pulsation in the head, tinnitus, nosebleeds, palpitations, pain in the heart area, shortness of breath. On examination, pallor of the skin. During palpation, a weakening of the pulse on the femoral arteries was revealed. Blood pressure on the upper extremities - 125/60 mm Hg, on the lower - 110/60 mm Hg. Auscultation loud systolic noise radiating during sleep arteries. Which of the listed diagnoses is the most probable?

Коарктація аорти Coarctation of the aorta

Стеноз устя легеневої артерії Stenosis of the mouth of the pulmonary artery

Відкрита артеріальна протока Open ductus arteriosus

Стеноз устя аорти Stenosis of the mouth of the aorta

Дефект міжшлуночкової перетинки Defect of interventricular membrane

63 / 200
Жінка 37-ми років звернулася до лікаря зі скаргами на болі у ділянці попереку, підвищення температури тіла до 38°6'. При фізикальному обстеженні позитивний симптом Пастернацького. більше виражений справа. При лабораторному дослідженні у загальному аналізі крові: гемоглобін - 115 г/л, еритроцити - 3,9 • 1012/л, лейкоцити - 15,2 • 109/л, ШОЕ- 28 мм/год. У загальному аналізі сечі: колір - світло жовтий, питома вага -1018, білок - 0,42 г/л, лейкоцити - 20-30 у полі зору, багато слизу. Який діагноз є найбільш імовірним? A 37-year-old woman consulted a doctor with complaints of pain in the lower back, an increase in body temperature to 38°6'. During the physical examination, Pasternacki's symptom was positive. more expressed on the right. During laboratory research in the general blood analysis: hemoglobin - 115 g/l, erythrocytes - 3.9 • 1012/l, leukocytes - 15.2 • 109/l, ESR - 28 mm/h. In the general analysis of urine: color - light yellow, specific gravity -1018, protein - 0.42 g/l, leukocytes - 20-30 in the field of vision, a lot of mucus. What diagnosis is most likely?

Гострий гломерулонефрит Acute glomerulonephritis

Тубулоінтерстиційний нефрит Tubulointerstitial nephritis

Гострий апендицит Acute appendicitis

Гострий аднексит Acute adnexitis

Гострий пієлонефрит Acute pyelonephritis

64 / 200
Жінка 36-ти років скаржиться на біль, обмеження рухів у дрібних суглобах рук, ускладнення при ковтанні твердої їжі. слабкість, сухий кашель. Об'єктивно: шкіра кистей та передпліччя щільна, гладка. Проксимальні суглоби II-IV пальців кистей рук набряклі, болючі при пальпації. Над легенями сухі розсіяні хрипи, межі серця зміщені вліво на 2 см, тони приглушені. У крові: швидкість осідання еритроцитів - 36 мм/год, у-глобулінів - 24%. У сечі: змін немає. Який найбільш вірогідний діагноз? A 36-year-old woman complains of pain, limitation of movement in the small joints of her hands, difficulty swallowing solid food. weakness, dry cough. Objectively: the skin of the hands and the forearm is dense, smooth. The proximal joints of the II-IV fingers of the hands are swollen, painful on palpation. Over the lungs, dry scattered rales, the borders of the heart are shifted to the left by 2 cm, the sounds are muffled. In the blood: the sedimentation rate of erythrocytes is 36 mm/h, in -globulins - 24%. In the urine: there are no changes. What is the most likely diagnosis?

Системний червоний вовчак Systemic lupus erythematosus

Ревматоїдний артрит Rheumatoid arthritis

Дерматоміозит Dermatomyositis

Системна склеродермія Systemic scleroderma

Саркоїдоз Sarcoid

65 / 200
Жінка 60-ти років надійшла до відділення невідкладної допомоги зі скаргами на сильний головний біль у потиличній ділянці, нудоту, блювання, запаморочення. При огляді хвора трохи загальмована, обличчя гіперемоване, артеріальний тиск - 220/130 мм рт.ст., пульс - 78/хв. При неврологічному обстеженні парезів не виявлено, рефлекси рівномірно жваві, м'язовий тонус не змінений, чутливість збережена. Менінгеальні симптоми відсутні. Який стан найбільш імовірно розвинувся у хворої? A 60-year-old woman came to the emergency department with complaints of a severe headache in the back of the head, nausea, vomiting, dizziness. On examination, the patient is slightly depressed, her face hyperemic, blood pressure - 220/130 mm Hg, pulse - 78/min. During the neurological examination, no paresis was detected, reflexes were uniformly lively, muscle tone did not change, sensitivity was preserved. There were no meningeal symptoms. What condition most likely developed in the patient?

Гіпертонічний криз Hypertensive crisis

Ішемічний інсульт Ischemic stroke

Субарахноїдальний крововилив Subarachnoid hemorrhage

Пухлина головного мозку Brain tumor

Транзиторне порушення мозкового кровообігу Transient disturbance of cerebral circulation

66 / 200
Хлопчик 3-х років надійшов з вираженим набряковим синдромом. Об'єктивно: блідий. Артеріальний тиск - 90/60 мм рт.ст. Олігурія. Загальний аналіз сечі: колір - жовтий, відносна густина -1020, білок - 3,5 г/л, еритроцити - вилужені, 4-5 в п/з, лейкоцити - 2-3 в п/з. Добова протеїнурія - 6,6 г. Холестерин - 9,8 ммоль/л. Який наступний крок у веденні пацієнта буде найбільш доречним? A 3-year-old boy was admitted with severe edematous syndrome. Objectively: pale. Blood pressure - 90/60 mm Hg. Oliguria. General analysis of urine : color - yellow, relative density -1020, protein - 3.5 g/l, erythrocytes - leached, 4-5 in p/z, leukocytes - 2-3 in p/z. Daily proteinuria - 6.6 g. Cholesterol - 9.8 mmol/l. What next step in patient management would be most appropriate?

Комп'ютерна томографія без контрасту Computed tomography without contrast

Аналіз сечі за Нечипоренко Urine analysis according to Nechiporenko

Проба Зимницького Zymnytsky's trial

Спостереження протягом тижня Observations during the week

Біопсія нирки Kidney biopsy

67 / 200
Хворий 38-ми років скаржиться на гнійне виділення із лівої половини носа, утруднене носове дихання, головний біль, важкість в області лівої щоки, підвищення температури тіла до 37,5°С.Хворіє впродовж 6-ти днів. Захворювання пов'язує з переохолодженням. Об'єктивно: болісність при пальпації в області лівої щоки. Слизова оболонка лівої половини носової порожнини гіперемована, набухла, в середньому носовому ході - гнійна смужка. Який найбільш імовірний діагноз? A 38-year-old patient complains of purulent discharge from the left half of the nose, difficulty breathing through the nose, headache, heaviness in the left cheek area, an increase in body temperature to 37, 5°C. He has been sick for 6 days. The disease is associated with hypothermia. Objectively: pain upon palpation in the left cheek area. The mucous membrane of the left half of the nasal cavity is hyperemic, swollen, and there is a purulent strip in the middle nasal passage. Which is the most probable diagnosis?

Лівобічний гострий гайморит Left-sided acute sinusitis

Лівобічний гострий фронтит Left-sided acute frontitis

Викривлення носової перегородки вліво Curvation of the nasal septum to the left

Лівобічний гострий етмоїдит Left-sided acute ethmoiditis

Гострий риніт (нежить) Acute rhinitis (runny nose)

68 / 200
Хвора 23-х років доставлена ургентно зі скаргами на білі, внизу живота, більш інтенсивний справа, з іррадіацією в пряму кишку, запаморочення. Вище зазначені скарги з'явилися раптово вночі. Остання менструація 2 тижні тому. Об'єктивно: шкірні покриви бліді, пульс - 92/хв.,температуратіла36,6°С,артеріальнийтиск - 100/60 мм рт.ст. Живіт дещо напружений, незначно болючий в нижніх відділах, симптоми подразнення очеревини слабо позитивні. Гемоглобін - 98 г/л. Який попередній діагноз? A 23-year-old patient was brought urgently with complaints of white, lower abdomen, more intense on the right, with irradiation in the rectum, dizziness. The above complaints appeared suddenly at night. The last menstruation was 2 weeks ago. Objectively: the skin is pale, the pulse is 92/min, the body temperature is 36.6°C, the blood pressure is 100/60 mm Hg. The abdomen is somewhat tense, slightly painful in the lower parts, symptoms of peritoneal irritation are weakly positive. Hemoglobin - 98 g/l. What is the previous diagnosis?

Кишкова непрохідність Intestinal obstruction

Ниркова колька Renal colic

Порушена позаматкова вагітність Disrupted ectopic pregnancy

Апоплексія яєчника Ovarian apoplexy

Гострий апендицит Acute appendicitis

69 / 200
У ході проведення медичного огляду студентів, лікар звернув увагу на хлопця 24-х років з ростом 176 см та масою тіла 68 кг, у якого спостерігалась сухість шкіри та гіперкератоз. При офтальмологічному обстеженні виявлене суттєве зниження гостроти сутінкового зору. У харчуванні студента практично ніколи не були включені в раціон молочні продукти, овочі та фрукти. Яке із зазначених захворювань можна припустити? During the medical examination of students, the doctor noticed a 24-year-old boy with a height of 176 cm and a body weight of 68 kg, who had dry skin and hyperkeratosis . During an ophthalmological examination, a significant decrease in the acuity of twilight vision was found. In the student's diet, dairy products, vegetables and fruits were practically never included in the diet. Which of the indicated diseases can be assumed?

С-гіповітаміноз C-hypovitaminosis

А-гіповітаміноз A hypovitaminosis

Астигматизм Astigmatism

Діенцефальний синдром Diencephalic syndrome

Міопія Myopia

70 / 200
Чоловік 32-х років звернувся до лікаря зі скаргами на висип, що з'явився на колінах 4 місяці тому і зараз вперше починає з'являтися на ліктях. Зі слів пацієнта болю не відчуває, але місце висипу злегка свербить та кровоточить при розчісуванні. Він відзначив, що на ліктях висип з'явився після подряпин, які йому наніс його домашній кіт. При фізикальному обстеженні лікарем виявлено на колінах та ліктях еритематозні бляшки діаметром від 2 до 3 см із чітким контуром, щільні, вкриті сріблястими лусочками. Який діагноз є найбільш імовірним? A 32-year-old man went to the doctor complaining of a rash that appeared on his knees 4 months ago and is now beginning to appear on his elbows for the first time. With according to the patient, he does not feel pain, but the rash is slightly itchy and bleeds when combed. He noted that the rash appeared on his elbows after being scratched by his pet cat. Physical examination by the doctor revealed erythematous plaques on the knees and elbows with a diameter of 2 up to 3 cm with a clear contour, dense, covered with silvery scales. What diagnosis is most likely?

Хвороба котячих подряпин Cat Scratch Disease

Псоріаз Psoriasis

Екзема Eczema

Контактний дерматит Contact dermatitis

Себорейний дерматит Seborrheic dermatitis

71 / 200
У хворого 56-ти років, що знаходиться в палаті інтенсивної терапії з приводу гострого інфаркту міокарда у ділянці передньої стінки лівого шлуночка, посилилась загальна слабкість та з'явилось відчуття 'завмирання' у роботі серця. При аналізі ЕКГ виявлено: ритм синусовий, частота серцевих скорочень - 76/хв. Періодично з'являються поширені шлуночкові комплекси тривалістю 0,22 с., неправильної форми, після яких виявляються повні компенсаторні паузи. Яке ускладнення виникло у даного хворого? A 56-year-old patient, who is in the intensive care unit due to an acute myocardial infarction in the area of the anterior wall of the left ventricle, has increased general weakness and a feeling of 'freeze' in the work of the heart. An ECG analysis revealed: sinus rhythm, heart rate - 76/min. Periodically, widespread ventricular complexes lasting 0.22 s, of an irregular shape, followed by complete compensatory pauses, appeared. What complication occurred in this patient?

Повна блокада лівої ніжки пучка Гіса Complete blockade of the left leg of the bundle of His

Передсердна екстрасистолія Atrial extrasystole

Шлуночкова екстрасистолія Ventricular extrasystole

Повна блокада правої ніжки пучка Гіса Complete blockade of the right leg of the bundle of His

Атріовентрикулярна екстрасистолія Atrioventricular extrasystole

72 / 200
Рідні чоловіка 36-ти років звернулися до лікаря зі скаргами на зміни у його поведінці. Під час опитування чоловік повідомив, що чує голоси у своїй голові. Його мовою хтось керує та примушує говорити. Він зазначає, що бачить події, які трапляються за межами кімнати у якій він знаходиться. Думки його плинуть самі по собі, всупереч його бажанням. Інколи навіть два потоки думок одночасно. Під час подальшого опитування він повідомив лікаря, що має підозру щодо свого перебування під наглядом засекреченої наукової організації, яка ставить над ним експерименти. Який першочерговий висновок щодо стану пацієнта буде найбільш точним? Relatives of a 36-year-old man turned to the doctor with complaints about changes in his behavior. During the interview, the man reported that he hears voices in his head. In his language, someone controls and compels him to speak. He states that he sees events that occur outside the room he is in. His thoughts flow by themselves, against his will. Sometimes even two streams of thought at once. On further questioning, he told the doctor that he had suspicion of being under the supervision of a classified scientific organization that is experimenting on him. What primary conclusion about the patient's condition would be the most accurate?

Синдром психічного автоматизму Psychic automatism syndrome

Психосенсорний розлад Psychosensory disorder

Синдром слухового галюцинозу Syndrome of auditory hallucinosis

Параноїдний синдром Paranoid Syndrome

Парафренний синдром Paraphrenic syndrome

73 / 200
У хворої 10-ти років на тлі хронічної ниркової недостатності продовжує прогресувати анемічний синдром. Вкажіть препарат вибору для патогенетичного лікування вказаного синдрому: Anemic syndrome continues to progress in a 10-year-old patient against the background of chronic renal failure. Specify the drug of choice for the pathogenetic treatment of the specified syndrome:

Еритропоетин Erythropoietin

Еритроцитарна маса Erythrocyte mass

Ціанокобаламін Cyanocobalamin

Препарати заліза Iron preparations

Фолієва кислота Folic acid

74 / 200
Чоловік 52-х років звернувся до лікаря зі скаргами на безсоння, роздратованість, підвищену збудливість, підвищену пітливість, кровоточивість ясен. Працює на виробництві медичного обладнання у відділі, що спеціалізується на конструюванні сфігмоманометрів та термометрів. При фізикальному обстеженні неритмічний та асиметричний тремор пальців витягнутих рук, підвищена збудливість вегетативної нервової системи, червоний дермографізм, гінгівіт, стоматит, ціанотична смуга вздовж країв ясен.При лабораторному дослідженні крові виявлено лімфоцитоз і моноцитоз. Хронічне отруєння якою речовиною є найбільш імовірним у пацієнта? A 52-year-old man consulted a doctor with complaints of insomnia, irritability, increased excitability, increased sweating, bleeding gums. He works in the production of medical equipment in the department that specializes in the construction of sphygmomanometers and thermometers. During a physical examination, non-rhythmic and asymmetric tremor of the fingers of outstretched hands, increased excitability of the autonomic nervous system, red dermographism, gingivitis, stomatitis, a cyanotic band along the edges of the gums. A laboratory blood test revealed lymphocytosis and monocytosis. Chronic poisoning by what substance is most likely in the patient?

Свинцем Lead

Бензолом Benzol

Марганцем Manganese

Ртуттю Mercury

Чотирихлористим вуглецем Carbon tetrachloride

75 / 200
Дитина, що народилась 1 день тому, проходить огляд у зв'язку з виникненням висипу. Її мати на облік у жіночу консультацію не ставала. Після народження дитина оцінена за шкалою Апгар у 7 та 9 балів на 1 та 5 хвилині відповідно. При фізикальному обстеженні температура - 37°С,пульс - 145/хв., частота дихання - 33/хв. При аускультації серця вислуховується систоло- діастолічний шум у лівій підключичній ділянці. При огляді по всьому тілу розсіяні петехії. Який збудник найбільш імовірно викликав інфікування дитини? A child who was born 1 day ago is being examined due to the appearance of a rash. Her mother was not registered at the women's consultation. After birth, the child was assessed for on the Apgar scale of 7 and 9 points at 1 and 5 minutes, respectively. During the physical examination, the temperature is 37°C, the pulse is 145/min., the respiratory rate is 33/min. During auscultation of the heart, a systolic-diastolic murmur is heard in the left subclavian area. On examination, petechiae are scattered all over the body. What pathogen most likely caused the child's infection?

Цитомегаловіру Cytomegalovirus

Токсоплазмоз Toxoplasmosis

Краснуха Krasnukha

Вірус простого герпесу Herpes simplex virus

Сифіліс Syphilis

76 / 200
Здорова дівчина 16-ти років прийшла на профілактичний медичний огляд. Під час збору анамнезу вона повідомила, що у З роки перехворіла на вітряну віспу. Інших серйозних захворювань в анамнезі немає. Батьки відмов від щеплень не писали, вакцинувалася у дитячому садочку та школі відповідно до Національного календаря профілактичних щеплень. Фізикальне обстеження без відхилень. Призначення яких вакцин буде найбільш доречним на даний момент? A healthy 16-year-old girl came for a preventive medical examination. During the collection of anamnesis, she reported that she contracted chicken pox in the year 3. Other serious diseases in the anamnesis no. Parents did not write refusals from vaccinations, she was vaccinated in kindergarten and school in accordance with the National Calendar of preventive vaccinations. Physical examination without abnormalities. Which vaccines would be the most appropriate to prescribe at the moment?

АДП-М. ВПЛ ADP-M. VPL

Гепатит В. АДП-М Hepatitis B. ADP-M

КПК, АДП, ВПЛ KPC, ADP, VPL

ІПВ. АаКДП IPV. AaKDP

Гепатит В, АаКДП Hepatitis B, AaKDP

77 / 200
У лікаря-стоматолога 26-ти років при обстеженні виявили помірну гіперферментемію, позитивний тест на HBsAg. Його дружина 24-х років не має щеплення від вірусного гепатиту В. Дитина 2-х років належно вакцинована за віковим графіком. Яка профілактика вірусного гепатиту В повинна бути проведена у родині? A 26-year-old dentist was diagnosed with moderate hyperfermentemia during the examination, a positive test for HBsAg. His 24-year-old wife is not vaccinated against viral hepatitis B. A 2-year-old child is properly vaccinated according to the age schedule. What prevention of viral hepatitis B should be carried out in the family?

Призначення дружині специфічної хіміопрофілактики ненуклеозидними аналогами Prescribing specific chemoprophylaxis to the wife with non-nucleoside analogues

Призначення специфічного імуноглобуліну дружині та дитині Prescribing specific immunoglobulin to wife and child

Призначення специфічного імуноглобуліну дитині Prescribing specific immunoglobulin to a child

Негайна вакцинація дружини при негативному тесті на HBsAg Immediate vaccination of wife with a negative HBsAg test

Призначення дружині та дитині специфічної хіміопрофілактики ненуклеозидними аналогами Prescribing specific chemoprophylaxis to the wife and child with non-nucleoside analogs

78 / 200
Жінка 27-ми років на 8-му тижні вагітності скаржиться лікарю, що за останні 8 днів вона відмічає тривалу нудоту та блювання майже після усіх прийомів їжі. За останній тиждень пацієнтка втратила 3кг ваги. Зараз при рості 160 см жінка важить 46 кг. Пульс - 100/хв., артеріальний тиск - 90/50 мм рт.ст. При огляді відмічаються сухість слизових оболонок, зниження тургору шкіри та астенічна статура. Гінекологічний огляд виявив розмір матки, що відповідає 8-му тижню вагітності, без патологічних змін. На УЗД виявлена вагітність одним плодом. Концентрація гемоглобіну - 150 г/л. У загальному аналізі сечі виявлені кетонові тіла (+++). Що із перерахованого є найбільш доречним наступним кроком у веденні пацієнтки? A 27-year-old woman in the 8th week of pregnancy complains to the doctor that for the past 8 days she has been experiencing prolonged nausea and vomiting after almost every meal. Over the last a week, the patient lost 3 kg of weight. Now, with a height of 160 cm, the woman weighs 46 kg. Pulse - 100/min., blood pressure - 90/50 mm Hg. Dryness of the mucous membranes, decreased skin turgor, and asthenic physique are noted during the examination. Gynecological examination revealed the size of the uterus, corresponding to the 8th week of pregnancy, without pathological changes. Ultrasound revealed a pregnancy with one fetus. The concentration of hemoglobin - 150 g/l. In the general analysis of urine, ketone bodies (+++) were found. Which of the following is most appropriate the next step in the management of the patient?

Постільний режим та часте годування малими порціями Bed rest and frequent feeding in small portions

Внутрішньовенна інфузійна терапія та призначення антиеметиків Intravenous infusion therapy and prescription of antiemetics

Внутрішньовенне введення в-адрено блокаторів та парентеральне харчування Intravenous administration of adrenoblockers and parenteral nutrition

Ендоскопічне дослідження та промивання шлунка Endoscopy and gastric lavage

Пероральний прийом антиеметиків та антихолінергічних препаратів Oral intake of antiemetics and anticholinergic drugs

79 / 200
Дівчина 22-х років звернулася до лікаря зі скаргами на задишку, загрудинний біль, відчуття страху та 'стороннього тіла' у горлі. Зазначені симптоми з'явилися в момент перегляду фільму з друзями і вже протягом 40 хвилин зберігаються і не зникають. Пацієнтка стверджує, що подібний напад у неї не вперше, але раніше вона могла опанувати цей стан та заспокоїтися. При фізикальному обстеженні перед лікаремтремтяча та спітніла дівчина, температура тіла - 37,5°С,артеріальний тиск - 130/90 мм рт.ст., пульс - 112/хв., частота дихання - 18/хв., SpO2 - 99% при кімнатному повітрі. Лікар не може заспокоїти пацієнтку. Вона стверджує, що помирає та благає про допомогу. Який наступний крок лікаря буде найбільш доречним? A 22-year-old girl went to the doctor with complaints of shortness of breath, chest pain, a feeling of fear and a 'foreign body' in her throat. These symptoms appeared at the moment watching a movie with friends and for 40 minutes they persist and do not disappear. The patient claims that this is not the first time she has had such an attack, but before she was able to master this condition and calm down. During a physical examination before the doctor, the girl is trembling and sweaty, body temperature - 37.5 °С, blood pressure - 130/90 mm Hg, pulse - 112/min, respiratory rate - 18/min, SpO2 - 99% on room air. The doctor cannot calm the patient down. She claims that she is dying and pleads for help. What would be the most appropriate next step for the doctor?

Призначити нітрогліцерин перорально та спостерігати мінімум 5 хвилин Give nitroglycerin orally and observe for at least 5 minutes

Ввести метопролол внутрішньовенно Enter metoprolol intravenously

Призначити аспірин перорально та доставити до найближчого реперфузійного центру Prescribe aspirin orally and deliver to the nearest reperfusion center

Ввести феназепам внутрішньовенно Inject phenazepam intravenously

Розпочати курс флуоксетину перорально Start oral fluoxetine course

80 / 200
Хворий 57-ми років перебуває на лікуванні з приводу політравми. Через 8 діб виникли скарги на виражений кашель з виділенням гнійного харкотиння, виражену задишку, підвищення температури до 38,8°С. В анамнезі: хронічний бронхіт, палить 20 років. Об'єктивно: в легенях вислуховується жорстке дихання, в нижніх відділах правої легені - вологі дрібнопухирчасті хрипи. Проба на ВІЛ - негативна. Який діагноз у даного хворого? A 57-year-old patient is being treated for polytrauma. After 8 days, there were complaints of severe cough with purulent sputum, severe shortness of breath, temperature rise to 38, 8°C. History: chronic bronchitis, has been smoking for 20 years. Objectively: hard breathing is heard in the lungs, moist fine-vesicular rales in the lower parts of the right lung. The HIV test is negative. What is the diagnosis of this patient?

Нозокоміальна пневмонія Nosocomial pneumonia

Позагоспітальна пневмонія Community-acquired pneumonia

Аспіраційна пневмонія Aspiration pneumonia

Пневмонія осіб з імунодефіцитом Pneumonia of persons with immunodeficiency

81 / 200
Жінка 28-ми років протягом 12-ти років хворіє на хронічний гломерулонефрит, який весь цей час мав латентний перебіг. За останні півроку виникла загальна слабкість, зниження апетиту, працездатності, нудота. Хвора скаржиться на головний біль, біль у суглобах. При обстеженні: анемія, підвищення рівня сечовини крові до 34,5 ммоль/л, креатинін крові - 766 мкмоль/л, гіперкаліємія. Чим ускладнився перебіг хвороби? A 28-year-old woman has been suffering from chronic glomerulonephritis for 12 years, which had a latent course all this time. Over the past six months, general weakness, decreased appetite, work capacity, nausea. The patient complains of headache, pain in the joints. During the examination: anemia, blood urea level increased to 34.5 mmol/l, blood creatinine - 766 μmol/l, hyperkalemia. What complicated the course of the disease?

Пієлонефритом Pyelonephritis

Амілоїдозом нирок Kidney amyloidosis

Хронічною нирковою недостатністю Chronic renal failure

Гострою нирковою недостатністю Acute kidney failure

Нефротичним синдромом Nephrotic syndrome

82 / 200
Пацієнтка 45-ти років скаржиться на дискомфорт під час читання, почервоніння краю повік, білі пінисті виділення в кутах очних щілин протягом останніх двох місяців. Об'єктивно спостерігаються гіперемія та потовщення вільного краю повік, розширення вивідних протоків залоз хряща повік. Яким буде діагноз? A 45-year-old female patient complains of discomfort while reading, redness of the edge of the eyelids, white foamy discharge in the corners of the eye sockets for the past two months. Hyperemia is objectively observed and thickening of the free edge of the eyelids, expansion of the excretory ducts of the eyelid cartilage glands. What will be the diagnosis?

Аденовірусний кон'юнктивіт Adenoviral conjunctivitis

Бленорейний кон'юнктивіт Blenorrhoeic conjunctivitis

Хронічний каналікуліт Chronic canaliculitis

Гострий дакріоаденіт Acute dacryoadenitis

Мейбомієвий блефарит Meibomian blepharitis

83 / 200
У породіллі 22-х років після відходження вод з'явились безперервні, дуже болючі перейми. Об'єктивно: розміри таза 25-28-31-21 см, маса плода - 4200 г. Матка постійно в тонусі, контракційне кільце на рівні пупка. Нижній сегмент матки болючий. При піхвовому дослідженні: шийка матки відкрита повністю, плідного міхура немає, голівка плода виповнює термінальну лінію, сагітальний шов в прямому розмірі. Серцебиття плода - 136/хв. Яка тактика лікаря буде найбільш доречною? A 22-year-old woman in labor had continuous, very painful contractions after her water broke. Objectively: pelvic dimensions 25-28-31-21 cm, fetal weight - 4200 g. The uterus is constantly in tone, the contraction ring is at the level of the navel. The lower segment of the uterus is painful. During vaginal examination: the cervix is completely open, there is no amniotic sac, the fetal head fills the terminal line, the sagittal suture is straight. The fetal heartbeat - 136/min. Which tactics of the doctor will be the most appropriate?

Виключити пологову діяльність та зробити кесарів розтин Exclude labor and perform a caesarean section

Накласти акушерські щипці Apply obstetric forceps

Зробити плодоруйнівну операцію Perform a destructive operation

Провести вакуум-екстракцію плода Carry out vacuum extraction of the fetus

Зробити комбінований поворот плода на ніжку, з наступною його екстракцією Make a combined rotation of the fetus on the leg, followed by its extraction

84 / 200
Молода жінка звернулася до лікаря зі скаргами на задишку при незначному фізичному навантаженні, інколи набряки на ногах, слабкість. В анамнезі часті запальні захворювання легень. Задишка вперше з'явилась у дитинстві та поступово зростала. При фізикальному обстеженні пульс - 78/хв., артеріальний тиск - 130/80 мм рт.ст., шкіра ціанотична, набряки на ногах. Аускультативно тони серця ритмічні, грубий систолічний шум уздовж лівого краю груднини, акцент та розщеплення II тону над легеневою артерією. Пальпаторно печінка збільшена. Вибір лікарем якого діагностичного методу буде найбільш доречним? A young woman consulted a doctor with complaints of shortness of breath during minor physical exertion, sometimes swelling of the legs, weakness. She has a history of frequent inflammatory lung diseases. Shortness of breath appeared for the first time in childhood and gradually increased. During physical examination, pulse - 78/min., arterial pressure - 130/80 mm Hg, cyanotic skin, edema on the legs. Auscultatory heart sounds are rhythmic, rough systolic murmur along the left edge of the sternum, accent and splitting of the II tone above the pulmonary artery. The liver is enlarged on palpation. Which diagnostic method will be the most appropriate for the doctor to choose?

Біопсія міокарда Myocardial biopsy

Коронароангіографія Coronary angiography

Електрокардіограма Electrocardiogram

Катетеризація серця Heart Catheterization

Ехокардіографія Echocardiography

85 / 200
Жінка 46-ти років доставлена до відділення невідкладної допомоги зі скаргами на постійний, інтенсивний абдомінальний біль протягом 8 годин, нудоту та блювання. В анамнезі гіпертригліцеридемія, яку пацієнтка не лікувала. При фізикальному обстеженні температура тіла - 38°С, артеріальний тиск - 100/60 мм рт.ст., пульс - 122/хв. Іктеричність кон'юнктив, при пальпації черевної стінки позитивний симптом флюктуації, позитивні симптоми Хвостека та Труссо. При лабораторному дослідженні в сироватці крові натрій -142 ммоль/л, калій - 3,1 ммоль/л, бікарбонат - 32 ммоль/л, креатинін - 106 мкмоль/л, кальцій - 0,9 ммоль/л. Який перший крок у веденні пацієнтки буде найбільш доречним? A 46-year-old woman was brought to the emergency department with complaints of constant, intense abdominal pain for 8 hours, nausea and vomiting. She has a history of hypertriglyceridemia, which the patient does not During physical examination, body temperature was 38°C, blood pressure was 100/60 mmHg, pulse was 122/min, conjunctival icterus, positive symptoms of fluctuation on palpation of the abdominal wall, positive symptoms of Khvostek and Trousseau. During a laboratory study, sodium - 142 mmol/l, potassium - 3.1 mmol/l, bicarbonate - 32 mmol/l, creatinine - 106 μmol/l, calcium - 0.9 mmol/l. What is the first step in management patients will be the most appropriate?

Ендоскопічна ретроградна холангіопан- креатографія (ЕРХПГ) Endoscopic retrograde cholangiopancreatography (ERCP)

Апротиніну 10 000 АТрОд (антитрипсинових одиниць) внутрішньовенно Aprotinin 10,000 ATrOd (antitrypsin units) intravenously

Негайна літотрипсія Immediate lithotripsy

Визначення концентрації алкоголю в крові Determination of alcohol concentration in blood

Натрію хлориду 0,9%, кальцію глюконат та фентаніл внутрішньовенно Sodium chloride 0.9%, calcium gluconate and fentanyl intravenous

86 / 200
Дівчина 22-х років прийшла до лікаря на планове обстеження. Вона викурює 1 пачку сигарет у день протягом 5-ти років. Має одного постійного статевого партнера, користуються презервативами. Дідусь по татовій лінії помер від серцевого нападу у 60 років. При фізикальному обстеженні пульс - 78/хв.. частота дихання - 14/хв., артеріальний тиск - 110/70 мм рт.ст. При аускультації серця вислуховується голосистолічний шум у II міжребер'ї зліва від грудини. Яка рекомендація лікаря цій пацієнтці буде найбільш доречною? A 22-year-old girl came to the doctor for a routine examination. She has smoked 1 pack of cigarettes a day for 5 years. She has one permanent sexual partner, uses condoms . The paternal grandfather died of a heart attack at the age of 60. During physical examination, the pulse is 78/min., the respiratory rate is 14/min., the blood pressure is 110/70 mm Hg. On auscultation of the heart, a holosystolic murmur is heard in II intercostal space to the left of the sternum. Which doctor's recommendation would be most appropriate for this patient?

Кольпоскопія з біопсією Colposcopy with biopsy

Пройти ПАП-тест та здати аналіз на ВПЛ Pass a Pap test and pass an analysis for HPV

Скринінг гіперліпідемії Screening for hyperlipidemia

Навчитися самообстеженню молочних залоз Learn breast self-examination

Пройти ПАП-тест Pass the Pap test

87 / 200
Хворий 42-х років доставлений до травматологічного відділення 2 години тому. Рентгенологічно: перелом кісток тазу. Об'єктивно: виведений з шоку. Самостійно не мочиться. Уретрорагія. Пальпується збільшений сечовий міхур. Болісна припухлість в ділянці промежини. Який попередній діагноз? A 42-year-old patient was brought to the trauma department 2 hours ago. X-ray: pelvic bone fracture. Objectively: brought out of shock. He does not urinate on his own. Urethrorrhagia. An enlarged bladder is palpated. Painful swelling in the perineum. What is the previous diagnosis?

Гостра ниркова недостатність Acute renal failure

Травма уретри Urethra trauma

Забій промежини Perineal bruise

Травма сечового міхура Bladder injury

Рефлекторна затримка сечовипускання Reflex micturition delay

88 / 200
У повторнороділлі 30-ти років пологи тривають 8 годин. Перейми через кожну хвилину по 50 секунд, активні. Серцебиття плоду - 156/хв., ритмічне. Під час зовнішнього дослідження голівка розташована в порожнині малого тазу. Вагінально: розкриття шийки матки повне, голівка плоду в площині виходу з малого тазу. Стрілоподібний шов в прямому розмірі, мале тім'ячко біля лона. Який це період пологів? In a repeat birth of 30 years, labor lasts 8 hours. Contractions every minute for 50 seconds, active. Fetal heartbeat - 156/min, rhythmic. During external examination, the head is located in the cavity of the small pelvis. Vaginal: opening of the cervix is complete, the fetal head is in the plane of exit from the small pelvis. Arrow-shaped seam in a straight size, small crown near the womb. What is the period of childbirth?

Прелімінарний період Preliminary Period

Латентна фаза першого періоду нормальних пологів Latent phase of the first period of normal childbirth

Стрімкі пологи Rapid childbirth

Активна фаза першого періоду нормальних пологів Active phase of the first period of normal childbirth

Другий період нормальних пологів Second period of normal childbirth

89 / 200
Вагітна 22-х років звернулась до жіночої консультації зі скаргами на кровотечу зі статевих шляхів, що розпочалася 2 дні тому, сильну втому та запаморочення. Термін вагітності 13 тижнів. За день вона використала 6 гігієнічних прокладок. При фізикальному обстеженні: артеріальний тиск - 90/60 мм рт.ст., температура - 37,8°6\ пульс - 125/хв., частота дихання - 15/хв. Розпочата інфузійна терапія. При вагінальному дослідженні: кров у піхві, шийка матки згладжена та розкрита. На УЗД плідне яйце в порожнині матки, серцебиття і рухи плода не визначаються. Який з наступних кроків у веденні пацієнтки буде найбільш доречним? A 22-year-old pregnant woman came to the women's consultation with complaints of bleeding from the genital tract that started 2 days ago, severe fatigue and dizziness. The pregnancy is 13 weeks. During the day, she used 6 sanitary napkins.On physical examination: blood pressure - 90/60 mmHg, temperature - 37.8°6\ pulse - 125/min, respiratory rate - 15/min.Infusion therapy was started. During a vaginal examination: blood in the vagina, the cervix is smoothed and opened. On ultrasound, a fertile egg is found in the uterine cavity, the heartbeat and movements of the fetus are not detected. Which of the following steps in the management of the patient would be the most appropriate?

Постільний режим та анальгетики Bed rest and analgesics

Метотрексат Methotrexate

Дилатація та кюретаж матки Dilatation and curettage of the uterus

Магнію сульфат внутрішньовенно Magnesium sulfate intravenously

Антибіотикотерапія Antibiotic therapy

90 / 200
Чоловік 33-х років прийшов до лікаря на профілактичний прийом. Скарг не має. При фізикальному обстеженні лікарем випадково виявлено грубий систолічний шум в точці Боткіна, який проводиться в точку аускультації верхівки та на судини шиї. Від подальшого обстеження пацієнт відмовився. Через 2 місяці на фоні грипу у пацієнта розвинулися симптоми серцевої недостатності. Яку ваду найбільш імовірно було виявлено лікарем? A 33-year-old man came to the doctor for a preventive appointment. He has no complaints. During the physical examination, the doctor accidentally discovered a rough systolic murmur at Botkin's point, which is performed at the point auscultation of the apex and on the vessels of the neck. The patient refused further examination. 2 months later, the patient developed symptoms of heart failure against the background of the flu. What defect was most likely detected by the doctor?

Аортальний стеноз Aortic stenosis

Аортальна недостатність Aortic insufficiency

Мітральний стеноз Mitral stenosis

Мітральна недостатність Mitral regurgitation

Вроджене ураження тристулкового клапана Congenital tricuspid valve lesion

91 / 200
До приймального покою швидкою допомогою доставлено хворого 46-ти років зі скаргами на різкий, нападонодібний біль у правій поперековій ділянці, що іррадіює у пахвинну ділянку та на внутрішню поверхню стегна. Біль з'явився раптово кілька годин тому. Напередодні у хворого з'явилася профузна безболісна гематурія зі згустками крові черв'якоподібної форми. Раніше нічим не хворів. Яке захворювання можна припустити в першу чергу? A 46-year-old patient was brought to the reception room by ambulance with complaints of sharp, attack-like pain in the right lumbar area, radiating to the inguinal area and to the inner surface of the thigh . The pain appeared suddenly a few hours ago. The day before, the patient had profuse painless hematuria with worm-shaped blood clots. He had never been sick before. What disease can be assumed first of all?

Некротичний папіліт Necrotic papillitis

Рак правої нирки Right kidney cancer

Сечокам'яна хвороба, камінь правої нирки Urolithiasis, right kidney stone

Пухлина сечового міхура Bladder tumor

Гострий гломерулонефрит Acute glomerulonephritis

92 / 200
При вивченні середнього рівня та характеру різноманітності деяких лабораторних показників отримані такі дані: для загального білку крові - середнє квадратичне відхилення ±4 г/л, коефіцієнт варіації - 6%; для швидкість осідання еритроцитів відповідно ±2 мм/год, 23%. Яка з ознак, що вивчаються, є найбільш різноманітною? When studying the average level and nature of the diversity of some laboratory indicators, the following data were obtained: for total blood protein - mean square deviation ±4 g/l, coefficient of variation - 6% ; for the sedimentation rate of erythrocytes, respectively, ±2 mm/h, 23%. Which of the signs being studied is the most diverse?

Швидкість осідання еритроцитів (ШОЕ) Erythrocyte Sedimentation Rate (ESR)

Для вивчення різноманітності потрібні додаткові дослідження More research is needed to study diversity

Загальний білок сироватки крові Total serum protein

Відмінності в різноманітності ознак відсутні There are no differences in the variety of signs

Для вивчення різноманітності погрібні додаткові розрахунки To study diversity, additional calculations are required

93 / 200
Чоловік 58-ми років звернувся до лікаря зі скаргами на кашель зі слизово-гнійним мокротинням, задишку при фізичному навантаженні. Зазначені симптоми турбують протягом 14-ти років. Щоденно викурює 1,5 пачки сигарет з 17-ти років. При фізикальному обстеженні пульс - 96/хв., артеріальний тиск - 130/80 мм рт.ст. Перкуторно над усією поверхнею легень коробковий звук. Аускультативно дихання ослаблене, на фоні подовженого видиху розсіяні сухі хрипи. За даними спірометрії співвідношення ОФВ1/ФЖЄЛ - 30%. Який діагноз є найбільш імовірним? A 58-year-old man consulted a doctor with complaints of a cough with mucous-purulent sputum, shortness of breath during physical exertion. These symptoms have been bothering him for 14 years. Daily smokes 1.5 packs of cigarettes since the age of 17. During physical examination, pulse - 96/min., blood pressure - 130/80 mm Hg. Percussion over the entire surface of the lungs, a box sound. Auscultatory breathing is weakened, against the background of prolonged exhalation, scattered dry wheezing. According to spirometry, the FEV1/FJEL ratio is 30%. What diagnosis is the most likely?

Рак легень Lung cancer

Бронхіальна астма Bronchial asthma

Хронічне обструктивне захворювання легень (ХОЗЛ) Chronic obstructive pulmonary disease (COPD)

Бронхоектатична хвороба Bronchoectatic disease

Фіброзуючий альвеолі Alveolar fibrosis

94 / 200
Хвора 36-ти років скаржиться на задишку, відчуття стискання в правій половині грудної клітки, підвищення температури до 38,7°С, кашель з виділенням невеликої кількості слизово-гнійного харкотиння. Хворіє понад тиждень. Скарги пов'язує з переохолодженням. Об'єктивно; легкий акроціаноз губ, пульс ритмічний. 90/хв.. артеріальний тиск - 140/85 мм рт.ст. Права половина грудної клітки відстає в акті дихання. Перкуторно справа нижче кута лопатки прослуховується тупість. В цій ділянці дихання відсутнє. Який найбільш імовірний діагноз? A 36-year-old patient complains of shortness of breath, a feeling of tightness in the right half of the chest, an increase in temperature to 38.7°C, a cough with the release of a small amount of mucous purulent sputum. Sick for more than a week. Complaints are associated with hypothermia. Objectively, slight acrocyanosis of the lips, rhythmic pulse. 90/min. arterial pressure - 140/85 mm Hg. The right half of the chest lags behind in the act of breathing. Dullness is heard on percussion on the right below the angle of the scapula. There is no breathing in this area. What is the most likely diagnosis?

Плеврит Pleuriitis

Ателектаз легені Atelectasis of the lung

Абсцес легені Lung abscess

Позагоспітальна пневмонія Community-acquired pneumonia

ТЕЛА BODIES

95 / 200
Чоловік 72-х років звернувся до лікаря зі скаргами на задишку, кашель, набряки гомілок, збільшення живота. Хворіє більше ніж 20 років на хронічне обструктивне захворювання легень (ХОЗЛ). Останні З роки є інвалідом II групи у зв'язку з патологією серця. При фізикальному обстеженні пульс - 92/хв, артеріальний тиск - 120/70 мм рт.ст., частота дихання - 24/хв. Шкірні покриви ціанотичні, набряки гомілок, при пальпації органів черевної порожнини позитивний симптом флуктуації. При аускультації акцент II тону над легеневою артерією, сухі хрипи над всією поверхнею легень. Який механізм розвитку змін з боку серця у цього хворого є найбільш імовірним? A 72-year-old man went to the doctor with complaints of shortness of breath, cough, swelling of the legs, an enlarged abdomen. He has been suffering from chronic obstructive pulmonary disease (COPD) for more than 20 years ). For the last 3 years, he has been disabled of the II group due to heart pathology. During the physical examination, the pulse is 92/min, blood pressure - 120/70 mm Hg, respiratory rate - 24/min. The skin is cyanotic, edematous shins, during palpation of the abdominal organs, a positive symptom of fluctuation. During auscultation, the accent of the II tone over the pulmonary artery, dry rales over the entire surface of the lungs. What is the most likely mechanism of development of changes in the heart in this patient?

Кардіоваскулярний рефлекс Cardiovascular reflex

Рефлекс Китаєва Reflex Kitaev

Дихальний рефлекс Respiratory reflex

Рефлекс Бейнбріджа Bainbridge Reflex

Вторинна легенева гіпертензія Secondary pulmonary hypertension

96 / 200
Чоловік 19-ти років, звернувся до лікаря зі скаргами на безболісні ерозії на голівці статевого члена, які з'явилися 2-3 дні тому. У анамнезі незахищений статевий контакт приблизно місяць тому. При обстеженні на голівці статевого члена ерозія овальної форми діаметром до 5 мм, безболісна, яскраво-червоного кольору. Пахвинні лімфовузли не збільшені. Яке першочергове лабораторне дослідження буде найбільш доречним? A 19-year-old man turned to the doctor with complaints of painless erosions on the head of the penis that appeared 2-3 days ago. He has a history of unprotected sex contact about a month ago. When examining the head of the penis, an oval-shaped erosion up to 5 mm in diameter, painless, bright red in color. Inguinal lymph nodes are not enlarged. What primary laboratory test would be most appropriate?

Мікрореакція преципітації з кардіоліпіновим антигеном Microreaction of precipitation with cardiolipin antigen

Реакція імунофлюоресценції Immunofluorescence reaction

Культуральне дослідження Cultural Research

Реакція іммобілізації блідих трепонем Reaction of immobilization of pale treponems

97 / 200
Хлопчика 2-х років госпіталізовано з приводу зменшення маси тіла, нестійких випорожнень, анорексії, які з'явилися після введення в раціон манної каші (з 5 місяців). Дитина адинамічна, млява, шкіра бліда, суха, підшкірно-жировий шар відсутній. Живіт здутий, напружений. Під час перкусії у верхній частині живота тимпаніт. шум плеску, випорожнення пінисті, світлого кольору, смердючі. В копрооцитограмі: нейтральний жир - багато. Який наступний крок у веденні пацієнта буде найбільш доречним? A 2-year-old boy was hospitalized due to a decrease in body weight, unstable stools, anorexia, which appeared after the introduction of semolina into the diet (from 5 months). The child is adynamic, lethargic, the skin is pale, dry, the subcutaneous fat layer is absent. The abdomen is distended, tense. During percussion in the upper part of the abdomen, tympanitis. Slap noise, stools are foamy, light-colored, smelly. In the coprocytogram: neutral fat - a lot. What What would be the most appropriate next step in patient management?

Розробка індивідуального плану харчування з великим вмістом клітковини Development of an individual meal plan with high fiber content

Рентгенографія органів черевної порожнини X-ray of abdominal organs

Негайне оперативне втручання Immediate operative intervention

Призначення антибіотиків широкого спектру дії Prescription of broad-spectrum antibiotics

Визначення *Ig*А до тканинної тран*c*глютамінази Determination of *Ig*A to tissue trans*c*glutaminase

98 / 200
Хворий 48-ми років госпіталізований з нападами судом. Багато років страждає на виразку 12-ти палої кишки (ДПК). Протягом останнього місяця спостерігає щоденне блювання, схуд на 20 кг. Об'єктивно: хворий виснажений. В епігастральній ділянці визначається 'шум плеску'. Нижня межа шлунка на рівні гребінцевої лінії. Лабораторно: загальний білок - 47 г/л; К - 2,1 ммоль/л, Na -118 ммоль/л,Са -1,6 ммоль/л, хлориди - 82 ммоль/л, гематокрит - 64%. Який попередній діагноз? A 48-year-old patient is hospitalized with seizures. He has been suffering from a duodenal ulcer for many years. During the last month, he has been vomiting daily, lost weight 20 kg. Objectively: the patient is exhausted. In the epigastric area, a 'slap sound' is detected. The lower border of the stomach is at the level of the comb line. Laboratory: total protein - 47 g/l; K - 2.1 mmol/l, Na - 118 mmol /l, Ca - 1.6 mmol/l, chlorides - 82 mmol/l, hematocrit - 64%. What is the previous diagnosis?

Субкомпенсований стеноз воротаря Undercompensated stenosis of the goalkeeper

Кровоточива виразка 12-ти палої кишки Bleeding duodenal ulcer

Компенсований стеноз воротаря Compensated stenosis of the goalkeeper

Малігнізована виразка шлунка Malignant gastric ulcer

Декомпенсований стеноз воротаря Decompensated gatekeeper stenosis

99 / 200
Чоловік 36-ти років звернувся до лікаря зі скаргами на серцебиття, пітливість, схуднення на 5-6 кг за останні 3 місяці. Апетит збережений. При фізикальному обстеженні: артеріальний тиск - 130/60 мм рт.ст., пульс - 140/хв.. шкіра волога, тепла. При пальпації рівномірне збільшення щитоподібної залози. Призначення якого дослідження буде найбільш доречним? A 36-year-old man consulted a doctor with complaints of palpitations, sweating, weight loss of 5-6 kg over the past 3 months. Appetite is preserved. During physical examination: blood pressure - 130/60 mm Hg, pulse - 140/min. skin moist, warm. On palpation, a uniform increase in the thyroid gland. What research would be most appropriate?

Рівень тропонінів у крові Level of troponins in blood

Загальний аналіз сечі General urinalysis

Загальний аналіз крові General blood test

Рівень глюкози крові Blood glucose level

Рівень тиреотропних гормонів у крові Level of thyroid-stimulating hormones in the blood

100 / 200
Хвора 25-ти років скаржиться на наявність декількох щільних, болючих вузлів у правій пахвовій ділянці, підвищення температури тіла до 38°С, загальну слабкість. Хворіє 3 доби. Локально: в правій пахвовій області є три вузлоподібних утворення, які підвищуються над поверхнею шкіри, розмірами від 0,5 см до 1,5 см в діаметрі, різко болючі при пальпації, з чіткими контурами, шкіра над ними багрово-синюшного відтінку, підняття верхньої кінцівки обмежено через біль. Поставте діагноз: A 25-year-old patient complains of the presence of several dense, painful nodes in the right armpit, an increase in body temperature to 38°C, general weakness. She has been ill for 3 days. Locally: in the right axillary region there are three nodular formations that rise above the surface of the skin, the size of which is from 0.5 cm to 1.5 cm in diameter, sharply painful when palpated, with clear contours, the skin above them has a purplish-bluish tint, raising the upper limbs are limited due to pain. Diagnose:

Абсцес Abscess

Гнійний гідраденіт Suppurative hidradenitis

Карбункул Carbuncle

Гнійний лімфаденіт Suppurative lymphadenitis

Фурункул Furnish

101 / 200
У дівчини 19-ти років скарги на головний біль пульсуючого характеру в лівій лобноскронево-тім'яній ділянці, який турбує протягом п'яти років. Головний біль виникає нападоподібно і супроводжується нудотою, блюванням, фонофобією та фотофобією. Тривалість нападу від кількох годин до 2-3 діб. Напад цефалгії часто виникає при перевтомі, емоційному перенавантаженні. Частота нападів 1-2 рази на місяць. Неврологічний статус без патологічних змін. Схожі головні болі були у матері хворої. Який діагноз є найбільш імовірним? A 19-year-old girl complains of a throbbing headache in the left frontotemporal-parietal region, which has been bothering her for five years. The headache occurs in an attack-like manner and is accompanied by nausea, vomiting, phonophobia and photophobia. The duration of an attack is from several hours to 2-3 days. An attack of cephalgia often occurs during overwork, emotional overload. The frequency of attacks is 1-2 times a month. Neurological status without pathological changes. Similar headaches were in the patient's mother. What is the most likely diagnosis?

Лікворно-динамічний головний біль Liquor-dynamic headache

Головний біль напруги Tension headache

Мігрень Migraine

Хронічна цервікокраніалгія Chronic cervicocranialgia

Кластерний головний біль Cluster Headache

102 / 200
У післяопераційного хворого, якому 6 днів тому проведено видалення нижньої частки лівої легені в зв'язку з бактеріальною деструкцією і розвитком пневмотораксу, наросла задишка до 48/хв., РаО2знизився до 50 мм рт.ст., РаСО2 підвищився до 65 мм рт.ст., з'явився кашель, виділення великої кількості харкотиння, загальний ціаноз шкіри, гіпотонія - 60/20 мм рт.ст. При аускультації правої і лівої легень - маса різнокаліберних хрипів, різко ослаблене дихання. Який об'єм невідкладної допомоги потрібно надати хворому першочергово для ліквідації дихальної недостатності? In a postoperative patient who had the lower lobe of the left lung removed 6 days ago due to bacterial destruction and the development of pneumothorax, shortness of breath increased to 48/min. PaO2 decreased to 50 mm Hg, PaCO2 increased to 65 mm Hg, cough appeared, a large amount of sputum, general cyanosis of the skin, hypotension - 60/20 mm Hg. When auscultating the right and left lungs - mass of various caliber wheezes, sharply weakened breathing. What volume of emergency aid should be provided to the patient as a priority to eliminate respiratory failure?

Провести лаваж трахеобронхіального дерева Perform tracheobronchial tree lavage

Киснева терапія Oxygen therapy

Положення ортопное Position orthopnea

Провести інтубацію трахеї з ШВЛ Carry out tracheal intubation with ventilator

Пункція плевральної порожнини Puncture of the pleural cavity

103 / 200
Жінка 56-ти років звернулася до лікаря зі скаргами на підвищення температури до 38,5°С, підвищену пітливість вночі та збільшення лімфовузлів в лівій пахвовій та лівій надключичній ділянках. У сімейному анамнезі рак молочної залози у бабусі. При фізикальному обстеженні в зазначених ділянках пальпуються збільшені, безболісні, щільні лімфовузли. При рентгенологічному дослідженні органів грудної порожнини розширення тіні середостіння за рахунок збільшених лімфовузлів. Після ексцизійної біопсії лімфовузла надключичної ділянки, виявлені атипові багатоядерні клітини РідШтернберга. Який діагноз є найбільш імовірним? A 56-year-old woman consulted a doctor with complaints of a temperature rise to 38.5°C, increased sweating at night, and an increase in lymph nodes in the left axillary and left supraclavicular areas . There is a family history of breast cancer in the grandmother. During the physical examination, enlarged, painless, dense lymph nodes are palpable in the indicated areas. During the X-ray examination of the chest cavity, the expansion of the mediastinal shadow due to enlarged lymph nodes. After excisional biopsy of the supraclavicular lymph node, atypical multinucleated Ryd-Sternberg cells were detected . What diagnosis is the most probable?

Бронхогенна карцинома Bronchogenic carcinoma

Метастази раку молочної залози Breast cancer metastases

Лімфома Ходжкіна Hodgkin lymphoma

Туберкульоз Tuberculosis

Інфекційний мононуклеоз Infectious mononucleosis

104 / 200
Під час профілактичного медичного огляду у одного з учнів технічного ліцею виявлені ознаки хейлозу, який проявляється мацерацією епітелію в місці змикання губ, губи яскраво-червоного кольору з одиничними вертикально розташованими тріщинами, які вкриті кірочками червоно-бурого кольору. Дані клінічні прояви найімовірніше пов'язані з недостатнім надходженням з їжею в організм: During a preventive medical examination, one of the students of the technical lyceum showed signs of cheilosis, which is manifested by maceration of the epithelium in the place where the lips close, the lips are bright red with single vertically located cracks, which are covered with red-brown crusts. These clinical manifestations are most likely associated with insufficient intake of food into the body:

Ретинолу Retinol

Кальциферолів Calciferol

Рибофлавіну Riboflavin

Аскорбінової кислоти Ascorbic acid

Тіаміну Thiamine

105 / 200
Хворий 30-ти років працює клепальником протягом 6-ти років. Скарги на різкий біль у плечовому поясі, особливо в нічний час. Пальці рук німіють та біліють при охолодженні. Став дратівливим. Об'єктивно: кисті набряклі, холодні, ціанотичні. Різко знижені всі види чутливості (больова, температурна. вібраційна). При пробі на холод - симптом 'мертвих пальців' Відзначається слабкість у привідних м'язах V пальця; змінена електрозбудженість. Сухожилкові та періостальні рефлекси живі. Який попередній діагноз? A 30-year-old patient has been working as a riveter for 6 years. Complains of sharp pain in the shoulder girdle, especially at night. The fingers are numb and turn white when cooled . Became irritable. Objectively: the hands are swollen, cold, cyanotic. Sharply reduced all types of sensitivity (pain, temperature, vibration). When tested for cold - the symptom of 'dead fingers' There is weakness in the adductor muscles of the V finger; electrical excitability has changed Tendon and periosteal reflexes are alive. What is the previous diagnosis?

Вібраційна хвороба загальної дії Vibration disease of general effect

Синдром Рейно Raynaud's Syndrome

Вібраційна хвороба локальної дії Vibration disease of local effect

Вегетативно-сенсорна полінейропатія Vegetative-sensory polyneuropathy

Вузликовий пері артеріїт Nodular periarteritis

106 / 200
Мати прийшла на прийом до педіатра зробити своїй 2-місячній дитині планові щеплення. Для зручності, вона бажає аби дитині було введено комбіновану 6-компонентну вакцину, що містить антигени усіх збудників, проти яких рекомендовано вакцинуватися у цьому віці. Під час збору анамнезу вона зазначила, що іноді після пробудження її дитина починає раптово напружуватися, згинати голівку, ручки та ніжки. Зазвичай такі напади тривають не більше декількох секунд з інтервалами 'спокою' у 10 секунд. Останній напад був декілька днів тому. Температура тіла - 37,2°С . частота дихання - 25/хв.. пульс - 104/хв., артеріальний тиск -100/70 мм рт.ст. Яке рішення має прийняти лікар щодо подальшої імунізації цієї дитини? A mother came to the pediatrician's appointment to give her 2-month-old child routine vaccinations. For convenience, she wants the child to be given a combined 6-component vaccine containing antigens all pathogens against which vaccination is recommended at this age. During the history taking, she noted that sometimes after waking up, her child begins to tense up suddenly, flexing the head, arms and legs. These attacks usually last no more than a few seconds with intervals of 'rest' of 10 seconds. The last attack was a few days ago. Body temperature - 37.2°C. Breathing rate - 25/min.. Pulse - 104/min., blood pressure - 100/70 mm Hg. What decision should the doctor make regarding further immunization of this child?

Ввести комбіновану 6-компонентну вакцину Enter a combined 6-component vaccine

Провести вакцинацію за схемою АКДП+гепатит В+Ніb-інфекція Vaccinate according to the scheme AKDP+hepatitis B+NiB-infection

Провести вакцинацію за схемою АДП+поліо+гепатит В+Ніb-інфекція Vaccinate according to the scheme ADP+polio+hepatitis B+NiB-infection

Введення будь-яких вакцин на даний момент протипоказано Introduction of any vaccines is currently contraindicated

Провести вакцинацію за схемою гепатит В+Ніb-інфекція Vaccinate according to the scheme hepatitis B + Nib-infection

107 / 200
Хвора 21-го року скаржиться на часті позиви до сечовиділення, біль та печію при сечовипусканні. Захворювання пов'язує з випадковим статевим актом 4 дні тому. При огляді: уретра пальпується у вигляді потовщеного болючого тяжа, набряк та гіперемія зовнішнього отвору сечівника, при натискуванні на уретру виділяється краплина гною. При бактеріоскопічному дослідженні виділень з уретри виявлено розміщені всередині лейкоцитів грамнегативні диплококи. Який діагноз є найбільш імовірним? A 21-year-old patient complains of frequent urges to urinate, pain and heartburn during urination. The disease is associated with accidental sexual intercourse 4 days ago. On examination: the urethra is palpated in the form of a thickened painful mass, swelling and hyperemia of the external opening of the urethra, when pressing on the urethra, a drop of pus is released. Bacterioscopic examination of discharge from the urethra revealed gram-negative diplococci located inside leukocytes. What diagnosis is most likely?

Неспецифічний уретрит Nonspecific urethritis

Уретрит викликаний Neisseria gonorrhoeae Urethritis caused by Neisseria gonorrhoeae

Уретрит викликаний Chlamidiya trachomatis Urethritis caused by Chlamydia trachomatis

Вагініт Vaginitis

Гострий цистит Acute cystitis

108 / 200
Чоловік 32-х років госпіталізований на 5-й день хвороби зі скаргами на підвищення температури до 38,4°С, загальну слабкість, появу в лівій пахвовій ділянці щільного утвору 5x6 см, малоболючого, рухомого. Шкіра над утвором не змінена. Обличчя хворого гіперемоване, пастозне. Частота серцевих скорочень - 95/хв., артеріальний тиск - 100/80 мм рт.ст. В загальному аналізі крові - помірна лейкопенія. Працює ветсанітаром. Який діагноз є найбільш імовірним? A 32-year-old man was hospitalized on the 5th day of illness with complaints of an increase in temperature to 38.4°C, general weakness, the appearance of dense lesion 5x6 cm, mildly painful, mobile. The skin above the lesion is not changed. The patient's face is hyperemic, pasty. Heart rate - 95/min., blood pressure - 100/80 mm Hg. In the general blood test - moderate leukopenia. He works as a veterinarian . What diagnosis is the most probable?

Гідраденіт Hydradenitis

Ієрсиніоз Yersiniosis

Туляремія Tularemia

Доброякісний лімфоретикульоз Benign lymphoreticulosis

Чума Plague

109 / 200
До лікаря звернувся чоловік 36-ти років зі скаргами на пекучий загрудинний біль, що зазвичай виникає через 1-1,5 години після вживання їжі. Зазначає, що біль підсилюється в горизонтальному положенні. При верхній ендоскопії виявлені 2 вогнищевих ураження нижньої третини слизової оболонки стравоходу до 5 мм в межах однієї складки. Яка тактика лікаря буде найбільш доречною? A 36-year-old man went to the doctor with complaints of burning chest pain, which usually occurs 1-1.5 hours after eating. He notes that the pain intensifies in a horizontal position. Upper endoscopy revealed 2 focal lesions of the lower third of the esophageal mucosa up to 5 mm within one fold. What tactics of the doctor would be most appropriate?

Призначення інгібіторів протонної помпи Prescription of proton pump inhibitors

Призначення ненаркотичних анальгетиків Prescription of non-narcotic analgesics

Хірургічне лікування Surgical treatment

Призначення кларитроміцину Clarithromycin Prescription

110 / 200
63-річний чоловік звернувся до лікаря зі скаргами на пекучий та ниючий біль шкіри грудної клітки протягом 5-ти днів. Три дні тому на цій ділянці виникло почервоніння та макулопапульозна висипка, яка через день прогресувала у везикули. По буднях він доглядає двох онуків віком 1 та 3 роки, які щеплені згідно Національного календаря профілактичних щеплень. Фізикальне обстеження виявило везикулярний висип на грудній клітці ззаду в межах дерматомів Т6-Т8 . Неврологічний статус без змін. Який метод постконтактної профілактики дітям буде найбільш ефективним та доречним? A 63-year-old man consulted a doctor with complaints of burning and aching pain in the skin of the chest for 5 days. Three days ago, redness and maculopapular a rash that progressed to vesicles after a day. On weekdays, he cares for his two grandsons, ages 1 and 3, who are vaccinated according to the National Schedule of Preventive Immunizations. Physical examination revealed a vesicular rash on the posterior chest within dermatomes T6-T8. Neurological status is unchanged. What method of post-contact prevention for children will be the most effective and appropriate?

Призначити дітям специфічний імуноглобулін до збудника Varicella Zoster Prescribe specific immunoglobulin for Varicella Zoster to children

Призначити Varicella Zoster-вмісну вакцину дітям Prescribe Varicella Zoster-containing vaccine to children

Уникати контакту з дідусем до зникнення у нього висипу Avoid contact with grandfather until his rash disappears

Призначити терапію ацикловіром дітям Prescribe acyclovir therapy for children

Призначити терапію ацикловіром дідусеві Prescribe acyclovir therapy to grandfather

111 / 200
Хворий 34-ти років перебуває на лікуванні в психіатричній лікарні з приводу загострення шизофренії. Об'єктивно: перебуває в ліжку, рухливо загальмований, контакт відсутній. На запитання не відповідає. Поза одноманітна, пацієнт гіпомімічний, наявні симптом 'хоботка',воскова гнучкість м'язів, симптом 'повітряної подушки'. В такому стані лишається близько тижня. Харчування парентеральне. Визначте наявний синдром розладу рухово-вольової сфери: A 34-year-old patient is being treated in a psychiatric hospital due to an exacerbation of schizophrenia. Objectively: he is in bed, he is physically inhibited, there is no contact. When asked, no responds. The posture is monotonous, the patient is hypomimic, there is a 'proboscis' symptom, waxy muscle flexibility, and an 'air cushion' symptom. He remains in this state for about a week. Parenteral nutrition. Define the existing syndrome of motor-volition disorder:

Кататонічний ступор Catatonic stupor

Екзогенний ступор Exogenous stupor

Депресивний ступор Depressive stupor

Апатичний ступор Apathetic stupor

Психогенний ступор Psychogenic stupor

112 / 200
Чоловік 53-х років хворіє на цукровий діабет протягом 5-ти років. Звертається до лікаря зі скаргами на болі та судоми в нижніх кінцівках, відчуття оніміння в них, мерзлякуватість. При обстеженні нижніх кінцівок ступні звичайного кольору, теплі на дотик, усі види чутливості значно знижені, гіперкератози, пульсація на артеріях кінцівок збережена. Яке ускладнення найбільш імовірно виникло у пацієнта? A 53-year-old man has been suffering from diabetes for 5 years. He goes to the doctor with complaints of pain and cramps in the lower limbs, a feeling of numbness in them, frostbite. When examining the lower extremities, the feet are normal in color, warm to the touch, all types of sensitivity are significantly reduced, hyperkeratosis, pulsation in the arteries of the extremities is preserved. What complication most likely occurred in the patient?

Облітеруючий ендартеріїт Endarteritis obliterans

Синдром діабетичної стопи, нейропатична форма Diabetic foot syndrome, neuropathic form

Синдром діабетичної стопи, змішана форма Diabetic foot syndrome, mixed form

Синдром діабетичної стопи, ішемічна форма Diabetic foot syndrome, ischemic form

Хронічна венозна недостатність Chronic venous insufficiency

113 / 200
Чоловік 58-ми років, протягом 20-ти років страждає на варикозне розширення поверхневих вен правої нижньої кінцівки. Рік тому у нього по медіальній поверхні в нижній третині правої гомілки утворилася трофічна виразка, яка після консервативного лікування загоїлась. Хворому запропоновано оперативне лікування - видалення поверхневих варикозних вен та перев'язка перфорантних. Яке інструментальне обстеження перед операцією буде найбільш доречним? A 58-year-old man has been suffering from varicose veins of the superficial veins of the right lower limb for 20 years. A year ago, he had a rash on the medial surface in the lower third of the right lower leg a trophic ulcer was formed, which healed after conservative treatment. The patient was offered surgical treatment - removal of superficial varicose veins and ligation of perforating veins. What instrumental examination before surgery would be most appropriate?

Флебоманометрія Phlebomanometry

Ультразвукове дуплексне сканування вен Ultrasound duplex scanning of veins

Плетизмографія Plethysmography

Осцилографія Oscillography

Реовазографія Rheovasography

114 / 200
Чоловіка 48-ми років доставлено до лікаря у непритомному стані. В анамнезі зловживання алкоголем. При обстеженні шкіра бліда, на тулубі 'судинні зірочки', печінковий запах з рота, язик обкладений коричневим нальотом. Підшкірні вени передньої стінки живота розширені у вигляді (caput medusae).При пальпації органів черевної порожнини позитивний симптом флуктуації, гепатомегалія, спленомегалія. Набряки нижніх кінцівок. Який стан у хворого є найбільш імовірним? A 48-year-old man was brought to the doctor in an unconscious state. He has a history of alcohol abuse. On examination, the skin is pale, 'vascular stars' on the body, liver odor , the tongue is covered with a brown coating. The subcutaneous veins of the anterior abdominal wall are dilated in the form of (caput medusae). When palpating the organs of the abdominal cavity, there is a positive symptom of fluctuation, hepatomegaly, splenomegaly. Swelling of the lower extremities. What condition is the most likely in the patient?

Перфоративна виразка шлунка Perforative gastric ulcer

Гіперглікемічна кома Hyperglycemic coma

Гостра печінкова недостатність Acute liver failure

Гіпоглікемічна кома Hypoglycemic coma

Госгра шлунково-кишкова кровотеча Gosgra gastrointestinal bleeding

115 / 200
Жінка 52-х років звернулася до лікаря зі скаргами на швидку втомлюваність та задишку протягом останнього року з періодичним сухим кашлем. Пацієнтка відзначає, що останнім часом їй стало важко піднімати обидві руки та розчісувати волосся. При фізикальному обстеженні температура тіла - 37,4°С пульс - 76/хв., артеріальний тиск - 130/85 мм рт.ст., SpO2-95% при кімнатному повітрі. При неврологічному дослідженні незначне зниження сили дельтоподібного м'яза. При аускультації легень - розповсюджені сухі хрипи. На щоках, навколо орбіт та на ліктях еритематозний висип. Який метод найбільш імовірно підтвердить діагноз у цієї пацієнтки? A 52-year-old woman came to the doctor with complaints of rapid fatigue and shortness of breath for the past year with a periodic dry cough. The patient notes that recently it has become difficult for her to lift both hands and comb hair. On physical examination, body temperature is 37.4°С, pulse is 76/min, blood pressure is 130/85 mm Hg, SpO2 is 95% on room air. On neurological examination, a slight decrease in strength of the deltoid muscle. On auscultation of the lungs - widespread dry rales. On the cheeks, around the orbits and on the elbows, an erythematous rash. Which method is most likely to confirm the diagnosis in this patient?

Рентгенографія ОГП OHP X-ray

Біопсія м'яза Muscle biopsy

Біопсія шкіри Skin biopsy

КТ легень CT lung

Виявлення антинуклеарних антитіл Detection of antinuclear antibodies

116 / 200
Здорова жінка 47-ми років прийшла до свого гінеколога на планове взяття мазку Папаніколау. Скарг не має. Викурює 20 сигарет на день протягом 30-ти років (індекс паління - 30 пачко/років). Рік тому самостійно зробила аналіз крові на визначення концентрації загального холестерину. ЛПНЩ, ЛПВЩ та тригліцеридів. Усі показники аналізу в межах норми. При фізикальному обстеженні температура - 37,1°С. пульс - 76/хв., частота дихання - 13/хв.. артеріальний тиск - 120/70 мм рт.ст. Аускультативних змін у легенях не виявлено. Лабораторні показники без відхилень. Який наступний крок є найбільш доречним? A 47-year-old healthy woman came to her gynecologist for a routine pap smear. She has no complaints. She has smoked 20 cigarettes a day for 30 years (smoking index - 30 packs/years). A year ago, she did a blood test on her own to determine the concentration of total cholesterol. LDL, HDL and triglycerides. All indicators of the analysis are within the normal range. During the physical examination, the temperature is 37.1°C. pulse - 76/min., respiratory rate - 13/min. arterial pressure - 120/70 mm Hg. Auscultatory changes in the lungs were not detected. Laboratory indicators without deviations. What next step is most appropriate?

Мамографія через *3 *роки Mammogram in *3 *years

Колпоскопія після ПАП-тесту Colposcopy after Pap test

Низькодозова КТ легень через 3 роки Low-dose lung CT after 3 years

Визначення загального холестерину та ЛПВЩ через 2 роки Determination of total cholesterol and HDL after 2 years

Колоноскопія через 1 рік Colonoscopy in 1 year

117 / 200
В гінекологічне відділення онкодиспансеру доставлено хвору 35-ти років зі скаргами на кров'янисті виділення після статевого акту. При дослідженні в дзеркалах: на шийці матки навколо зовнішнього вічка дефект епітелію. Матка, додатки і параметрії без патологічних змін. При гістологічному дослідженні біоптату шийки матки виявлено ракіп sitи. Яке лікування буде найбільш доречним? A 35-year-old patient was brought to the gynecological department of the oncology clinic with complaints of bleeding after sexual intercourse. When examined in mirrors: a defect is found on the cervix around the outer eye epithelium. Uterus, appendages, and parameters without pathological changes. A histological examination of a biopsy of the cervix revealed sarcomas. What treatment would be most appropriate?

Конусоподібна електроексцизія шийки матки Cone-shaped electroexcision of the cervix

Променева терапія Radiotherapy

Кріодеструкція шийки матки Cryodestruction of the cervix

Розширена екстирпація матки з додатками Advanced uterine extirpation with attachments

Хіміотерапевтичне лікування Chemotherapy treatment

118 / 200
Хворий 72-х років звернувся до лікаря зі скаргами на тремор рук. більше справа, загальну скутість, сповільнення ходи, мовлення, утруднення при виконанні звичайної домашньої роботи. При фізикальному обстеженні загальна брадикінезія, гіпомімія, емпростотонус, хода 'човгаюча', ахейрокінез, тремор спокою в руках, D>S, підвищений м'язовий тонус за пластичним типом, феномен 'зубчастого колеса' виражена носгуральна нестійкість. Які ліки, насамперед, мають бути у схемі лікування цього хворого? A 72-year-old patient turned to the doctor with complaints of hand tremors. More right, general stiffness, slowness of gait, speech, difficulty in performing usual housework. on physical examination, general bradykinesia, hypomimia, emprostotonus, shuffling gait, acheirokinesis, rest tremor in the hands, D>S, increased muscle tone of the plastic type, the phenomenon of the 'cogwheel', pronounced nasgural instability. What drugs, first of all, should be in the treatment scheme of this patient?

Холінолітики Cholinolytics

Антихолінестеразні препарати Anticholinesterase drugs

Препарати, що покращують мікроциркуляцію Drugs that improve microcirculation

Антидепресанти Antidepressants

Препарати, що містять леводопу Drugs containing levodopa

119 / 200
Жінка 40-ка років звернулася до лікаря зі скаргами на тупий ниючий біль в лівій поперековій ділянці, пітливість, зниження працездатності, субфебрильну температуру. У дитинстві перенесла туберкульоз шийних хребців. Лікувалася, знята з диспансерного обліку. При фізикальному обстеженні нирки не пальпуються, болючість в проекції лівої нирки. При лабораторному дослідженні в загальному аналізі сечі спостерігаються протеїнурія, піурія. Реакція сечі кисла. Яке дослідження найбільш імовірно дозволить уточнити діагноз цієї хворої? A 40-year-old woman consulted a doctor with complaints of dull, aching pain in the left lumbar region, sweating, reduced work capacity, low-grade fever. She suffered from tuberculosis of the cervical vertebrae as a child . She was treated, removed from the dispensary register. During the physical examination, the kidneys are not palpable, tenderness in the projection of the left kidney. During the laboratory examination, in the general analysis of the urine, proteinuria, pyuria are observed. The reaction of the urine is acidic. Which examination is most likely to clarify the diagnosis of this patient?

Цистографія Cystography

Проба Манту та повторний візит через 2 дні Mantoux test and return visit in 2 days

Ультразвукове дослідження нирок Ultrasound study of kidneys

Полімеразна ланцюгова реакція (ПЛР) сечі Polymerase chain reaction (PCR) of urine

Екскреторна урографія Excretory urography

120 / 200
Чоловік 45-ти років, доставлений у відділення невідкладної допомоги зі скаргами на раптовий інтенсивний біль у поперековій ділянці, часте болісне сечовиділення, блювоту. Температура тіла - 36,8°С, пульс - 82/хв„ артеріальний тиск - 130/80 мм рт.ст. При фізикальному обстеженні болісність при пальпації поперекової ділянки, позитивний симптом Пастернацького. При лабораторному дослідженні у загальному аналізі сечі білок - 0,06 г/л, реакція - слабко кисла, лейкоцити - 3-4 в полі зору, еритроцити - 30-40 в полі зору. Який діагноз є найбільш імовірним? A 45-year-old man was brought to the emergency department with complaints of sudden intense pain in the lumbar region, frequent painful urination, vomiting. Body temperature - 36.8 °С, pulse - 82/min, blood pressure - 130/80 mm Hg. During physical examination, pain during palpation of the lumbar region, positive Pasternacki's symptom. During laboratory research, in the general analysis of urine, protein - 0.06 g/l, reaction - weakly acidic, leukocytes - 3-4 in the field of vision, erythrocytes - 30-40 in the field of vision. What diagnosis is most likely?

Гостра ниркова недостатність Acute renal failure

Полікістоз нирок Polycystic kidney disease

Сечокам'яна хвороба Urolithiasis

Гострий пієлонефрит Acute pyelonephritis

Гострий гломерулонефрит Acute glomerulonephritis

121 / 200
Жінка 36-ти років звернулася до лікаря зі скаргами на пригнічений настрій, тривожність. дратівливість та нагрубання молочних залоз, що виникають кожного місяця. Найчастіше подібні симптоми виникають за 2 тижні до менструації та зникають після неї. Який з перерахованих діагнозів є найбільш імовірним? A 36-year-old woman consulted a doctor with complaints of depressed mood, anxiety, irritability, and engorgement of the mammary glands that occur every month. Most often, similar symptoms occur after 2 weeks before menstruation and disappear after it. Which of the following diagnoses is most likely?

Альгодисменорея Algodysmenorrhoea

Ранній патологічний клімакс Early pathological menopause

Преклімактеричний синдром Pre-menopausal syndrome

Вторинна психогенна аменорея Secondary psychogenic amenorrhea

Передменструальний синдром Premenstrual syndrome

122 / 200
Чоловік 38-ми років захворів 2 тижні тому, з'явився кашель, слабкість, підвищилася температура до 38,0°С. Стан різко погіршився до кінця 1-го тижня, коли з'явилися остуда, проливний піт, надвечір температура підвищилася до 39,0°С. За 2 дні до госпіталізації у хворого під час кашлю виділилася велика кількість смердючого харкотиння з кров'ю, після чого стан хворого покращився. Пульс - 80/хв., частота дихання - 20/хв., температура тіла - 37,6С'С. Які зміни можливі на рентгенограмі грудної порожнини? A 38-year-old man fell ill 2 weeks ago, cough, weakness appeared, the temperature rose to 38.0°C. The condition worsened sharply by the end of 1- th week, when chills, profuse sweat appeared, in the evening the temperature rose to 39.0° C. 2 days before hospitalization, the patient coughed up a large amount of foul-smelling sputum with blood, after which the patient's condition improved. 80/min., respiratory rate - 20/min., body temperature - 37.6C. What changes are possible on the X-ray of the chest cavity?

Зміщення середостіння в сторону гомогенної тіні Displacement of the mediastinum towards the homogeneous shadow

Затемнення частки легені Obscuration of a lobe of the lung

Наявність порожнини з горизонтальним рівнем рідини Presence of a cavity with a horizontal liquid level

Тінь в нижньому відділі з косою верхньою межею Shadow in the lower section with an oblique upper border

Однорідна округла тінь в легеневому полі Uniform round shadow in the lung field

123 / 200
Робітник очисної лави вугільної шахти зі стажем роботи,15 років звернувся до лікаря зі скаргами на біль у грудній клітці, задишку, сухий кашель, швидку втомлюваність. На рентгенограмі легень дифузний інтерстиціальний фіброз, емфізема, дрібновогнищеві утворення 3-5 мм у нижніх і середніх відділах. На спірограмі порушення функції зовнішнього дихання відсутні. За даними санітарно-гігієнічної характеристики робочої зони запиленість повітря кам'яним вугіллям становить 138 мг/м3, вміст силіцію (IV) оксиду 2% (норма - 10 мг/м3, якщо вугільний пил вміщує до 5% вільного діоксиду кремнію). Який діагноз є найбільш імовірним? A coal mine scrubbing worker with 15 years of experience turned to the doctor with complaints of chest pain, shortness of breath, dry cough, and rapid fatigue. On an X-ray of the lungs diffuse interstitial fibrosis, emphysema, small focal formations of 3-5 mm in the lower and middle parts. The spirogram shows no disturbances in the function of external breathing. According to the sanitary and hygienic characteristics of the working area, the dustiness of the air with coal is 138 mg/m3, the content of silicon (IV ) of 2% oxide (the norm is 10 mg/m3, if the coal dust contains up to 5% of free silicon dioxide). What diagnosis is most likely?

Коніотуберкульоз Coniotuberculosis

Антракоз Anthracosis

Саркоїдоз Sarcoidosis

Силікоз Silicosis

Аденокарцинома Adenocarcinoma

124 / 200
Жінка 39-ти років вперше звернулась до лікаря зі скаргами на значні менструації протягом 10-12 днів. Остання менструація почалась 3 тижні тому та триває дотепер. З анамнезу: цикл порушився 3 місяці тому, пологів - 2, абортів - 5. При дослідженні: шийка матки циліндричної форми, вічко зімкнуте. Матка та придатки без патологічних змін. Виділення кров'яні, надмірні. Яка тактика лікаря буде найбільш доречною? A 39-year-old woman first consulted a doctor with complaints of heavy menstruation for 10-12 days. The last menstruation began 3 weeks ago and continues until now. From the history: the cycle broke 3 months ago, 2 deliveries, 5 abortions. On examination: the cervix is cylindrical in shape, the eye is closed. The uterus and appendages are without pathological changes. The discharge is bloody, excessive. What tactics of the doctor will be most appropriate?

Надпіхвова ампутація матки Supravaginal amputation of the uterus

Гемостатична терапія Hemostatic therapy

Гормональна терапія Hormonal therapy

Вишкрібання порожнини матки Uterine scraping

Кріокоагуляція ендометрію Endometrial cryocoagulation

125 / 200
Жінка 37-ми років звернулася до лікаря зі скаргами на слабкість, швидку стомлюваність, запаморочення, поганий апетит, утруднене ковтання їжі. В анамнезі під час вагітності 5 років тому у хворої відзначалося зниження гемоглобіну до 86 г/л. При лабораторному дослідженні в загальному аналізі крові: еритроцити - 3,4 - 1012/л, Нb- 70 г/л, колірний показник - 0,6, ретикулоцити - 2%, лейкоцити - 4,7- 109/л, е- 2%, п- 3%, с- 64%, лімф,- 26%. мон,- 5%. швидкість осідання еритроцитів - 40 мм/год. Сироваткове залізо - 6,3 мкмоль/л. Який діагноз є найбільш імовірним? A 37-year-old woman consulted a doctor with complaints of weakness, rapid fatigue, dizziness, poor appetite, difficulty swallowing food. History during pregnancy 5 years ago the patient had a decrease in hemoglobin to 86 g/l. During the laboratory examination in the general blood test: erythrocytes - 3.4 - 1012/l, Hb - 70 g/l, color index - 0.6, reticulocytes - 2%, leukocytes - 4.7- 109/l, e- 2%, p- 3%, c- 64%, lymph,- 26%, mon,- 5%, erythrocyte sedimentation rate - 40 mm/h. Serum iron - 6.3 μmol/l. What diagnosis is most likely?

Фолієводефіцитна анемія Folate deficiency anemia

Аутоімунна гемолітична анемія Autoimmune hemolytic anemia

Анемія Мінковського-Шоффара Minkowski-Shofar Anemia

Гіпопластична анемія Hypoplastic anemia

Залізодефіцитна анемія Iron deficiency anemia

126 / 200
Хлопчик 12-ти років надійшов до клініки зі скаргами на відчуття серцебиття, перебої та біль у серці, задишку. Ці симптоми найчастіше виникали після фізичних навантажень і закінчувались синкопальними станами. Згодом лікарем було встановлено діагноз гіпертрофічної обструктивної кардіоміопатії. Який препарат* ПРОТИПОКАЗАНИЙ при лікуванні цієї патології? A 12-year-old boy came to the clinic with complaints of palpitations, interruptions and pain in the heart, shortness of breath. These symptoms most often occurred after physical exertion and ended with syncopal states Subsequently, the doctor diagnosed hypertrophic obstructive cardiomyopathy. Which drug* is CONTRAINDICATED in the treatment of this pathology?

Дигоксин Digoxin

Еналаприл Enalapril

Верапаміл Verapamil

Бісопролол Bisoprolol

127 / 200
Чоловік 65-ти років надійшов до стаціонару з приводу хірургічного лікування гострого некротичного панкреатиту. Оперативне втручання пройшло без ускладнень. На 5-й день після операції стан погіршився. При фізикальному обстеженні температура тіла підвищена до 38,5°C, артеріальний тиск - 110/65 мм рт.ст., пульс - 118/хв., частота дихання - 28/хв., SpO2 - 92%. При аускультації легень звучні хрипи у правій нижній ділянці легень. Післяопераційна рана загоюється добре, живіт при пальпації безболісний, ненапружений. Який діагноз є найбільш імовірним? A 65-year-old man was hospitalized for surgical treatment of acute necrotizing pancreatitis. The operation went without complications. On the 5th day after the operation, the condition worsened. during the physical examination, the body temperature increased to 38.5°C, blood pressure - 110/65 mm Hg, pulse - 118/min, respiratory rate - 28/min, SpO2 - 92%. Upon auscultation of the lungs, audible rales in in the right lower part of the lungs. The postoperative wound is healing well, the abdomen is painless on palpation, not tense. What is the most likely diagnosis?

Сепсиc Sepsis

Злоякісна гіпертермія Malignant hyperthermia

Медикаментозна лихоманка Medicine fever

Синдром системної запальної відповіді неінфекційної природи Systemic inflammatory response syndrome of a non-infectious nature

128 / 200
Дівчинка вагою 3100 г народилася від третьої вагітності в домашніх умовах. На 3-й день мати помітила дрібні крововиливи у пупковій ранці, а також 'червоні цятки' на склерах очей. Мати на обліку по вагітності у допологовому періоді не перебувала. Вигодовує дитину грудним молоком за потребою. При фізикальному обстеженні субкон'юнктивальні геморагії. При лабораторному дослідженні концентрація гемоглобіну - 155 г/л, група крові у матері В (III) Rh-позитивна,у дитини - А (II) Rh- негативна. Яка тактика з метою попередження прогресування цього стану у дитини буде найбільш доречною? A girl weighing 3100 g was born from the third pregnancy at home. On the 3rd day, the mother noticed small hemorrhages in the umbilical wound, as well as 'red spots' on the sclera eyes. The mother was not registered for pregnancy in the prenatal period. She feeds the child with breast milk as needed. During the physical examination, subconjunctival hemorrhages. During the laboratory examination, the hemoglobin concentration is 155 g/l, the mother's blood group is B (III) Rh-positive 'the child has A (II) Rh- negative. What tactic to prevent the progression of this condition in the child would be the most appropriate?

Ввести вітамін К Enter vitamin K

Переливання еритроцитарної маси Red blood cell transfusion

Введення імуноглобуліну антирезус Rh0 (D) людини Introduction of antirhesus Rh0 (D) human immunoglobulin

Перехід на штучне вигодовування залізовмісними сумішами Transition to artificial feeding with iron-containing mixtures

Призначити пероральний прийом препаратів заліза Prescribe oral iron preparations

129 / 200
Жінка 32-х років, вчителька молодших класів, проходить обов'язковий профілактичний медичний огляд. Відомо, що у регіоні проживання спалах кору. Жінка скарг не має. За даними карти профілактичних щеплень порушень календаря немає, у дитинстві отримала усі необхідні щеплення. Останнє щеплення від дифтерії та правця у 26 років. Останній ПАП-тест зроблено у 30 років, без відхилень. При фізикальному обстеженні Температура тіла - 37,2°С, пульс - 80/хв.. артеріальний тиск - 130/80 мм рт.ст. Що з перерахованого є найбільш доречною тактикою лікаря відносно цієї пацієнтки? A 32-year-old woman, a primary school teacher, undergoes a mandatory preventive medical examination. It is known that there is an outbreak of measles in the region of residence. The woman has no complaints. according to the preventive vaccination card, there are no violations of the calendar, she received all the necessary vaccinations in childhood. The last diphtheria and tetanus vaccination was at the age of 26. The last Pap test was done at the age of 30, without any abnormalities. During the physical examination, the body temperature is 37.2°C, pulse - 80/min.. blood pressure - 130/80 mm Hg. Which of the following is the most appropriate tactic of the doctor regarding this patient?

Зробити тест на вірус папіломи людини (ВПЛ-тест) Do a human papillomavirus test (HPV test)

Визначити рівень IgG до вірусу кору Determine the level of IgG to the measles virus

Направити на мамографію Send for mammography

Вакцинації та скринінгу не потребує Does not require vaccination or screening

Ввести вакцину АДС-М Enter ADS-M vaccine

130 / 200
Чоловіка 45-ти років доставлено до лікаря після повідомлення про випадок активного туберкульозу у його колеги з яким він працює в одному офісному приміщенні. Скарг не має. Після проведення туберкулінової проби розмір папули становить 14 мм. Даних прорезультат попередньої проби немає. Який з наступних кроків лікаря є найбільш доречним? A 45-year-old man was taken to the doctor after reporting a case of active tuberculosis in his colleague with whom he works in the same office space. He has no complaints. After performing a tuberculin test, the size of the papule is 14 mm. There are no data on the results of the previous test. Which of the following steps is the most appropriate for the doctor?

Призначення ізоніазиду, рифампіцину та етамбутолу Prescription of isoniazid, rifampicin and ethambutol

Забір мокротиння для виявлення мікобактерій туберкульозу Sputum sampling to detect tuberculosis mycobacteria

Повторна туберкулінова проба через З місяці Repeated tuberculin test after 3 months

Карантин на 3 місяці Quarantine for 3 months

Рентгенографія органів грудної клітки X-ray of chest organs

131 / 200
Хвора 37-ми років, хімік-технолог. Турбує поява синців на животі, стегнах, гомілках після невеликих травм або без причини. Під час огляду виявлені численні дрібні та дещо більші синці, які місцями зливаються (від 1x1 до 4x5 см). Інших змін немає. Менструація триває до 7 днів. Селезінка не пальпується. В крові: Нb- 92г/л, лейкоцити - 7.2 • 109/л, тромбоцити - 6 • 109/л. швидкість осідання еритроцитів - 33 мм/год. Який патогенез даного захворювання? A 37-year-old patient, a chemist-technologist. She is concerned about the appearance of bruises on the abdomen, thighs, lower legs after minor injuries or for no reason. During the examination, numerous small and slightly larger bruises that merge in places (from 1x1 to 4x5 cm). There are no other changes. Menstruation lasts up to 7 days. The spleen is not palpable. In the blood: Hb - 92g/l, leukocytes - 7.2 • 109/l, platelets - 6 • 109/l. erythrocyte sedimentation rate - 33 mm/h. What is the pathogenesis of this disease?

Зниження рівня фібриногену Decrease in fibrinogen

Зниження рівня антигемофільного глобуліну Decrease in antihemophilic globulin

Скорочення тривалості життя тромбоцитів Decreased life expectancy of platelets

Підвищення фібринолітичної активності Increased fibrinolytic activity

Зниження рівня протромбіну крові Decrease in blood prothrombin

132 / 200
У хворої 33-х років після нападу епілептичних судом виникли задуха, сухий кашель та підвищення температури тіла. При аускультації легень справа знизу дихання відсутнє, перкуторно - притуплення звуку. Огляд ротової порожнини виявив відсутність одного різця нижньої щелепи. Який діагностичний метод найбільш імовірно дозволить лікарю визначити причину дихальних розладів пацієнтки? A 33-year-old patient developed shortness of breath, a dry cough, and an increase in body temperature after an attack of epileptic seizures. On auscultation of the lungs from the lower right, there is no breathing, and on percussion - dulling of the sound. Examination of the oral cavity revealed the absence of one incisor of the lower jaw. Which diagnostic method is most likely to allow the doctor to determine the cause of the patient's respiratory disorders?

Непряма ларингоскопія Indirect laryngoscopy

Бронхоскопія Bronchoscopy

Фіброезофагогастроскопія Fibroesophagogastroscopy

Пряма ларингоскопія Direct laryngoscopy

133 / 200
Раніше здорова жінка 47-ми років звернулася до сімейного лікаря через підвищену температуру та кашель із жовтим мокротинням, що тривають вже 5 днів. Температура тіла - 37,4°С , пульс - 82/хв..частота дихання - 15/хв.. артеріальний тиск - 130/80 мм рт.ст. При аускультації вислуховуються двосторонні розсіяні сухі хрипи. На оглядовій рентгенограмі грудної клітки патологічних змін не виявлено. Яка тактика лікування буде найбільш доречною для цієї пацієнтки? A previously healthy 47-year-old woman consulted a family doctor because of a high temperature and a cough with yellow sputum that has been going on for 5 days. Body temperature is 37.4° C, pulse - 82/min.. respiratory rate - 15/min.. blood pressure - 130/80 mm Hg. On auscultation, bilateral diffuse dry rales are heard. No pathological changes were detected on the X-ray examination of the chest. What treatment tactics will be used most appropriate for this patient?

Аскорбінова кислота (вітамін С) перорально на 5 днів Ascorbic acid (vitamin C) orally for 5 days

Комбінований препарат із противірусним та антигістамінним компонентом перорально на 5 днів Combined drug with antiviral and antihistamine component orally for 5 days

Інтерферону альфа-2b у сиропі протягом тижня Interferon alfa-2b in syrup for a week

Вживання достатньої кількості рідини Drinking enough fluids

Ципрофлоксацин перорально на 7 днів Ciprofloxacin orally for 7 days

134 / 200
Чоловік 25-ти років звернувся до лікаря зі скаргами на наявність висипки протягом 2 тижнів, яка з'явилась несподівано. Під час збору анамнезу пацієнт повідомив, що почуває себе повністю здоровим, але зазначив що 2 місяці тому влітку мав 'жахливу, грипоподібну' застуду. При фізикальному обстеженні на шкірі лобка, геніталій та стегон розташовані сферичні папули, рожевого та тілесного кольору, без запалення, з центральним заглибленням. Який діагноз є найбільш імовірним? A 25-year-old man came to the doctor complaining of a rash for 2 weeks that appeared unexpectedly. During history taking, the patient reported that he felt completely healthy, but noted that he had a 'horrible, flu-like' cold 2 months ago in the summer. On physical examination, there are spherical papules, pink and flesh-colored, non-inflammatory, with a central indentation, on the skin of the pubis, genitals, and thighs. What is the most likely diagnosis?

Фурункул Furnish

Контагіозний молюск Mollus contagion

Папілома Papilloma

Оперізуючий герпес Herpes zoster

Вторинний сифіліс Secondary syphilis

135 / 200
Хвора 38-ми років, бухгалтер. На прийомі багато говорить, мова прискорена, швидко переключається з теми на тему, що робить розуміння мови хворої скрутним. Не може всидіти на стільці, жестикулює, несподівано починає співати, сміятися, декламувати вірші. Вважає себе 'найздоровішою',' 'найщасливішою' заявляє, що 'всі заздрять її зовнішності та чудовому голосу'. Зі слів родичів, останні 5 днів не спить вночі, відзначається підвищений апетит, гнівливість. З роки тому лікувалася у психіатра з приводу депресії. Який синдромальний діагноз є найбільш імовірним? The patient is 38 years old, an accountant. At the appointment, she talks a lot, her speech is accelerated, she quickly switches from topic to topic, which makes it difficult to understand the patient's language. She cannot sit still on a chair, gesturing, suddenly starts singing, laughing, reciting poems. She considers herself the 'healthiest,' the 'happiest,' and declares that 'everyone envies her appearance and wonderful voice.' appetite, anger. Years ago, she was treated by a psychiatrist for depression. What syndromic diagnosis is the most likely?

Депресивний синдром Depressive syndrome

Маніакальний синдром Manic syndrome

Шизофренія, параноїдна форма Schizophrenia, paranoid form

Деліріозний синдром Delirious syndrome

Синдром гіперактивності з дефіцитом уваги Attention deficit hyperactivity disorder

136 / 200
60-річна жінка доставлена у відділення невідкладної допомоги зі скаргами на раптовий інтенсивний головний біль та нудоту. Останні нівроку спостерігалася легка диплопія. Протягом багатьох років хворіє на артеріальну гіпертензію та цукровий діабет II типу. Артеріальний тиск - 160/90 мм рт.ст., частота серцевих скорочень - 82/хв. Фізикальне обстеження виявило правосторонній птоз, легку анізокорію та ригідність потиличних м'язів. Атаксія не спостерігається. Який діагноз є найбільш імовірним? A 60-year-old woman was brought to the emergency department with complaints of sudden intense headache and nausea. In the last year, mild diplopia was observed. She has been suffering from arterial hypertension for many years and diabetes mellitus type II. Blood pressure 160/90 mmHg, heart rate 82/min. Physical examination revealed right-sided ptosis, mild anisocoria, and rigidity of the occipital muscles. No ataxia. What is the most likely diagnosis?

Метаболічна енцефалопатія Metabolic encephalopathy

Гліома стовбура головного мозку Brain stem glioma

Лакунарний інсульт Lacunar stroke

Субарахноїдальний крововилив Subarachnoid hemorrhage

Невралгія трійчастого нерва Trigeminal neuralgia

137 / 200
Чоловік 34-х років звернувся до лікаря зі скаргами на ранкову скутість, болі в поперековій ділянці, плечових суглобах, суглобах шиї, хребта, які посилюються при різких рухах. Захворювання почалося з болів у поперековій ділянці після переохолодження. При фізикальному обстеженні болючість при пальпації остистих відростків та здухвиннокрижового сполучення, сегментарна ригідність поперекової ділянки хребта, атрофія прилеглих м'язів. На рентгенограмі поперекового відділу хребта осифікація зв'язок і сухожиль, синдесмофіти, повне зрощення хребців. При лабораторному дослідженні в крові лейкоцити - 4,3 • 109/л, ШЗЕ- 37 мм/год. Який з наведених діагнозів є найбільш імовірним? A 34-year-old man consulted a doctor with complaints of morning stiffness, pain in the lumbar region, shoulder joints, neck joints, and spine, which are aggravated by sudden movements. The disease began with pain in the lumbar region after hypothermia. During the physical examination, pain during palpation of the spinous processes and the sacroiliac joint, segmental rigidity of the lumbar region of the spine, atrophy of the adjacent muscles. On the X-ray of the lumbar spine, ossification of ligaments and tendons, syndesmophytes, complete fusion vertebrae. During a laboratory study, leukocytes in the blood - 4.3 • 109/l, SZE - 37 mm/h. Which of the following diagnoses is the most probable?

Ревматоїдний артрит Rheumatoid arthritis

Анкілозуючий спондилоартрит Ankylosing spondylitis

Реактивний артрит Reactive arthritis

Остеоартроз Osteoarthrosis

Псоріатичний артрит Psoriatic arthritis

138 / 200
В організм людини з атмосферного повітря надходять декілька хімічних речовин. Як називається тип комбінованої дії. при якому сумісний її ефект перевищує суму ефектів кожної речовини, що входить у комбінацію, при їх ізольованій дії на організм? Several chemical substances enter the human body from atmospheric air. What is the type of combined action called, in which its combined effect exceeds the sum of the effects of each substance included in the combination, with their isolated effect on the body?

Потенціювання Potentiation

Ізольована дія Isolated Action

Поєднана дія Combined Action

Комплексна дія Complex Action

Антагонізм Antagonism

139 / 200
35-річний чоловік звернувся до лікаря зі скаргами на сильний біль у правому оці, світлобоязнь, сльозотечу, погіршення зору. Скарги з'явились після роботи зі зварювальним апаратом. Контактні лінзи не використовує. Під час огляду: очна щілина звужена, помірна перикорнеальна ін'єкція, гострота зору збережена. Після інстиляції розчину анестетика, стан полегшився, що дало можливість провести огляд у щілинній лампі та виявити дефект рогівки розміром 3 мм, що зафарбувався флюоресцеїном. Яке лікування буде найбільш доречним? A 35-year-old man consulted a doctor with complaints of severe pain in the right eye, photophobia, lacrimation, impaired vision. The complaints appeared after working with a welding machine. He does not use contact lenses. During the examination: the eye slit is narrowed, a moderate pericorneal injection, visual acuity is preserved. After instillation of the anesthetic solution, the condition improved, which made it possible to conduct a slit-lamp examination and reveal a 3 mm corneal defect stained with fluorescein What treatment would be most appropriate?

Інстиляція дексаметазону Dexamethasone instillation

Сухе тепло Dry heat

Інстиляція таурину Taurine instillation

Інстиляція сульфацилу натрію Sulfacyl sodium instillation

Інстиляція офлоксацину Ofloxacin instillation

140 / 200
При проведенні медичного огляду учнів середнього та старшого шкільного віку лікарі визначали відповідність біологічного розвитку та календарного віку за наступними критеріями: щорічне збільшення довжини тіла, осифікація кісток кисті, кількість постійних зубів. Який додатковий показник розвитку у ці вікові періоди найбільш імовірно мають включити лікарі? When conducting a medical examination of middle and high school students, doctors determined the correspondence of biological development and calendar age according to the following criteria: annual increase in body length, ossification of hand bones, number of permanent teeth. What additional developmental indicator should doctors most likely include in these age periods?

Розвиток вторинних статевих ознак Development of secondary sexual characteristics

М'язова сила кисті Hand muscle strength

Життєва ємність легень Vital lung capacity

Обвід грудної клітки Chest Circumference

Маса тіла Body weight

141 / 200
Внаслідок дорожньо-транспортної пригоди потерпілий отримав множинні переломи кінцівок та кісток тазу. В анамнезі: гемофілія А. Під час обстеження формуються гематоми на ушкоджених ділянках. Стан погіршується. Артеріальний тиск - 90/50 мм рт.ст. Вкажіть найбільш доцільну комбінацію інфузійних препаратів для лікування пацієнта після застосування поліглюкіну та сольових розчинів: As a result of a traffic accident, the victim received multiple fractures of the limbs and pelvic bones. History: hemophilia A. During the examination, hematomas form on the damaged areas. The condition worsens. Arterial pressure - 90/50 mm Hg Specify the most appropriate combination of infusion drugs for the treatment of the patient after the use of polyglukin and saline solutions:

Еритроцитарна маса, свіжозаморожена плазма Erythrocyte mass, fresh frozen plasma

Свіжозаморожена плазма, альбумін Fresh-frozen plasma, albumin

Кріопреципітат, еритроцитарна маса Cryoprecipitate, erythrocyte mass

Кріопреципітат, глюкоза Cryoprecipitate, glucose

Еритроцитарна маса Erythrocyte mass

142 / 200
Чоловік 63-х років був госпіталізований 5 днів тому з приводу інфаркту міокарда. Раптом він поскаржився на сильний біль у грудях та втратив свідомість. Пульс не визначається, тони серця відсутні. ЕКГ: синусовий ритм, QS та підйом сегмента SТ в V1-V4. Реанімаційні заходи неефективні. При пункції виявлена кров в порожнині перикарда. Вкажіть найбільш імовірне ускладнення: A 63-year-old man was hospitalized 5 days ago due to a myocardial infarction. He suddenly complained of severe chest pain and lost consciousness. The pulse is not detectable, heart sounds absent. ECG: sinus rhythm, QS and elevation of the ST segment in V1-V4. Resuscitation measures are ineffective. Blood was detected in the pericardial cavity during puncture. Specify the most likely complication:

Порушення цілісності стінки лівого шлуночка Violation of the integrity of the wall of the left ventricle

Тромбоемболія легеневої артерії Thromboembolism of the pulmonary artery

Гострий перикардит Acute pericarditis

Гостра мітральна недостатність Acute mitral regurgitation

Порушення цілісності міжшлуночкової перегородки Disruption of the integrity of the interventricular septum

143 / 200
Жінка 49-ти років звернулася до лікаря зі скаргами на головний біль, припливи жару до голови, шиї, підвищену пітливість, серцебиття, підвищення артеріального тиску до 170/100 мм рт.ст., дратівливість, безсоння, плаксивість, послаблення пам'яті, рідкі мізерні менструації, збільшення маси тіла на 5 кг протягом останніх півроку. Який діагноз є найбільш імовірним? A 49-year-old woman consulted a doctor with complaints of headache, hot flushes to the head, neck, increased sweating, palpitations, blood pressure increase to 170/100 mm Hg, irritability, insomnia, tearfulness, memory loss, infrequent scanty menstruation, weight gain of 5 kg during the last six months. What diagnosis is most likely?

Соматоформний розлад Somatoform disorder

Передменструальний синдром Premenstrual syndrome

Клімактеричний синдром Climacteric syndrome

Артеріальна гіпертензія Hypertension

Посткастраційний синдром Post-castration syndrome

144 / 200
У лікарню швидкої допомоги поступив хворий, що скаржиться на нудоту, багаторазове блювання, пронос, розлади зору - поява сітки перед очима, нечітке бачення ближніх предметів. З анамнезу хворого з'ясувалося, що напередодні ввечері він споживав м'ясну консерву домашнього приготування зі смаженою картоплею. Перші симптоми з'явились вночі. Який діагноз найбільш імовірний? A patient was admitted to the emergency hospital complaining of nausea, repeated vomiting, diarrhea, visual disturbances - the appearance of a net in front of the eyes, blurred vision of nearby objects. From the patient's history it turned out that he had consumed home-made canned meat with fried potatoes the night before. The first symptoms appeared at night. What is the most likely diagnosis?

Шигельоз Shigelosis

Стафілококовий токсикоз Staphylococcal toxicosis

Сальмонельоз Salmonellosis

Ботулізм Botulism

Дизентерія Dysentery

145 / 200
Восьмирічна дитина скаржиться на тривалий вологий кашель з виділенням великої кількості харкотиння гнійного характеру з неприємним запахом, інколи з домішкою крові. Загальний стан тяжкий, температура тіла -38,7°С, шкіра бліда, периоральний ціаноз, пальці у вигляді 'барабанних паличок'. Над легенями: ослаблене дихання. різнокаліберні вологі хрипи. Рентгенологічно: ателектатичні ділянки. Яке з обстежень для уточнення діагнозу буде найбільш доцільним? An eight-year-old child complains of a long wet cough with the release of a large amount of sputum of a purulent nature with an unpleasant smell, sometimes with an admixture of blood. The general condition is serious, the body temperature is -38.7 °С, the skin is pale, perioral cyanosis, fingers in the form of 'drumsticks'. Above the lungs: weakened breathing. wet rales of various calibers. X-ray: atelectatic areas. Which of the examinations to clarify the diagnosis would be the most appropriate?

Комп'ютерна томографія з високою розподільною здатністю Computed tomography with high resolution

Бронхоскопія Bronchoscopy

Рентгенографія ОГК Roentgenography of OGK

Ехокардіографія Echocardiography

146 / 200
Чоловік 78-ми років, правильно називає своє прізвище, ім'я, дату народження. Не може назвати поточну дату, свій вік, дезорієнтований у місці перебування. Не пам'ятає про смерть дружини, яка померла 5 років тому, а також нічого не може повідомити про останні роки свого життя. Мова уповільнена, словниковий запас обмежений, часто не може згадати назви предметів. У пробі на запам'ятовування десяти слів відтворює три слова. Прості арифметичні дії виконує з помилками. Не може пояснити сенсу поширених прислів'їв і приказок. Який синдромальний діагноз є найбільш імовірним? A 78-year-old man correctly names his surname, first name, date of birth. He cannot name the current date, his age, disorientated in his location. No remembers the death of his wife, who died 5 years ago, and cannot report anything about the last years of his life.Speech is delayed, vocabulary is limited, often cannot recall the names of objects.Reproduces three words on a ten-word recall test . Performs simple arithmetic operations with errors. Cannot explain the meaning of common proverbs and sayings. What syndromic diagnosis is the most probable?

Деменція Dementia

Маячний синдром Delirious Syndrome

Маніакальний синдром Manic syndrome

Галюцинаторний синдром Hallucinatory syndrome

Депресивний синдром Depressive syndrome

147 / 200
Хворий 29-ти років на другий день після приїзду з Індії госпіталізований у клініку зі скаргами на біль у животі, рідкі випорожнення зі склоподібним слизом та кров'ю у вигляді 'малинового желе'. Загальний стан задовільний, шкіра бліда, висипу немає. Живіт м'який, чутливий в ділянці сліпої і висхідної кишок. Який етіотропний препарат для лікування цієї патології буде найбільш доречно застосувати? On the second day after arriving from India, a 29-year-old patient was hospitalized in the clinic with complaints of abdominal pain, loose stools with glassy mucus and blood in the form 'raspberry jelly'. The general condition is satisfactory, the skin is pale, there is no rash. The abdomen is soft, sensitive in the area of the cecum and ascending colon. What etiotropic drug would be most appropriate to use for the treatment of this pathology?

Метронідазол Metronidazole

Ніфуроксазид Nifuroxazide

Дексазон Dexazone

Цефтриаксон Ceftriaxone

Еритроміцин Erythromycin

148 / 200
Дівчина 21-го року звернулася до лікаря з грипоподібною картиною захворювання, субфебрильною температурою, нездужанням та легкою жовтяницею протягом 2-х днів. Лабораторні показники сироватки крові: аспартатамінотрансфераза (ACT) - 456 Од/л, аланінамінотрансфераза (АЛТ) - 745 Од/л, IgM до HAV - позитивний результат. Яка тактика по відношенню до контактних осіб буде найбільш доречною? A 21-year-old girl consulted a doctor with a flu-like illness, low-grade fever, malaise, and mild jaundice for 2 days. Serum laboratory parameters: aspartate aminotransferase ( ACT) - 456 U/l, alanine aminotransferase (ALT) - 745 U/l, IgM to HAV - a positive result. Which tactics in relation to contact persons will be the most appropriate?

Профілактичний прийом софосбувіру/ледіпасвіру протягом 1 місяця Prophylactic sofosbuvir/ledipasvir for 1 month

Введення інтерферону альфа-2b протягом першого тижня Introduction of interferon alfa-2b during the first week

Вакцинація від гепатиту В протягом першого тижня Hepatitis B vaccination during the first week

Вакцинація від гепатиту А протягом першого тижня Vaccination against hepatitis A during the first week

Введення імуноглобуліну людини нормального протягом 24 годин Injection of normal human immunoglobulin within 24 hours

149 / 200
Дівчинку 7-ми років доставлено до відділення невідкладної допомоги у важкому стані. Зі слів матері дитина почала скаржитися на біль у спині та шиї, після чого згодом їй було дуже важко вимовляти слова та випити склянку води. Об'єктивно: спастичність жувальних м'язів. Під час збору анамнезу лікар дізнався, що тиждень тому дитина гуляла надворі з друзями та уколола палець дерев'яною палицею, після якої у шкірі залишилася скіпка. Матір про інцидент завчасно повідомлена не була і видалила стороннє тіло декілька днів тому, коли випадково його помітила. Дівчинка не отримала жодного щеплення через особисті переконання матері. Скільки доз входить у первинний вакцинальний комплекс від збудника, що найбільш імовірно викликав захворювання у дівчинки? A 7-year-old girl was brought to the emergency department in critical condition. According to the mother, the child began to complain of back and neck pain, after which she was very it is difficult to pronounce words and drink a glass of water. Objectively: spasticity of the masticatory muscles. During the history taking, the doctor learned that a week ago the child was walking outside with friends and pricked his finger with a wooden stick, after which a lump remained in the skin. The mother about the incident was not reported in advance and removed the foreign body a few days ago, when she accidentally noticed it. The girl did not receive any vaccination due to the mother's personal beliefs. How many doses are included in the primary vaccine complex against the pathogen that most likely caused the girl's disease?

Три Three

П'ять Five

Дві Two

Шість Six

Чотири Four

150 / 200
Мати дитини 1-го року скаржиться на постійний нав'язливий, частий, малопродуктивний кашель, іноді до блювання. Під час об'єктивного дослідження у пацієнта спостерігається прискорене дихання, помірне втягнення нижніх міжреберних м'язів, збільшення передньозаднього розміру грудної клітки. Під час респіраторних інфекцій з'являється бронхообструкція. У періоді новонародженості переніс меконіальний ілеус. Оберіть першочергове обстеження: The mother of a 1-year-old child complains of constant intrusive, frequent, low-productive cough, sometimes leading to vomiting. During an objective examination, the patient has rapid breathing , moderate retraction of the lower intercostal muscles, an increase in the anteroposterior size of the chest. Bronchoobstruction appears during respiratory infections. Meconium ileus was suffered as a newborn. Choose the primary examination:

Дослідження хлоридів поту Sweat Chloride Study

КТ легень CT lung

Генетичне тестування Genetic testing

Рентгенографія органів грудної клітки X-ray of chest organs

Дослідження на хламідії та мікоплазму Research on chlamydia and mycoplasma

151 / 200
64-річний чоловік прийшов до лікаря на щорічний профілактичний огляд. Скарг не має. Хворіє на артеріальну гіпертензію, ХОЗЛ та глаукому. Викурює одну пачку сигарет на день протягом 30-ти років (індекс паління - 30 пачко/років). При фізикальному обстеженні: температура - 37,2°С, артеріальний тиск - 140/70 мм рт.ст, пульс - 79/хв., частота дихання - 16/хв., пряма, співдружня реакція зіниць на світло. При аускультації серця акцент II тону над аортою, аускультація легень без патологічних змін, грудна клітка 'бочкоподібна'. Який з перерахованих методів скринінгу є найбільш доречним для цього пацієнта? A 64-year-old man came to the doctor for an annual preventive examination. He has no complaints. He suffers from arterial hypertension, COPD and glaucoma. He smokes one pack of cigarettes a day for 30 years (smoking index - 30 pack/years). During physical examination: temperature - 37.2°С, blood pressure - 140/70 mm Hg, pulse - 79/min., respiratory rate - 16/min. , a direct, cooperative reaction of the pupils to light. During auscultation of the heart, the accent of the II tone over the aorta, auscultation of the lungs without pathological changes, the chest is 'barrel-shaped'. Which of the listed screening methods is the most appropriate for this patient?

Низькодозова КТ легень Low-dose lung CT

Легеневі функціональні тести Pulmonary function tests

Рентгенографія ОГК Roentgenography of OGK

Бронхоальвеолярний лаваж Bronchoalveolar lavage

МРТ легень Lung MRI

152 / 200
Пацієнтка 30-ти років впродовж 5-ти років хворіє на системний червоний вовчак (СЧВ) і щоденно приймає 16 мг метил преднізолону. Яке твердження щодо вакцинації у даному випадку є вірним? A 30-year-old female patient has been suffering from systemic lupus erythematosus (SLE) for 5 years and has been taking 16 mg of methylprednisolone daily. What is the statement about vaccination in this case is true?

Вакцинація від грипу після зменшення дози преднізолону до мінімальної Influenza vaccination after reducing the prednisone dose to the minimum

Будь-яка вакцинація протипоказана Any vaccination is contraindicated

Вакцинація від грипу протипоказана хворим, що отримують глюкокортикоїди Influenza vaccination is contraindicated in patients receiving glucocorticoids

Вакцинація від грипу проводиться під прикриттям антигістамінних препаратів Flu vaccination is carried out under the guise of antihistamines

Рекомендована щорічна планова вакцинація від грипу Recommended annual scheduled flu vaccination

153 / 200
У породіллі через 4 тижні після термінових пологів підвищилась температура тіла до 39°С,з'явилися слабкість та біль у правій молочній залозі, озноб. Молочна залоза нагрубла, збільшена, чутлива при пальпації. Флуктуації у ділянці інфільтрату немає. В аналізі крові помірний лейкоцитоз. Який діагноз є найбільш імовірним? 4 weeks after the emergency delivery, the mother's body temperature rose to 39°C, weakness and pain in the right mammary gland appeared, chills. The mammary gland thickened, enlarged, sensitive to palpation. There is no fluctuation in the area of the infiltrate. In the blood analysis, moderate leukocytosis. What diagnosis is most likely?

Лактостаз Lactostasis

Серозний мастит Serous mastitis

Мастопатія Mastopathy

Абсцедуючий мастит Abscessing mastitis

Гангренозний мастит Gangrenous mastitis

154 / 200
Мати привела на прийом до лікаря хлопчика 5-ти років зі скаргами на висип обличчя, що з'явився 5 днів тому. Об'єктивно: на обличчі помітні декілька пухирів розмірами 0,5-1,5 см в діаметрі, з тонкими в'ялими кришками, виповнені серозним каламутним ексудатом і розташовані на гіперемованій, набряклій основі. Поряд з цими вогнищами ураження спостерігаються ерозії з виділенням серозно-гнійної рідини та товсті медово-жовті кірки. Поставте клінічний діагноз: The mother brought a 5-year-old boy to the doctor with complaints of a facial rash that appeared 5 days ago. Objectively: several visible blisters 0.5-1.5 cm in diameter, with thin flaccid lids, filled with serous cloudy exudate and located on a hyperemic, swollen base. Along with these lesions, there are erosions with the release of serous-purulent fluid and thick honey-yellow crusts. Make a clinical diagnosis:

Дитяча екзема Children's eczema

Кандидоз Candida

Контактний дерматит Contact dermatitis

Простий пухирцевий лишай Lichen simplex

Вульгарне імпетиго Vulgar impetigo

155 / 200
Жінка 54-х років звернулася до лікаря зі скаргами на загальну слабкість протягом 2-х місяців, шум у голові, осиплість голосу. У 34 роки перенесла субтотальну резекцію шлунка з приводу виразкової хвороби. Фізикальне обстеження без особливостей. У загальному аналізі крові: Нb- 58 г/л, еритроцити -1,2-1012/л. лейкоцити -2,8-109/л. тромбоцити -140 • 109/л, швидкість осідання еритроцитів - 17 мм/год. анізоцитоз, пойкілоцитоз - виражені (++). Який з наступних діагностичних методів буде найбільш доречно призначити пацієнтці? A 54-year-old woman consulted a doctor with complaints of general weakness for 2 months, noise in the head, hoarseness of voice. At the age of 34, she underwent a subtotal gastrectomy due to peptic ulcer disease. Physical examination without features. In the general blood test: Hb - 58 g/l, erythrocytes -1.2-1012/l, leukocytes -2.8-109/l, platelets -140 • 109/l, erythrocyte sedimentation rate - 17 mm/h. anisocytosis, poikilocytosis - pronounced (++). Which of the following diagnostic methods would be the most appropriate to prescribe to the patient?

Пряма проба Кумбса Direct Coombs test

Рівень печінкових ферментів крові Hepatic blood enzyme level

Фіброгастроскопія Fibrogastroscopy

Непряма проба Кумбса Indirect Coombs Test

Рівень ціанокобаламіну крові Cyanocobalamin blood level

156 / 200
Серед причин смертності населення України переважають нещасні випадки, отруєння та травми, злоякісні новоутворення, хвороби системи кровообігу, ендокринні, органів дихання тощо. Який клас хвороб найбільш імовірно посідає перше рангове місце? Among the causes of mortality of the population of Ukraine, accidents, poisoning and injuries, malignant neoplasms, diseases of the circulatory system, endocrine, respiratory organs, etc. prevail. Which class of diseases is most likely to rank first rank place?

Ендокринні захворювання Endocrine diseases

Хвороби системи кровообіг*у Diseases of the circulatory system

Нещасні випадки, отруєння та травми Accidents, poisonings and injuries

Злоякісні новоутворення Malignant neoplasms

Хвороби органів дихання Diseases of respiratory organs

157 / 200
Мати з дівчинкою 11-ти років звернулись в приймальне відділення лікарні зі скаргами на виражений біль в правій здухвинній ділянці. При обстеженні виявлені зміни в сечі: протеїнурія. лейкоцитурія, бактеріурія. Ультразвукове дослідження (УЗД) показало відсутність правої нирки в типовому місці. Яке дослідження найдоцільніше провести для уточнення діагнозу виявленого під час УЗД? A mother and an 11-year-old girl went to the hospital's reception department with complaints of severe pain in the right sinus area. During the examination, changes in the urine were detected: proteinuria. leukocyturia, bacteriuria. An ultrasound examination (US) showed the absence of the right kidney in a typical location. What study is most appropriate to conduct to clarify the diagnosis revealed during the ultrasound?

Магнітно-резонансна томографія Magnetic resonance imaging

Оглядова урографія Review urography

Ниркова артеріографія Renal arteriography

Мікційна цистографія Victory cystography

Цистографія Cystography

158 / 200
Чоловік 23-х років звернувся до лікаря зі скаргами на наявність набряків обличчя, головні болі, запаморочення, зменшення виділення сечі, бурий колір сечі. Наведені скарги з'явились після перенесеного фарингіту. При фізикальному обстеженні температура тіла - 37,4°С, артеріальний тиск - 170/110 мм рт.ст., пульс - 86/хв., набряки на обличчі, шкірні покриви бліді. Аускультативно серцеві тони приглушені, акцент II тону над аортою. Які зміни найбільш імовірно будуть спостерігатись у загальному аналізі сечі? A 23-year-old man went to the doctor with complaints of facial swelling, headaches, dizziness, decreased urine output, brown urine. These complaints appeared after pharyngitis. During physical examination, body temperature is 37.4°C, blood pressure is 170/110 mm Hg, pulse is 86/min, swelling on the face, skin is pale. Auscultatory heart sounds are muffled, accent II tons above the aorta. What changes are most likely to be observed in the general urinalysis?

Лейкоцитурія, протеїнурія, епітеліальні циліндри Leukocyturia, proteinuria, epithelial cylinders

Еритроцитурія. оксалатурія. еритроцитарні циліндри Erythrocyturia. oxalaturia. erythrocyte cylinders

Еритроцитурія, лейкоцитурія, лейкоцитарні циліндри Erythrocyturia, leukocyturia, leukocyte cylinders

Еритроцитурія, протеїнурія, гіалінові циліндри Erythrocyturia, proteinuria, hyaline cylinders

Гемоглобінурія, білірубінурія, зернисті циліндри Hemoglobinuria, bilirubinuria, granular cylinders

159 / 200
Серед мешканців присадибних ділянок с. Іванівка, які прилягають до поля агрофірми 'Дружба', через 1 тиждень після зрошування його пестицидами (гексахлораном) з'явилися симптоми отруєння пестицидами. В результаті лабораторних досліджень вміст гексахлорану в ґрун ті сусідніх з полем приватних садиб перевищував гранично до- пустиму концентрацію в 2 рази, в повітрі - в 10 разів, у воді криниць (водопостачання населеного пункту є децентралізованим) - в 2 рази. Назвіть вид виливу пестицидів на організм людей, що має місце: Symptoms of pesticide poisoning appeared among the residents of the farmsteads of the village of Ivanivka, which are adjacent to the field of the agricultural company Druzhba, 1 week after it was irrigated with pesticides (hexachloran) As a result of laboratory studies, the content of hexachlorane in the soil of private estates adjacent to the field exceeded the maximum permissible concentration by 2 times, in the air by 10 times, and in well water (water supply of the settlement is decentralized) by 2 times. Name the type of spill of pesticides on the human body, which takes place:

Ізольований Isolated

Комплексний Comprehensive

Комбінований Combined

Поєднаний Combined

160 / 200
Жінка 32-х років звернулась до лікаря зі скаргами на відсутність вагітності впродовж 4-х років. У анамнезі: 5 років тому перша вагітність закінчилась штучним абортом. За даними вагінального дослідження та ультразвукового дослідження (УЗД) встановлено діагноз:* ендометріоїдна кіста правого яєчника. Який оптимальний метод лікування? A 32-year-old woman turned to the doctor with complaints about the absence of pregnancy for 4 years. In the anamnesis: 5 years ago, the first pregnancy ended in an artificial abortion. According to the data vaginal examination and ultrasound examination (ultrasound) established the diagnosis:* endometrioid cyst of the right ovary. What is the optimal method of treatment?

Терапія андрогенами Androgen therapy

Протизапальна терапія Anti-inflammatory therapy

Консервативна терапія естроген-гестагенними препаратами Conservative therapy with estrogen-gestagen drugs

Санаторно-курортне лікування Sanatorium-resort treatment

Оперативна лапароскопія Operative laparoscopy

161 / 200
Швидкою допомогою доставлена жінка з виниклим після затримки менструації переймоподібним болем в правій здухвинній ділянці, що іррадіює в пряму кишку, кров'янистими виділеннями зі статевих шляхів. Об'єктивно: частота серцевих скорочень - 100/хв., артеріальний тиск - 90/60 мм рт.ст. Шкірні покриви бліді. Живіт болючий при пальпації, позитивний симптом Щоткіна-Блюмберга. При гінекологічному дослідженні - зсуви шийки болісні, праві придатки збільшені, болючі, заднє склепіння нависає, виділення кров'янисті. Поставте попередній діагноз: A woman was delivered by ambulance with cramp-like pain in the right pubic area, radiating into the rectum, blood discharge from the genital tract, which arose after the delay of menstruation. Objectively : heart rate - 100/min., blood pressure - 90/60 mm Hg. Pale skin. Abdomen painful on palpation, positive symptom of Stotkin-Blumberg. During gynecological examination - cervical dislocations are painful, right appendages are enlarged, painful, the posterior vault overhangs, the discharge is bloody. Make a preliminary diagnosis:

Апендицит Appendicitis

Апоплексія правого яєчника Apoplexy of the right ovary

Гострий правобічний аднексит Acute right-sided adnexitis

Позаматкова вагітність, що перервалася Interrupted ectopic pregnancy

Аборт в ходу Abortion in progress

162 / 200
Чоловік, 52 роки, звернувся до лікаря зі скаргами на епізоди нестерпного болю у великому пальці руки. Під час збору анамнезу лікар виявив, що пацієнт нещодавно розпочав приймати гідрохлортіазид. Під час фізикального обстеження зліва в області проксимального міжфалангового суглобу 1 пальця виявлені припухлість, еритематозна та тепла на дотик шкіра. Лабораторний аналіз синовіальної рідини виявив кристали моноурату натрію. Яку рекомендацію має дати лікар стосовно дієти пацієнта? A 52-year-old man presented to the doctor complaining of episodes of excruciating pain in his thumb. During history taking, the doctor discovered that the patient had recently started taking hydrochlorothiazide. During the physical examination, the left proximal interphalangeal joint of the 1st finger revealed swelling, erythematous, and warm-to-the-touch skin. Laboratory analysis of the synovial fluid revealed sodium monourate crystals. What recommendation should the physician make regarding the patient's diet?

Зменшити вживання зернових продуктів Reduce consumption of grain products

Змін у дієті не потребує No need to change diet

Збільшити вживання овочів та фруктів Increase the consumption of vegetables and fruits

Збільшити вживання молочних продуктів Increase consumption of dairy products

Зменшити вживання м'ясних продуктів Reduce consumption of meat products

163 / 200
Потерпілий, робочий хімзаводу, працюючи з агресивною рідиною, по необережності вилив її собі на спецодяг, отримавши хімічний опік правого стегна та гомілки. Після зняття одягу виявлено: на передньо- внутрішній поверхні правого стегна, передній поверхні правої гомілки з переходом на тил стопи ділянки жовтаво-сірого кольору, місцями обривки епідермісу. Тактильна та больова гіпестезія уражених ділянок шкіри. Який з наступних кроків невідкладної допомоги має бути виконаний першочергово? The victim, a worker at a chemical plant, while working with an aggressive liquid, carelessly spilled it on his work clothes, receiving a chemical burn on his right thigh and lower leg. After removing the clothes, it was found: on the front - the inner surface of the right thigh, the front surface of the right lower leg with the transition to the back of the foot, areas of yellowish-gray color, in places of fragments of the epidermis. Tactile and painful hypesthesia of the affected areas of the skin. Which of the following steps of emergency care should be performed as a priority?

Накладання асептичної пов'язки Applying an aseptic bandage

Обробка розчином соди Treatment with soda solution

Накладання жирових пов'язок Applying fat bandages

Обробка етиловим спиртом Ethyl alcohol treatment

Промивання проточною водою Rinsing with running water

164 / 200
На репрезентативній сукупності хворих на грип проведено вивчення зв'язку між температурою тіла і частотою пульсу. Розрахований коефіцієнт кореляції дорівнює +0,5. Яка характеристика сили і направленості зв'язку між ознаками, що вивчаються є найбільш точною? The relationship between body temperature and pulse rate was studied on a representative population of flu patients. The calculated correlation coefficient is +0.5. What is the characteristic of the strength and directionality of 'is the connection between the signs being studied the most accurate?

Середній зворотній Average inverse

Сильний прямий Strong straight

Слабкий прямий Weak direct

Середній прямий Average straight

165 / 200
Жінка 35-ти років надійшла до відділення інтенсивної терапії з нападами судом з періодичністю 2-3 хвилини, між якими не відбувається покращення свідомості, реакція зіниць на світло відсутня. З дитинства хворіє на епілепсію. Зазвичай напади виникають 1-2 рази на місяць у нічний час та супроводжуються мимовільним сечовиділенням та дефекацією. Почастішання нападів та вищевказані ускладнення виникли на тлі перенесеного грипу. Який стан найбільш імовірно розвинувся у хворої? A 35-year-old woman came to the intensive care unit with convulsions with a frequency of 2-3 minutes, between which there is no improvement in consciousness, the reaction of the pupils to light is absent. She has suffered from epilepsy since childhood. Usually, attacks occur 1-2 times a month at night and are accompanied by involuntary urination and defecation. The frequency of attacks and the above-mentioned complications arose against the background of the flu. What condition most likely developed in the patient?

Епілептичний психоз Epileptic psychosis

Гіпокальціємічний криз Hypocalcemic crisis

- -

Епілептичний статус Status epilepticus

Істеричний невроз Hysterical neurosis

166 / 200
Чоловік 45-ти років, звернувся до лікаря зі скаргами на дратівливість, підвищену втомленість, схуднення, серцебиття, перебої в роботі серця. При пальпації лівої долі щитоподібної залози пальпується утворення, щільно-еластичної консистенції, безболісне. яке при ковтанні зміщується разом з щитоподібною залозою. Позитивні очні симптоми, екзофтальм, порушення конвергенції. Артеріальний тиск - 135/80 мм рт.ст., пульс - 110/хв. При ультразвуковому дослідженні щитоподібної залози у лівій долі візуалізується гіперехогенне округле утворення розмірами 2x3 см, з чіткими контурами, однорідної структури. Який висновок лікаря після проведеного обстеження буде найбільш коректним? A 45-year-old man went to the doctor with complaints of irritability, increased fatigue, weight loss, palpitations, interruptions in the work of the heart. When palpating the left lobe of the thyroid gland, palpable formation, dense-elastic consistency, painless. which when swallowed moves together with the thyroid gland. Positive eye symptoms, exophthalmos, convergence disorder. Blood pressure - 135/80 mm Hg, pulse - 110/min. During ultrasound examination of the thyroid gland in the left lobe, a hyperechoic rounded formation measuring 2x3 cm, with clear contours and a uniform structure is visualized. What conclusion of the doctor after the examination will be the most correct?

Рак щитоподібної залози Thyroid cancer

Кіста щитоподібної залози Thyroid cyst

Гострий тиреоїдит Acute thyroiditis

Вузол щитоподібної залози Thyroid nodule

Дифузно-токсичний зоб Diffuse toxic goiter

167 / 200
Хлопчика 5-ти років привели на прийом до лікаря зі скаргами на стрибкоподібне підвищення температури до 39.3°С протягом 12-ти днів. При фізикальному обстеженні виявлені: ін'єкція бульбарних кон'юнктив з обох сторін, потріскані червоні губи та 'малиновий' язик, шийна лімфаденопатія та еритематозний папульозний висип промежини, набряклість рук та ніг. Що з перерахованого лікар має призначити першочергово? A 5-year-old boy was brought to a doctor's appointment with complaints of a rapid rise in temperature to 39.3°C for 12 days. Physical examination revealed: other' ection of bulbar conjunctiva on both sides, cracked red lips and 'raspberry' tongue, cervical lymphadenopathy and erythematous papular rash of the perineum, swelling of the hands and feet. Which of the following should the doctor prescribe first?

Валацикловір перорально Valacyclovir Oral

Фуросемід перорально Furosemide Oral

Пеніцилін внутрішньовенно Penicillin IV

Преднізолон внутрішньовенно Prednisone IV

Імуноглобулін людини нормальний внутрішньовенно Human immunoglobulin normal IV

168 / 200
Хвора 50-ти років скаржиться на напад болю у правому підребер'ї, блювання з домішками жовчі, впродовж 5-ти років турбував біль у епігастрії, нудота, порушення випорожнення. Об'єктивно: частота серцевих скорочень - 92/хв. Підвищеної повноти, язик обкладений, іктеричні склери. Живіт м'який, болючий у проекції жовчного міхура, локальне м'язове напруження у правому підребер'ї, позитивний симптом Мерфі. В загальному аналізі крові: лейкоцити - 9,6 • 109/л, швидкість осідання еритроцитів - 14 мм/год. Яке дослідження доцільно призначити для підтвердження діагнозу в даному випадку? A 50-year-old patient complains of an attack of pain in the right hypochondrium, vomiting with bile impurities, for 5 years she has been bothered by pain in the epigastrium, nausea, disturbances defecation. Objectively: heart rate - 92/min. Increased fullness, coated tongue, icteric sclera. Abdomen is soft, painful in the projection of the gallbladder, local muscle tension in the right hypochondrium, positive Murphy's sign. B general blood test: leukocytes - 9.6 • 109/l, erythrocyte sedimentation rate - 14 mm/h. What research should be prescribed to confirm the diagnosis in this case?

Ретроградну холангіопанкреатографію Retrograde cholangiopancreatography

Бактеріологічне дослідження жовчі Bacteriological examination of bile

Ультразвукове дослідження жовчного міхура Ultrasound examination of the gallbladder

Холецистографію Cholecystography

Сцинтіграфію печінки Liver scintigraphy

169 / 200
Хвора 54-х років скаржиться на мерзлякуватість, сонливість, слабкість. В минулому лікувалася з приводу аутоімунного вузлового зоба. За рік її стан погіршився: з'явилася слабкість, набрала вагу, стала повільною. Щитоподібна залоза - І ст. Шкіра холодна, суха, бліда, язик потовщений. Голос захриплий. Тони серця ритмічні, глухі, частота серцевих скорочень - 56/хв., живіт здутий. Випорожнення - закрепи. Який препарат слід призначити для профілактики значного збільшення щитоподібної залози? A 54-year-old patient complains of chills, drowsiness, weakness. In the past, she was treated for autoimmune nodular goiter. Over the course of a year, her condition worsened: weakness appeared, gained weight, became sluggish. Thyroid gland - 1st stage. Skin cold, dry, pale, tongue thickened. Voice hoarse. Heart sounds rhythmic, dull, heart rate - 56/min., abdomen distended. Stools - constipation. What drug should be taken appoint for the prevention of a significant increase in the thyroid gland?

Радіоактивний йод Radioactive iodine

Метилтіоурацил Methylthiouracil

Мерказоліл Mercazolil

Тироксин Tyroxin

Преднізолон Prednisone

170 / 200
Жінка 58-ми років перебуває на стаціонарному лікуванні з приводу гіпертонічної хвороби. Вранці, після прийому каптоприлу та празозину різко піднялася з ліжка та короткочасно знепритомніла. Об'єктивно: шкіра бліда, волога. Пульс - 100/хв.. слабкий. артеріальний тиск - 70/40 мм рт.ст. На ЕКГ: ритм синусовий, правильний, ознаки перенавантаження лівого шлуночка. Змін з боку центральної нервової системи не виявлено. Яка причина стану хворої є найбільш імовірною? A 58-year-old woman is undergoing inpatient treatment for hypertension. In the morning, after taking captopril and prazosin, she suddenly got out of bed and briefly fainted. Objectively: the skin is pale, moist. Pulse - 100/min.. weak. blood pressure - 70/40 mm Hg. On the ECG: sinus rhythm, correct, signs of left ventricular overload. No changes in the central nervous system were detected. What is the cause of the condition patient is the most likely?

Анафілактичний шок Anaphylactic shock

Ортостатична гіпотензія Orthostatic hypotension

Кардіогенний шок Cardiogenic shock

Напад Морганьї-Адамса-Стокса Morganhi-Adams-Stokes attack

171 / 200
Хлопець 27-ми років під час купання в річці раптово знепритомнів та зник з поверхні води. Через 2 хвилини був доставлений до берега. Об'єктивно: непритомний, шкіра вираженого синюшного кольору, пінисті виділення з рота та носа, дихання та пульс на сонних артеріях відсутні. Який вид допомоги необхідно надати в цьому випадку? A 27-year-old boy while swimming in the river suddenly fainted and disappeared from the surface of the water. After 2 minutes, he was brought to the shore. Objectively: unconscious, skin pronounced bluish color, foamy secretions from the mouth and nose, breathing and pulse on the carotid arteries are absent. What kind of help should be provided in this case?

Екстрена медична допомога Emergency medical assistance

Паліативна медична допомога Palliative medical care

Третинна медична допомога Tertiary medical care

Первинна медична допомога Primary medical care

Вторинна медична допомога Secondary medical care

172 / 200
Дівчинка, 7 років, надійшла у відділення зі скаргами на біль у горлі, підвищення температури до 39°С. Під час огляду стан дитини важкий, шийні лімфатичні вузли до 1,5 см. При пальпації печінка на 3 см, селезінка на 2 см виступає з-під краю реберної дуги. При лабораторному дослідженні крові: еритроцити - 4.0 • 1012/л, Нb- 121 г/л, кольоровий показник - 0.9, тромбоцити - 190 • 109/л, лейкоцити - 19 • 109/л, е- 0, п/я- 1, с/я- 0, л- 87, м- 2, швидкість зсідання еритроцитів - 36 мм/год. Що з перерахованого буде найбільш доречним наступним кроком у веденні пацієнта? A 7-year-old girl came to the department with complaints of a sore throat, a temperature rise to 39°C. During the examination, the child's condition is serious, the cervical lymph nodes are 1.5 cm. During palpation, the liver is 3 cm, the spleen protrudes 2 cm from under the edge of the costal arch. During the laboratory blood test: erythrocytes - 4.0 • 1012/l, Hb - 121 g/l, color indicator - 0.9, platelets - 190 • 109/l, leukocytes - 19 • 109/l, e- 0, p/ya- 1, s/ya- 0, l- 87, m- 2, erythrocyte sedimentation rate - 36 mm/h. What about of the above would be the most appropriate next step in the management of the patient?

Спостереження протягом 2 тижнів Observation for 2 weeks

Аналіз крові на виявлення гетерофільних антитіл Blood analysis for detection of heterophilic antibodies

Дослідження кісткового мозку Bone marrow research

Повторний загальний аналіз крові через 1 тиждень Repeat general blood test in 1 week

173 / 200
Хвора 65-ти років госпіталізована в реанімаційне відділення в коматозному стані. Хворіє на цукровий діабет ІІ-го типу 10 років. Останні 2 тижні спостерігалися виражена поліурія, полідипсія. Глюкоза сироватки крові - 30 ммоль/л, рН артеріальної крові - 7,3. Осмолярність плазми крові - 350 мОсм/л. Хворій встановлено діагноз: діабетична гіперосмолярна кома. Який основний патогенетичний механізм коми? A 65-year-old patient was hospitalized in the intensive care unit in a comatose state. She has been suffering from type II diabetes for 10 years. In the last 2 weeks, severe polyuria and polydipsia were observed. Blood serum glucose - 30 mmol/l, arterial blood pH - 7.3. Blood plasma osmolality - 350 mOsm/l. The patient was diagnosed with diabetic hyperosmolar coma. What is the main pathogenetic mechanism of coma?

Підвищення канальцевої реабсорбції Increasing tubular reabsorption

Гіпокоагуляція Hypocoagulation

Дегідратація Dehydration

Підвищення клубочкової фільтрації Increase glomerular filtration

Гіпонатріємія Hyponatremia

174 / 200
Дівчинка 14-ти років звернулася до лікаря зі скаргами на відчуття стиснення у ділянці шиї, покашлювання, потовщення шиї. При фізикальному обстеженні щитоподібна залоза дифузно збільшена, щільна при пальпації, неболюча, поверхня гладка. При ультразвуковому дослідженні тканина щитоподібної залози неоднорідна. Яке лабораторне дослідження буде найбільш доречним? A 14-year-old girl went to the doctor with complaints of a feeling of compression in the neck area, coughing, thickening of the neck. On physical examination, the thyroid gland is diffusely enlarged, dense on palpation , painless, the surface is smooth. On ultrasound, the tissue of the thyroid gland is heterogeneous. What laboratory test would be most appropriate?

Титр антитіл до тиреоглобуліну Titer of antibodies to thyroglobulin

Вміст гормону росту Content of growth hormone

Вміст кальцитоніну Calcitonin content

Вміст йоду в добовій сечі Iodine content in daily urine

Вміст паратгормону Parathormone content

175 / 200
Мати хлопчика 6-ти місяців прийшла на прийом до педіатра стурбована тим, що її дитина не отримувала жодних щеплень. Скарг не мають. Об'єктивно: температура - 37.1 °С, частота дихання - 20/хв., артеріальний тиск - 100/70 мм рт.ст. Зріст та вага відповідають діапазону між 50 та 75 перцентилем. Відповідно до нормативно-правових документів МОЗ, якої тактики має дотримуватися лікар щодо імунізації цієї дитини? The mother of a 6-month-old boy came to see a pediatrician worried that her child had not received any vaccinations. They have no complaints. Objectively: the temperature is 37.1 °С, respiratory rate - 20/min, blood pressure - 100/70 mm Hg. Height and weight correspond to the range between the 50th and 75th percentile. According to the regulatory and legal documents of the Ministry of Health, what tactics should the doctor follow in the immunization of this child ?

Провести пробу Манту та записати на прийом через 2 дні Conduct a Mantoux test and make an appointment in 2 days

Ввести вакцину БЦЖ Enter BCG vaccine

Відкласти вакцинацію та записати на прийом через тиждень Postpone the vaccination and make an appointment in a week

Ввести вакцину БЦЖ, КПК та АКДС Inject BCG, CPC and DPT vaccine

176 / 200
При проведенні профогляду робітниць промислового підприємства була виявлена група жінок з лабораторно підтвердженою залізодефіцитною анемією. Окрім медикаментозного лікування, які продукти, як основні носії засвоюваного заліза, повинен рекомендувати лікар для споживання? During a professional examination of female workers at an industrial enterprise, a group of women with laboratory-confirmed iron-deficiency anemia was identified. In addition to medical treatment, which products, as the main carriers of absorbed iron, should the doctor recommend for consumption?

М'ясо та м'ясопродукти Meat and meat products

Овочі та фрукти Vegetables and fruits

Хліб та рибу Loaves and fishes

Хліб та хлібобулочні вироби Bread and bakery products

Молоко та молокопродукти Milk and milk products

177 / 200
Чоловік 18-ти років звернувся до лікаря зі скаргами на переймоподібний біль в животі, часті рідкі випорожнення з домішками слизу та свіжої крові, що тривають 4 місяці. За цей час схуд на 10 кг. При фізикальному обстеженні живіт м'який, болючий по ходу товстої кишки зліва, сигмоподібна кишка спазмована. При лабораторному дослідженні крові еритроцити - 3,2 • 1012/л, гемоглобін - 92 г/л, лейкоцити -10,6 • 109/л, ШЗЕ- 34 мм/год. Результат бак. посіву випорожнень негативний. На колоноскопїї виявлено запалення із втратою судинного малюнку, ділянка підвищеної кровоточивості 25 см з чіткими межами починаючи від краю анусу. Яка лікувальна тактика буде найбільш доречною? An 18-year-old man consulted a doctor with complaints of spasmodic abdominal pain, frequent loose stools with mucus and fresh blood, lasting 4 months. time lost weight by 10 kg. During physical examination, the abdomen is soft, painful along the course of the colon on the left, the sigmoid colon is spasmodic. During the laboratory blood test, erythrocytes - 3.2 • 1012/l, hemoglobin - 92 g/l, leukocytes -10, 6 • 109/l, SZE- 34 mm/h. The result of a stool culture is negative. Colonoscopy revealed inflammation with a loss of vascular pattern, an area of increased bleeding of 25 cm with clear borders starting from the edge of the anus. What treatment tactics would be most appropriate?

Повторний забір випорожнень на бактеріологічне дослідження Re-sampling of feces for bacteriological examination

Призначення преднізолону та месаламіну Prednisone and mesalamine prescription

Хірургічне лікування Surgical treatment

Призначення метронідазолу Prescription of metronidazole

Переливання еритроцитарної маси Red blood cell transfusion

178 / 200
Чоловік 31-го року доставлений до відділення невідкладної допомоги зі скаргами на утруднене дихання, відчуття стороннього тіла в горлі, осиплість, сильний набряк обличчя та шиї. Вищевказані симптоми швидко наростають та з'явились протягом декількох хвилин після куштування меду. Який стан найбільш імовірно розвинувся у хворого? A 31-year-old man was brought to the emergency department with complaints of difficulty breathing, feeling of a foreign body in the throat, dizziness, severe swelling of the face and neck. The above symptoms quickly build up and appeared within a few minutes after tasting the honey. Which condition most likely developed in the patient?

Запальний набряк гортані Inflammatory swelling of the larynx

Ангіоневротичний набряк Квінке Angioedema of Quincke

Токсичний епідермальний некроліз Toxic epidermal necrolysis

Синдром Стівенса-Джонсона Stevens-Johnson syndrome

Гострий ларинготрахеїт Acute laryngotracheitis

179 / 200
Жінка 36-ти років звернулася до лікаря зі скаргами на загальну слабкість, біль у животі та пожовтіння шкіри. Зазначені скарги поступово наростали протягом останніх 3-х місяців. Пацієнтка повідомила, що не мала серйозних захворювань в анамнезі. Вживання наркотиків та незахищені статеві стосунки заперечує. При фізикальному обстеженні іктеричність склер та жовтушність шкіри, болісність при пальпації правої підреберної ділянки. При лабораторному дослідженні сироватки крові: загальний білірубін - 64.5 мкмоль/л, прямий - 22,7 мкмоль/л, ACT- 822 Од/л. АЛТ- 1237 Од/л, HBsAg - позитивно, HBeAg - позитивно. Яка тактика лікування цієї хворої є найбільш доречною? A 36-year-old woman consulted a doctor with complaints of general weakness, abdominal pain, and yellowing of the skin. These complaints gradually increased over the past 3 months. The patient reported that she had no serious diseases in the anamnesis. Denies drug use and unprotected sex. On physical examination, scleral icterus and yellowness of the skin, pain on palpation of the right subcostal area. On laboratory examination of blood serum: total bilirubin - 64.5 μmol/l, direct - 22.7 μmol/l, ACT- 822 U/l. ALT- 1237 U/l, HBsAg - positive, HBeAg - positive. Which treatment tactics for this patient is the most appropriate?

ПЕГ-інтерферон альфа-2а PEG interferon alfa-2a

Преднізолон та рибавірин Prednisolone and Ribavirin

Вакцина для профілактики гепатиту В та ламівудин Hepatitis B vaccine and lamivudine

Імуноглобулін людини нормальний Human immunoglobulin is normal

Специфічний імуноглобулін проти вірусу гепатиту В Specific immunoglobulin against hepatitis B virus

180 / 200
Хвора 38-ми років звернулася до жіночої консультації зі скаргами на помірні кров'янисті виділення зі статевих шляхів, які виникли після затримки чергової менструації на 1,5 місяці. В ході вагінального обстеження: шийка матки не ерозована, симптом 'зіниці' (+++): матка не збільшена, щільна, рухома, неболюча; придатки з обох боків не збільшені, неболючі; склепіння глибокі. Який діагноз найімовірніший? A 38-year-old patient turned to a women's consultation with complaints of moderate bleeding from the genital tract, which occurred after the next menstruation was delayed for 1.5 months. During the vaginal examination: the cervix is not eroded, the 'pupil' symptom (+++): the uterus is not enlarged, dense, mobile, painless; the appendages on both sides are not enlarged, painless; the vaults are deep. What is the most likely diagnosis?

Рак тіла матки Cancer of the uterine body

Маткова вагітність Uterine pregnancy

Внутрішній ендометріоз тіла матки Internal endometriosis of the uterine body

Дисфункціональна маткова кровотеча Dysfunctional uterine bleeding

Позаматкова вагітність Ectopic pregnancy

181 / 200
Мати 22-місячного хлопчика звернулася до лікаря зі скаргами на погане набирання ваги та худобу у дитини. Годує грудним молоком на вимогу, не менше 5 разів на день. Вага та зріст хлопчика нижче 3-го стандартного відхилення для його віку. У нього дугоподібна деформація ніг та потовщені зап'ястя. При лабораторному дослідженні підвищений рівень лужної фосфатази. Дефіцит якого вітаміну найбільш імовірно спричинив подібний стан у дитини? The mother of a 22-month-old boy went to the doctor with complaints of poor weight gain and cattle in the child. Breastfeeds on demand, at least 5 times a day. Weight and the boy's height is less than 3 standard deviations for his age. He has bowed legs and thickened wrists. Laboratory tests show elevated alkaline phosphatase. Which vitamin deficiency is most likely to cause this condition in the child?

Вітаміну В6 Vitamin B6

Вітаміну РР Vitamin RR

Вітаміну D Vitamin D

Вітаміну А Vitamin A

Вітаміну Е Vitamin E

182 / 200
Чоловік 56-ти років, звернувся до лікаря з носовою кровотечею, що розпочалася з лівої ніздрі 30 хвилин тому. Будь-які травми носа пацієнт заперечує. В анамнезі: частих носових кровотеч немає, подібний епізод вперше. Хворіє на фібриляцію передсердь (приймає варфарин) та артеріальну гіпертензію (лікується гідрохлортіазидом, атенололом). Температура тіла - 37,2°С, пульс - 86/хв., артеріальний тиск - 120/70 мм рт.ст. Перед тим, як визначити необхідність тампонади носа, який першочерговий крок лікаря буде найбільш доречним? A 56-year-old man consulted a doctor with a nosebleed that started from the left nostril 30 minutes ago. The patient denies any injuries to the nose. In the anamnesis: there are no frequent nosebleeds, this is the first such episode. He has atrial fibrillation (takes warfarin) and hypertension (treated with hydrochlorothiazide, atenolol). Body temperature - 37.2°C, pulse - 86/min., blood pressure - 120/70 mm before determining the need for nasal tamponade, what would be the most appropriate first step of the doctor?

Ввести вітамін К внутрішньом'язово Inject vitamin K intramuscularly

Прикласти холодний компрес на перенісся Apply a cold compress to the bridge of the nose

Призначити каптонрил перорально Prescribe kaptonril orally

Оксиметазоліну інтраназально, попросити хворого затиснути крило носа та нахилитися вперед Oxymetazoline intranasally, ask the patient to pinch the wing of the nose and lean forward

183 / 200
Роділля 28-ми років доставлена до пологового будинку з бурхливою пологовою діяльністю. Пологи перші. Розміри таза: 23-25-28-18 см. Ознака Генкеля-Вастена позитивна. Роділля збуджена, живіт напружений, болісний в нижніх відділах. Контракційне кільце на рівні пупка, розташоване косо. Голівка плода притиснута до входу у малий таз. Серцебиття плоду - 140/хв. Яке ускладнення виникло у роділлі? A 28-year-old woman in labor was brought to the maternity hospital with violent labor. First delivery. Pelvic dimensions: 23-25-28-18 cm. Henkel-Wasten's sign positive. The woman in labor is excited, the abdomen is tense, painful in the lower parts. The contraction ring is at the level of the navel, located obliquely. The head of the fetus is pressed against the entrance to the small pelvis. The heart rate of the fetus is 140/min. What complication occurred in the woman in labor?

Загроза розриву матки Threat of uterine rupture

Завершений розрив матки Completed uterine rupture

Дискоординація пологової діяльності Discoordination of labor

Розпочатий розрив матки Started rupture of the uterus

Надмірна пологова діяльність Excessive reproductive activity

184 / 200
Породілля 27-ми років, пологи II, термінові. нормальні. З доба післяпологового періоду. Температура тіла - 36,8°6\ пульс - 72/хв., артеріальний тиск - 120/80 мм рт.ст. Молочні залози помірно нагрублі, соски чисті. Живіт м'який, безболісний. Дно матки на 3 п/п нижче пупка. Jloxiï кров'янисті, помірні. Який можна встановити діагноз? 27-year-old woman in labor, labor II, urgent. normal. Since the postpartum period. Body temperature - 36.8°6\ pulse - 72/min. , blood pressure - 120/80 mm Hg. The mammary glands are moderately thickened, the nipples are clean. The abdomen is soft, painless. The bottom of the uterus is 3 p/p below the navel. Jloxiï are bloody, moderate. What can be diagnosed?

Фізіологічний перебіг післяпологового періоду Physiological course of the postpartum period

Лактостаз Lactostasis

Субінволюція матки Subinvolution of uterus

Післяпологовий метроендометрит Postpartum metroendometritis

Залишки плацентарної тканини після пологів Remains of placental tissue after childbirth

185 / 200
Пацієнтка 22-х років звернулася до жіночої консультації зі скаргами на затримку менструації протягом 1,5 місяців, нудоту, втомлюваність, сонливість, дратівливість. В ході огляду на її обличчі та сосках виявлена виражена пігментація. З боку внутрішніх органів патології не виявлено. Під час огляду в дзеркалах визначається ціаноз слизової оболонки піхви та шийки матки; в ході бімануального дослідження - збільшення матки, її гіперантефлексія, асиметрія. З чим найімовірніше пов'язані перераховані скарги та дані бімануального дослідження? A 22-year-old patient turned to a women's consultation with complaints of delayed menstruation for 1.5 months, nausea, fatigue, drowsiness, irritability. During the examination of her pronounced pigmentation was found on the face and nipples. No pathology was found on the part of the internal organs. During the examination in the mirrors, cyanosis of the mucous membrane of the vagina and cervix was determined; during the bimanual examination - the enlargement of the uterus, its hyperanteflexion, asymmetry. What are the most likely reasons for the listed complaints and bimanual research data?

Маткова вагітність Uterine pregnancy

Ектопічна вагітність Ectopic pregnancy

Пухлина матки Uterine tumor

Порушення менструального циклу Menstrual cycle disorder

Захворювання шлунково-кишкового тракту Diseases of the gastrointestinal tract

186 / 200
Жінка 35-ти років звернулася до лікаря зі скаргами на зростаючий біль під час менструації протягом 1-го року. Тривалість менструального циклу 28 днів. 2 роки тому була проведена лапароскопічна перев'язка маткових труб. При пальпації матка збільшена до 8 тижнів вагітності, м'якої консистенції, болюча. Придатки з обох сторін не збільшені, при пальпації безболісні. Який діагноз є найбільш імовірним? A 35-year-old woman consulted a doctor with complaints of increasing pain during menstruation during the 1st year. The length of the menstrual cycle is 28 days. 2 years ago she was laparoscopic tubal ligation. On palpation, the uterus is enlarged up to 8 weeks of pregnancy, soft in consistency, painful. The appendages on both sides are not enlarged, on palpation they are painless. What diagnosis is most likely?

Гідросальпінкс Hydrosalpinx

Позаматкова вагітність Ectopic pregnancy

Аденоміоз Adenomyosis

Зовнішній ендометріоз External endometriosis

Полікістоз яєчників Polycystic ovary

187 / 200
Хлопчик 8-ми років, звернувся до лікаря зі скаргами на підвищену втомлюваність, зниження апетиту, іктеричність склер, біль у животі. У періоді новонародженості була затяжна жовтяниця. У фізичному розвитку не відстає. Об'єктивно: шкіра бліда, слизові оболонки і склери іктеричні. Печінка +2 см, злегка болюча при пальпації. Випорожнення і сечовиділення в нормі. При лабораторному дослідженні крові: еритроцити - 4,5 • 1012/л. гемоглобін - 115 г/л, лейкоцити - 7 • 109/л, швидкість осідання еритроцитів - 8 мм/год, білірубін прямий - 10 мкмоль/л. непрямий - 39 мкмоль/л, аспартатамінотрансфераза (АСТ) - 17 Од/л, аланінамінотрансфераза (АЛТ) - 21 Од/л. Який діагноз є найбільш імовірним? An 8-year-old boy turned to the doctor with complaints of increased fatigue, decreased appetite, icterus of the sclera, abdominal pain. In the newborn period, he had prolonged jaundice. In does not lag behind in physical development. Objectively: the skin is pale, mucous membranes and sclera are icteric. Liver +2 cm, slightly painful on palpation. Defecation and urination are normal. In the laboratory blood test: erythrocytes - 4.5 • 1012/l. hemoglobin - 115 g/l, leukocytes - 7 • 109/l, erythrocyte sedimentation rate - 8 mm/h, direct bilirubin - 10 μmol/l, indirect - 39 μmol/l, aspartate aminotransferase (AST) - 17 U/l, alanine aminotransferase ( ALT) - 21 units/l. What diagnosis is the most probable?

Хронічний вірусний гепатит Chronic viral hepatitis

Гемолітична анемія Hemolytic anemia

Синдром Жильбера Gilbert syndrome

Дискінезія жовчовивідних шляхів Biliary tract dyskinesia

188 / 200
35-річного чоловіка доставлено до приймального відділення у непритомному стані. Його дружина повідомила, що він втратив свідомість після відкриття крану з водою. За день до цього протягом тижня чоловік скаржився на запаморочення, слабкість та порушення координації. Працює спелеологом, проводить екскурсії печерами. Під час лікування у стаціонарі, спостерігалися постійні парестезії, дисфагія, дезорієнтація та атаксія. Стан швидко прогресував. пацієнт почав нерозбірливо говорити, приєдналися галюцинації, ажитація, що потребували седації та інтубації пацієнта. На 14-й день госпіталізації пацієнт помер. Вакцинація від якого збудника найбільш імовірно попередила б смерть цього пацієнта? A 35-year-old man was brought to the reception department in an unconscious state. His wife reported that he lost consciousness after opening the water tap. A day before during the week, the man complained of dizziness, weakness and incoordination. Works as a speleologist, conducts cave tours. During hospital treatment, persistent paresthesias, dysphagia, disorientation and ataxia were observed. The condition progressed rapidly. The patient began to speak indistinctly, accompanied by hallucinations, agitation, which required sedation and intubation of the patient. On the 14th day of hospitalization, the patient died. Vaccination against which pathogen would most likely have prevented the death of this patient?

Вірусу японського енцефаліту Japanese encephalitis virus

Вірусу кліщового енцефаліту Tick-borne encephalitis virus

Вакцина від цього збудника не розроблена A vaccine against this pathogen has not been developed

Вірусу поліомієліту Polio virus

Вірусу сказу Rabies virus

189 / 200
26-річна вагітна жінка доставлена до відділення невідкладної терапії на 36-му тижні вагітності зі скаргами на інтенсивний головний біль у лобній ділянці. При фізикальному обстеженні: артеріальний тиск - 170/90 мм рт.ст., пульс - 85/хв., частота дихання - 15/хв., температура - 36,9°С, набряки кінцівок. Серцебиття плоду - 159/хв. Під час огляду у жінки розвиваєтьсянапад генералізованих тоніко-клонічних судом. Який препарат лікар має ввести першочергово? A 26-year-old pregnant woman was brought to the emergency department at the 36th week of pregnancy with complaints of an intense headache in the frontal area. On physical examination: blood pressure - 170/90 mm Hg, pulse - 85/min, respiratory rate - 15/min, temperature - 36.9°С, edema of the extremities. Fetal heart rate - 159/min. During the examination, the woman develops an attack of generalized tonics - clonic convulsions. What drug should the doctor administer first?

Ламотриджин Lamotrigine

Діазепам Diazepam

Фенітоїн Phenytoin

Магнію сульфат Magnesium sulfate

Натрію вальпроат Sodium valproate

190 / 200
Раніше здоровий хлопець 22-х років прийшов на прийом до лікаря зі скаргами на підвищену температуру та кашель з жовтим мокротинням, що продовжуються протягом 7-ми днів. При фізикальному обстеженні температура - 38,3°С, артеріальний тиск - 130/70 мм рт.ст.. частота серцевих скорочень - 79/хв., частота дихання - 17/хв., SpO2 - 95% при кімнатному повітрі. При аускультації звучні вологі хрипи зліва. На оглядовій рентгенограмі консолідація у нижній частці лівої легені. Який з перерахованих препаратів буде найбільш доцільно призначити цьому пацієнту? A previously healthy 22-year-old boy came to the doctor with complaints of high temperature and cough with yellow sputum, which continued for 7 days. With physical examination temperature - 38.3°C, blood pressure - 130/70 mm Hg, heart rate - 79/min, respiratory rate - 17/min, SpO2 - 95% in room air. Upon auscultation, sonorous wet wheezing on the left. Consolidation in the lower lobe of the left lung on the X-ray examination. Which of the listed drugs would be the most appropriate to prescribe for this patient?

Левофлоксацин Levofloxacin

Ципрофлоксацин Ciprofloxacin

Триметоприм-сульфаметоксазол Trimethoprim-sulfamethoxazole

Гентаміцин Gentamicin

Азитроміцин Azithromycin

191 / 200
До жіночої консультації звернулася жінка 40-ка років зі скаргами на порушення менструального циклу за типом гіперполіменореї протягом півроку, тягнучі болі в нижніх відділах живота, слабкість. В ході гінекологічного обстеження тіло матки збільшене до 12 тижнів вагітності, щільне, рухоме, безболісне. В крові: Нb - 90 г/л. Яка патологія є найбільш імовірною? A 40-year-old woman came to a women's consultation with complaints of irregular menstrual cycles in the form of hyperpolymenorrhea for six months, pulling pains in the lower abdomen, weakness. During a gynecological examination, the body of the uterus is enlarged up to 12 weeks of pregnancy, dense, mobile, painless. In the blood: Hb - 90 g/l. What pathology is the most probable?

Вагітність Pregnancy

Дисфункціональна маткова кровотеча Dysfunctional uterine bleeding

Кістома яєчника Ovarian cystoma

Рак тіла матки Cancer of the uterine body

Міома матки Uterine myoma

192 / 200
Пацієнтка 20-ти років проходить лікування з приводу анемії (гемоглобін - 72 г/л). Півтора роки тому після мимовільного викидня у терміні 16 тижнів та крововтрати, відмічає зниження пам'яті, втомлюваність, втрату апетиту, сухість шкіри, ламкість нігтів. набряклість, порушення менструальної функції. Об'єктивно: артеріальний тиск - 80/55 мм рт.ст.. пульс - 54/хв., зріст - 168 см. вага - 48 кг, гіпоплазія статевих органів. Призначення якого з перерахованих препаратів буде найбільш доречним цій пацієнтці? A 20-year-old female patient is being treated for anemia (hemoglobin - 72 g/l). One and a half years ago, after an involuntary miscarriage at 16 weeks and blood loss, she notes decreased memory, fatigue, loss of appetite, dry skin, brittle nails, swelling, menstrual dysfunction Objectively: blood pressure - 80/55 mm Hg, pulse - 54/min, height - 168 cm. weight - 48 kg, genital hypoplasia. Which of the listed drugs would be most appropriate for this patient?

Метотрексат Methotrexate

Інфліксимаб Infliximab

Гідроксихлорохін Hydroxychloroquine

Імуноглобулін людини нормальний Human immunoglobulin is normal

Гідрокортизон Hydrocortisone

193 / 200
Хлопчика 7-ми років, привели до лікаря зі скаргами на виражений тотальний ціаноз з фіолетовим відтінком, задишку, яка полегшується в положенні навпочіпки з притискуванням до живота колін. При огляді пальці у вигляді 'барабанних паличок' та нігті як 'годинникові скельця', аускультативно дуючий шум над легеневою артерією. При інструментальному дослідженні на електрокардіограмі вертикальна вісь серця. На ехокардіографії стеноз отвору легеневої артерії, гіпертрофія стінок правого шлуночка, дефект міжшлуночкової перегородки, декстрапозиція аорти. Який діагноз є найбільш імовірним? A 7-year-old boy was brought to the doctor with complaints of pronounced total cyanosis with a purple hue, shortness of breath, which is relieved in a squatting position with knees pressed to the abdomen. on examination, fingers in the form of 'drumsticks' and nails as 'watch glasses', auscultatory blowing noise over the pulmonary artery. During instrumental examination on the electrocardiogram, the vertical axis of the heart. On echocardiography, stenosis of the opening of the pulmonary artery, hypertrophy of the walls of the right ventricle, defect of the interventricular septum, dextraposition of the aorta . What diagnosis is the most probable?

Тетрада Фалло Tetrad of Fallot

Транспозиція магістральних судин Transposition of trunk vessels

Дефект міжшлуночкової перегородки Ventricular septal defect

Стеноз отвору легеневої артерії Stenosis of the opening of the pulmonary artery

Відкрита артеріальна протока Open ductus arteriosus

194 / 200
Давність виникнення синців можна визначити за: The age of bruising can be determined by:

За кількістю синців By number of bruises

Зміною форми синця By changing the shape of the bruise

Температурою шкіри в синці Skin temperature in the bruise

Забарвленням синця The color of the bruise

Швидкістю поширення крові по підшкірній клітковині By the speed of blood spreading through the subcutaneous tissue

195 / 200
З дому надійшла дитина 3-х років, у якої на тлі гігіертермічного синдрому, спричиненого грипом, протягом 50-ти хвилин тривають тоніко-клонічні генералізовані судоми. Терапія не проводилася. Для негайного лікування судомного синдрому необхідно: A 3-year-old child came from home, who, against the background of hygerthermic syndrome caused by influenza, has been having tonic-clonic generalized convulsions for 50 minutes. Therapy is not For the immediate treatment of a convulsive syndrome, it is necessary:

Призначити парацетамол у високих дозах Prescribe paracetamol in high doses

Ввести фенобарбітал внутрішньом'язово Inject phenobarbital intramuscularly

Терміново накласти дитині холодні компреси Urgently apply cold compresses to the child

Ввести кальцію глюконат внутрішньовенно Enter calcium gluconate intravenously

Ввести лоразепам внутрішньовенно Inject lorazepam intravenously

196 / 200
Жінка 65-ти років, вранці після сну відмітила затерплість в правих кінцівках, запаморочення, гикавку, зміну голосу, похлинання при вживанні рідини, затерплість та біль у лівій половині обличчя, хиткість ходи. Останні показники артеріального тиску-160/100 мм рт.ст. У анамнезі гіпертонічна хвороба (систематично не лікується). При фізикальному обстеженні горизонтальний середньоамплітудний ністагм, більше виражений вліво, синдром Горнера ліворуч, гіпестезія лівої половини обличчя за цибулинним типом, правобічна гемігінестезія, атаксія в позі Ромберга. Мова змінена - нечітка артикуляція, проте зміст і розуміння мови не порушені. Пацієнтці важко висунути язик з рота. Парезів кінцівок немає. Який метод буде найбільш доречним у підтверджені діагнозу цієї пацієнтки? A 65-year-old woman, in the morning after sleep, noted numbness in the right limbs, dizziness, hiccups, voice change, swallowing when drinking liquid, numbness and pain in the left half face, unsteadiness of gait. Recent blood pressure readings - 160/100 mm Hg. History of hypertension (not systematically treated). Physical examination revealed horizontal mid-amplitude nystagmus, more pronounced on the left, Horner's syndrome on the left, bulbous-type hypoesthesia of the left half of the face , right-sided hemigynaesthesia, ataxia in Romberg's position. The language is changed - unclear articulation, but the meaning and understanding of speech is not disturbed. The patient has difficulty sticking the tongue out of the mouth. There are no paresis of the limbs. What method will be most appropriate in confirming the diagnosis of this patient?

Ангіографія Angiography

Електроенцефалографія Electroencephalography

Комп'ютерна томографія (КТ) головного мозку Computed tomography (CT) of the brain

Реоенцефалографія Rheoencephalography

Магнітно-резонансна томографія (МРТ) головного мозку Magnetic resonance imaging (MRI) of the brain

197 / 200
Чоловік 52-х років надійшов до відділення невідкладної допомоги зі скаргами на раптову задишку та прискорене серцебиття. Протягом місяця лікувався у стаціонарі з приводу септичного шоку, що розвинувся після інфекції сечовивідних шляхів. При фізикальному обстеженні артеріальний тиск - 100/60 мм рт.ст., пульс - 128/хв., частота дихання - 28/хв.. Sp02- 89%. При лабораторному дослідженні у сироватці крові креатинін - 96 мкмоль/л, тропонін 1-1,0 нг/мл (норма <0,1 нг/мл). На рентгенограмі органів грудної порожнини незначний гідроторакс праворуч та субсегментарний ателектаз. На ЕКГ: синусова тахікардія, блокада правої ніжки пучка Гіса та інверсія зубців Т у відведеннях III та VI. Який перший крок у веденні пацієнта буде найбільш доречним? A 52-year-old man presented to the emergency department with complaints of sudden shortness of breath and palpitations. He had been hospitalized for a month for septic shock that developed after an infection urinary tract. During physical examination, blood pressure - 100/60 mm Hg, pulse - 128/min, respiratory rate - 28/min. Sp02 - 89%. During laboratory examination, creatinine in blood serum - 96 μmol/l , troponin 1-1.0 ng/ml (norm <0.1 ng/ml). Chest x-ray shows slight hydrothorax on the right and subsegmental atelectasis. ECG: sinus tachycardia, right bundle branch block and inversion of T waves in leads III and VI. What would be the most appropriate first step in patient management?

Призначення аспірину та внутрішньовенне введення метопрололу Prescription of aspirin and intravenous metoprolol

Компресійна ультрасонографія вен Compression ultrasonography of veins

Ехокардіографія Echocardiography

Невідкладна катетеризація серця Urgent cardiac catheterization

КТ-ангіографія CT-angiography

198 / 200
Дівчина 18-ти років звернулася ввечері до лікаря зі скаргами на сильний біль 'знизу справа' нудоту та одиничний епізод блювання. Зранку відвідувала фестиваль вуличної їжі. Біль розпочався 4 години тому, почався з епігастрію і поступово перейшов у праву здухвинну ділянку. При фізикальному обстеженні артеріальний тиск - 110/70 мм рт.ст.. частота серцевих скорочень - 79/хв., частота дихання - 16/хв., температура - 37,4°С. При пальпації лівого нижнього квадранту черевної стінки, біль виникає у правому. Гінекологічний та ректальний огляд без патологічних змін. Дані лабораторних досліджень: лейкоцити - 12 • 109л, еритроцити - 4,4 • 1012/л, гемоглобін - 130 г/л, гематокрит - 39%, тромбоцити - 225 • 109/л, в-ХГЛ - 2 мОд/мл (норма - менше 5 мОд/л у невагітних жінок). Який діагноз є найбільш імовірним? An 18-year-old girl went to the doctor in the evening with complaints of severe pain in the lower right side, nausea and a single episode of vomiting. In the morning, she attended a street food festival. The pain started 4 hours ago, started from the epigastrium and gradually moved to the right paranasal area. On physical examination, blood pressure - 110/70 mm Hg., heart rate - 79/min., respiratory rate - 16/min., temperature - 37, 4°С. When palpating the left lower quadrant of the abdominal wall, pain occurs in the right. Gynecological and rectal examination without pathological changes. Laboratory data: leukocytes - 12 • 109l, erythrocytes - 4.4 • 1012/l, hemoglobin - 130 g/ l, hematocrit - 39%, platelets - 225 • 109/l, v-hCG - 2 mU/ml (norm - less than 5 mU/l in non-pregnant women). What diagnosis is most probable?

Гострий апендицит Acute appendicitis

Емболія мезентеріальних судин Mesenteric vessel embolism

Кишкова непрохідність Intestinal obstruction

Харчова токсикоінфекція Food poisoning

Трубна вагітність Tapal pregnancy

199 / 200
Хвора 22-х років скаржиться на часте і болюче сечовипускання, поклики на сечопуск вночі, нетримання сечі, біль у надлобковій ділянці та у попереку. Часто сеча має колір пива. Місяць тому вийшла заміж. Об'єктивно: загальний стан задовільний. В легенях - везикулярне дихання. Тони серця ритмічні,частота серцевих скорочень - 78/хв., артеріальний тиск - 128/68 мм рт.ст. Живіт м'який, болючий у надлобковій ділянці. У сечі: еритроцити -12-18 в полі зору, лейкоцити - 12-15, бактерій +++ в полі зору. Який найбільш імовірний діагноз? A 22-year-old patient complains of frequent and painful urination, calls to urinate at night, urinary incontinence, pain in the suprapubic area and in the lower back. The urine is often the color of beer . Got married a month ago. Objectively: the general condition is satisfactory. In the lungs - vesicular breathing. Heart sounds are rhythmic, heart rate - 78/min., blood pressure - 128/68 mm Hg. Abdomen is soft, painful in the suprapubic area. In the urine: erythrocytes -12-18 in the field of view, leukocytes - 12-15, bacteria +++ in the field of view. What is the most likely diagnosis?

Гонорея Gonorrhea

Первинний сифіліс Primary syphilis

Інфекція верхніх сечових шляхів - пієлонефрит Upper urinary tract infection - pyelonephritis

Інфекція нижніх сечових шляхів - цистит Lower urinary tract infection - cystitis

Сечокам'яна хвороба Urolithiasis

200 / 200
Пацієнт 28-ми років скаржиться на періодичні болі в ногах та руках, мерзлякуватість стоп, зміну кольору 3 пальців кистей. Болі тривають вже протягом 1 року, але зміна кольору пальців виникла вперше. Він викурює по 20 сигарет в день протягом 12-ти років. Об'єктивно: шкіра ніг бліда, прохолодна, тургор знижений, гіпотрихоз. Який діагноз є найбільш імовірним? A 28-year-old patient complains of periodic pains in the legs and arms, frostbite of the feet, and discoloration of 3 fingers. The pain has lasted for 1 year, but the discoloration fingers appeared for the first time. He has been smoking 20 cigarettes a day for 12 years. Objectively: the skin of the legs is pale, cool, turgor is reduced, hypotrichosis. What diagnosis is most probable?

- -

Облітеруючий атеросклероз Obliterating atherosclerosis

Облітеруючий тромбангіїт Thrombangiitis obliterans

Діабетична нейропатія Diabetic neuropathy

Системна склеродермія Systemic scleroderma